09.01.2015 Views

Revista (format .pdf, 1 MB) - Recreaţii Matematice

Revista (format .pdf, 1 MB) - Recreaţii Matematice

Revista (format .pdf, 1 MB) - Recreaţii Matematice

SHOW MORE
SHOW LESS

You also want an ePaper? Increase the reach of your titles

YUMPU automatically turns print PDFs into web optimized ePapers that Google loves.

Anul XI, Nr. 1<br />

Ianuarie – Iunie 2009<br />

RECREAŢ II<br />

MATEMATICE<br />

REVISTĂ DE MATEMATICĂ PENTRU ELEVI Ş I PROFESORI<br />

e i 1<br />

Asociaţ ia “Recreaţ ii <strong>Matematice</strong>”<br />

IAŞ I - 2009


Semnificaţia formulei de pe copertă:<br />

Într-o formă concisă, formula<br />

i<br />

e 1<br />

leagă cele patru ramuri fundamentale ale<br />

matematicii:<br />

ARITMETICA reprezentată de 1<br />

GEOMETRIA<br />

reprezentată de <br />

ALGEBRA<br />

reprezentată de i<br />

ANALIZA MATEMATICĂ<br />

reprezentată de e<br />

Redacţia revistei :<br />

Petru ASAFTEI, Dumitru BĂTINEŢU-GIURGIU (Bucureşti), Temistocle BÎRSAN, Dan<br />

BRÂNZEI, Alexandru CĂRĂUŞU, Constantin CHIRILĂ, Eugenia COHAL, Adrian COR-<br />

DUNEANU, Mihai CRĂCIUN (Paşcani), Paraschiva GALIA, Paul GEORGESCU, Mihai<br />

HAIVAS, Gheorghe IUREA, Lucian - Georges LĂDUNCĂ, Mircea LUPAN, Gabriel MÎRŞANU,<br />

Andrei NEDELCU, Alexandru NEGRESCU (student, Iaşi), Gabriel POPA, Dan POPESCU<br />

(Suceava), Florin POPOVICI (Braşov), Maria RACU, Neculai ROMAN (Mirceşti), Ioan<br />

SĂCĂLEANU (Hârlău), Ioan ŞERDEAN (Orăştie), Dan TIBA (Bucureşti), Marian TETIVA<br />

(Bârlad), Lucian TUŢESCU (Craiova), Adrian ZANOSCHI, Titu ZVONARU (Comăneşti).<br />

COPYRIGHT © 2008, ASOCIAŢIA “RECREAŢII MATEMATICE”<br />

Toate drepturile aparţin Asociaţiei “Recreaţii <strong>Matematice</strong>”. Reproducerea integrală sau parţială<br />

a textului sau a ilustraţiilor din această revistă este posibilă numai cu acordul prealabil scris al<br />

acesteia.<br />

TIPĂRITĂ LA SL&F IMPEX IAŞI<br />

Bd. Carol I, nr. 3-5<br />

Tel. 0788 498933<br />

E-mail: simonaslf@yahoo.com<br />

ISSN 1582 - 1765


O sută de ani de la naşterea<br />

Academicianului Nicolae Teodorescu<br />

Am ales matematica dintr-un îndemn lăuntric, pentru că mă<br />

atrăgeau orizonturile abia întrevăzute ale acestei ştiinţe cu adânci<br />

rădăcini în realitate şi cu aripi neasemuite pentru zborul minunat<br />

spre adevărul şi frumosul universal. (N. Teodorescu)<br />

Istoria matematicii româneşti îl aşează pe<br />

Nicolae Teodorescu printre marii continuatori<br />

ai celor care au pus bazele învăţământului şi<br />

cercetării matematice din ţara noastră: Spiru<br />

Haret, M. Davidoglu, Gh. Ţiţeica, Tr. Lalescu,<br />

D. Pompeiu, Al. Myller, O. Onicescu ş.a.<br />

Nicolae Teodorescu a adus o contribuţie importantă<br />

la crearea şcolii matematice româneşti,<br />

fiind unul dintre marii matematicieni de la mijlocul<br />

secolului trecut care au predat la Facultatea de<br />

matematică a Universităţii din Bucureşti: S. Stoilow,<br />

Gr. Moisil, M. Nicolescu, Gh. Mihoc, V. Vâlcovici,<br />

Gh. Vrănceanu, C. Iacob, Al. Ghika ş.a.<br />

S-a născut la 18 iulie 1908, în Bucureşti. A urmat<br />

cursurile Liceului ”Spiru Haret” trecând bacalaureatul<br />

în 1926, la Seminarul pedagogic ”Titu<br />

Maiorescu”. În perioada liceului rezolvă probleme<br />

la ”Gazeta Matematică”, ”<strong>Revista</strong> matematică din Timişoara”, ”Curierul matematic”,<br />

”Vlăstarul” şi publică note matematice privind gradul unui polinom, dreapta<br />

lui Simson şi poziţia rădăcinilor trinomului de gradul al doilea. Este licenţiat al secţiei<br />

de matematică de la Facultatea de ştiinţe a Universităţii Bucureşti, din 1929. În 1931,<br />

susţine la Paris teza de doctorat La derivée aréolaire et ses applications à la Physique<br />

mathématique în faţa unei comisii <strong>format</strong>ă din Henri Vilat, specialist în mecanica<br />

fluidelor (preşedinte), Armand Denjoy, reprezentant de seamă al şcolii franceze de<br />

teoria funcţiilor, şi H. Béyghin, la lucrările comisiei fiind invitat şi Dimitrie Pompeiu.<br />

Apreciind valoarea deosebită a lucrării, comisia îl propune pe un post de profesor<br />

universitar în Maroc (care făcea parte din Imperiul francez). Se întoarce în România<br />

unde devine, în 1931, asistentul acad. Octav Onicescu la catedra de macanică şi este<br />

numit profesor la Şcoala de conductori arhitecţi şi Institutul de statistică şi actuariat<br />

condus de acad. Gheorghe Mihoc.<br />

Nicolae Teodorescu a fost numit profesor titular la Catedra de geometrie descriptivă<br />

de la Facultatea de arhitectură a Institutului Politehnic Bucureşti, în 1942.<br />

A funcţionat ca şef de catedră la Universitate, Institutul Politehnic şi Institutul de<br />

Construcţii din Bucureşti.<br />

A iniţiat şi dezvoltat predarea disciplinei ecuaţiile fizicii matematice şi a scris un<br />

curs, ce a apărut în cinci ediţii succesive (1956-1978).<br />

A fost membru fondator al Institutului de Matematică al Academiei Române. În<br />

1955 devine membru corespondent, iar din 1973 membru titular al Academiei Române.<br />

1


A fost decan al Facultăţii de matematică-mecanică de la Universtatea Bucureşti<br />

(1960-1972) şi director al Centrului de calcul de la Universitatea Bucureşti.<br />

Nicolae Teodorescu s-a ocupat în teza de doctorat şi în lucrări ulterioare de<br />

noţiunea de derivată areolară introdusă de Dimitrie Pompeiu în a doua decadă a secolului<br />

trecut. Înlocuieşte definiţia clasică a noţiunii de derivată areolară cu o definiţie<br />

bazată pe teoria integralei Lebesgue, iniţiind aplicaţiile acestei teorii în mecanică, geometrie<br />

diferenţială şi diverse generalizări. Este unul dintre primii matematicieni care<br />

utilizează noţiunea de soluţie slabă (sau generalizată), în teoria ecuaţiilor cu derivate<br />

parţiale. A avut contribuţii deosebite în teoria propagării undelor, teoria fronturilor<br />

de undă, matematizarea principiului lui Huygens. Este primul matematician care a<br />

sesizat importanţa utilizării spaţiilor Finsler. Rezultatele sale au fost aprofundate şi<br />

extinse de matematicieni ca Vekua, Bitzadze ş.a.<br />

A fost Doctor Honoris Causa al Universităţii din Caen, membru al academiilor<br />

din Palermo şi Mesina (din 1967), secretar general al Uniunii Balcanice a Matematicienilor<br />

şi a fost decorat cu Ordinul Metodiu şi Chiril al Bulgariei.<br />

Timp de peste 75 de ani, Nicolae Teodorescu a desfăşurat o activitate susţinută<br />

în cadrul Societăţii de Ştiinţe <strong>Matematice</strong> din România. A fost vicepreşedinte al<br />

S.S.M. în perioada 1948-1973 (preşedinte fiind acad. Grigore C. Moisil), preşedinte<br />

al S.S.M. în perioada 1973-1995 şi preşedinte de onoare din 1995 până la stingerea sa<br />

din viaţă. În 1980 a înfiinţat Gazeta Matematică metodică şi metodologică, pentru<br />

profesori. A restructurat conţinutul Gazetei <strong>Matematice</strong> adăugând rubrici noi: in<strong>format</strong>ică,<br />

concursuri, recenzii etc. A coordonat personal organizarea şi buna desfăşurare<br />

a olimpiadelor şcolare de matematică. Continuând tradiţia vechii Gazete <strong>Matematice</strong>,<br />

a organizat Concursul anual al rezolvitorilor. S-a preocupat de organizarea unor<br />

consfătuiri cu caracter metodic, metodologic şi ştiinţific pentru învăţători şi profesorii<br />

de matematică.<br />

În 1975 a reorganizat Societatea de Ştiinţe <strong>Matematice</strong> prin înfiinţarea a 88 de<br />

filiale şi subfiliale cu un program riguros de perfecţionare ştiinţifică şi metodică. A<br />

coordonat apariţia, în Biblioteca S.S.M., a unui ciclu de culegeri de probleme şi cărţi<br />

cu caracter metodic.<br />

Nicolae Teodorescu a fost un militant neobosit şi consecvent pentru o educaţie<br />

completă a tineretului, preocupându-se personal de conţinutul educativ al activităţilor<br />

organizate cu tinerii. Taberele de matematică erau organizate în localităţi al căror<br />

nume aveau o rezonanţă istorică, întotdeauna în judeţul Argeş, la Câmpulung Muscel<br />

sau Curtea de Argeş. Activităţile începeau cu un istoric al instituţiei şi oraşului gazdă<br />

şi expuneri privind istoria matematicii din România. Elevii aflându-se în vacanţă,<br />

activităţile matematice alternau cu acţiuni cultural educative şi distractive, cu excursii<br />

la numeroase obiective istorice din zonă: biserica ctitorită de Negru Vodă la<br />

descălecarea în Ţara Românească, mausoleul de la Rucăr şi alte locuri legate de faptele<br />

de vitejie ale ostaşilor români în primul război mondial.<br />

La împlinirea unui secol de la naşterea lui Nicolae Teodorescu, personalitate<br />

complexă a matematicii româneşti, ne plecăm fruntea în faţa amintirii sale cu convingerea<br />

că este un model stimulator pentru tânăra generaţie de matematicieni.<br />

2<br />

Prof. dr. Petru MINUŢ


O problemă de colecţie<br />

Marian TETIVA 1<br />

Priviţi cu atenţie mulţimile {0, 2, 4, 8} şi {−1, 3, 5, 7}. Ce observaţi Probabil că,<br />

la prima vedere, nu mare lucru (se vede doar că sunt două mulţimi de numere întregi<br />

cu acelaşi număr de elemente). Dar ele au o proprietate specială: toate sumele de câte<br />

două elemente dintr-una din aceste mulţimi coincid cu sumele de câte două elemente<br />

din cealaltă. (E vorba de sumele 0 + 2, 0 + 4, 0 + 8, 2 + 4, 2 + 8, 4 + 8 care sunt<br />

egale, într-o anumită ordine, cu −1 + 3, −1 + 5, −1 + 7, 3 + 5, 3 + 7, 5 + 7.) Cu<br />

puţină răbdare puteţi constata că şi mulţimile, să zicem, {0, 2, 4, 8, 10, 14, 16, 18} şi<br />

{−1, 3, 5, 7, 11, 13, 15, 19} au aceeaşi proprietate (dar trebuie calculate mult mai multe<br />

sume). Se vede că mulţimile din primul exemplu au câte patru elemente, cele din cel<br />

de-al doilea - câte opt. Oare ce număr de elemente ar putea să aibă două mulţimi cu<br />

această proprietate Răspunsul la această întrebare a fost dat, el spune că nu veţi<br />

găsi asemenea mulţimi nici cu trei, nici cu zece, nici cu o sută cincisprezece elemente,<br />

ci doar cu un număr de elemente care este putere a lui doi. Mai precis, are loc<br />

Teorema 1 (Erdős-Selfridge).<br />

distincte de numere reale astfel încât<br />

Fie {a 1 , . . . , a n } şi {b 1 , . . . , b n } două colecţii<br />

{a i + a j : 1 ≤ i < j ≤ n} = {b i + b j : 1 ≤ i < j ≤ n}.<br />

Atunci n este o putere a lui 2. Mai mult, pentru orice n putere a lui 2, se pot construi<br />

exemple de asemenea colecţii (fiecare având n elemente).<br />

Să remarcăm că este vorba de mulţimi într-un sens mai larg decât cel obişnuit,<br />

anume, de ”mulţimi” în care elementele se pot repeta - le-am numit (ca în [3])<br />

colecţii (se foloseşte termenul multisets în limba engleză). Tot astfel se consideră<br />

şi colecţiile (iar nu mulţimile) sumelor de câte două elemente <strong>format</strong>e de {a 1 , . . . , a n }<br />

şi {b 1 , . . . , b n }. Astfel, se poate verifica uşor că, de pildă, colecţiile {0, 2, 2, 2} şi<br />

{1, 1, 1, 3} produc aceeaşi colecţie de sume, anume {2, 2, 2, 4, 4, 4}.<br />

Puteţi găsi în [3, problema 4.26] acest rezultat, demonstraţia lui clasică (foarte<br />

cunoscută; o reluăm şi noi imediat) şi referirea la una din primele apariţii ale sale în<br />

literatură. Schiţăm pe scurt această soluţie, căci tot de la ea vom porni şi noi. Ideea<br />

principală este de a considera două funcţii<br />

f(x) = x a 1<br />

+ · · · + x an<br />

şi g(x) = x b 1<br />

+ · · · + x b n<br />

şi de a observa că ipoteza teoremei se transcrie în egalitatea<br />

(f(x)) 2 − f(x 2 ) = (g(x)) 2 − g(x 2 ) ⇔ (f(x) − g(x))(f(x) + g(x)) = f(x 2 ) − g(x 2 ).<br />

1 Profesor, Colegiul Naţional ”Gheorghe Roşca Codreanu”, Bârlad<br />

3


Apoi se scrie f(x) − g(x) = (x − 1) k h(x), cu h(1) ≠ 0 (adică se pune în evidenţă<br />

cea mai mare putere a lui x − 1 care divide diferenţa f(x) − g(x)), deci obţinem<br />

f(x 2 ) − g(x 2 ) = (x − 1) k (x + 1) k h(x 2 ) şi apoi egalitatea decisivă<br />

h(x)(f(x) + g(x)) = (x + 1) k h(x 2 );<br />

într-adevăr, pentru x = 1, aceasta ne furnizează concluzia: 2n = 2 k ⇔ n = 2 k−1 .<br />

Exerciţiul 1. Demonstraţi partea a doua a teoremei, adică arătaţi cum, pentru<br />

n putere a lui 2, se pot construi două colecţii de câte n numere reale (fiecare) care<br />

produc colecţii egale de sume de câte două elemente.<br />

Această demonstraţie are neajunsul că se aplică (în exact aceşti termeni) doar dacă<br />

numerele a i şi b i sunt întregi, pentru ca f şi g să fie polinoame în adevăratul înţeles<br />

al cuvântului (chiar şi atunci trebuie observat că proprietatea numerelor se păstrează<br />

dacă li se adună tuturor acelaşi număr, şi facem această operaţie, dacă e nevoie, pentru<br />

a le avea pe toate pozitive). Totuşi, câteva argumente simple din analiza matematică<br />

vor face demonstraţia acceptabilă chiar dacă avem de-a face cu numere reale oarecare<br />

(aşa cum arătăm mai jos). Ceea ce e foarte bine, căci, indiscutabil, avem în faţă o<br />

problemă foarte frumoasă cu o rezolvare pe măsură, care (ne-am gîndit noi) merită să<br />

fie reamintită din când în când şi (de ce nu) să fie exploatată ceva mai mult. Asta<br />

intenţionăm să facem mai departe, anume să demonstrăm<br />

Teorema 2.<br />

reale astfel încât<br />

Fie {a 1 , . . . , a n } şi {b 1 , . . . , b n } două colecţii distincte de numere<br />

{a i + a j : 1 ≤ i < j ≤ n} = {b i + b j : 1 ≤ i < j ≤ n}.<br />

Atunci n este o putere a lui 2 şi, dacă n = 2 k−1 , avem<br />

a j 1 + · · · + aj n = b j 1 + · · · + bj n,<br />

pentru orice 0 ≤ j ≤ k − 1.<br />

Demonstraţie. Folosim aceleaşi funcţii f şi g ca mai sus, despre care mai facem<br />

observaţia că sunt indefinit derivabile pe (0, ∞). Deoarece f(1) = g(1)(= n) se poate<br />

considera cel mai mic număr întreg pozitiv k astfel încât f (j) (1) = g (j) (1) pentru orice<br />

j = 0, 1, . . . , k − 1 şi f (k) (1) ≠ g (k) (1) (indicele superior pus în paranteze arată, ca de<br />

obicei, ordinul de derivare).<br />

Exerciţiul 2. Fie u o funcţie de k ori derivabilă cu derivata de ordinul k continuă<br />

într-o vecinătate a unui punct x 0 astfel încât u (j) (x 0 ) = 0 pentru j = 0, 1, . . . , k − 1<br />

şi u (k) (x 0 ) ≠ 0. Arătaţi că<br />

u(x)<br />

lim<br />

n→∞ (x − x 0 ) k = u(k) (x 0 )<br />

≠ 0.<br />

k!<br />

(Aceasta este o aplicaţie simplă a regulii lui l ′ Hospital, pe care o găsiţi în orice<br />

manual de analiză - de exemplu [2]. Observaţi analogia cu funcţiile polinomiale: se<br />

poate scrie u(x) = (x − x 0 ) k v(x) şi, dacă vrem ca funcţia v să fie continuă şi în x 0 ,<br />

valoarea ei în acest punct rezultă nenulă).<br />

4


Acum să scriem egalitatea (f(x)) 2 − f(x 2 ) = (g(x)) 2 − g(x 2 ) în forma<br />

f(x) − g(x)<br />

(x − 1) k (f(x) + g(x)) = f(x2 ) − g(x 2 )<br />

(x 2 − 1) k (x + 1) k<br />

(desigur, pentru x ≠ 1) şi să facem aici pe x să tindă la 1. Obţinem (folosind exerciţiul<br />

2 pentru funcţiile x ↦→ f(x) − g(x), respectiv x ↦→ f(x 2 ) − g(x 2 ) şi, evident, x 0 = 1)<br />

f (k) (1) − g (k) (1)<br />

(f(1) + g(1)) = f (k) (1) − g (k) (1)<br />

2 k<br />

k!<br />

k!<br />

deci 2n = 2 k , la<br />

nX<br />

fel ca şi în prima variantă de<br />

nX<br />

demonstraţie (cea mai cunoscută),<br />

graţie faptului că f (k) (1) ≠ g (k) (1). Ceea ce nu se spune de obicei când se face acea<br />

demonstraţie este că avem egalităţile f (j) (1) = g (j) (1), adică<br />

a i (a i − 1) · · · (a i − j + 1) = b i (b i − 1) · · · (b i − j + 1)<br />

i=1<br />

pentru j = 0, 1, . . . , k − 1.<br />

Acum se poate încheia demonstraţia cu câteva calcule algebrice simple; mai e<br />

nevoie doar să justificăm afirmaţia din<br />

Exerciţiul 3. Egalităţile f (j) (1) = g (j) (1) pentru j = 0, 1, . . . , k−1 sunt echivalente<br />

cu a j 1 + · · · + aj n = b j 1 + · · · + bj n pentru j = 0, 1, . . . , k − 1.<br />

Cititorul atent trebuie să aibă o nemulţumire: de unde ştim că, la un moment dat<br />

(asta însemnând pentru un anume k natural) avem f (k) (1) ≠ g (k) (1) De ce n-ar fi<br />

valorile derivatelor de acelaşi ordin ale funcţiilor f şi g în 1 egale oricare ar fi acest<br />

ordin E clar că în această situaţie s-ar obţine a j 1 + · · · + aj n = b j 1 + · · · + bj n pentru<br />

orice număr natural n (la fel ca mai sus), iar răspunsul la întrebare e dat de<br />

Exerciţiul 4. Dacă pentru numerele reale a 1 , . . . , a n şi b 1 , . . . , b n avem<br />

a j 1 + · · · + aj n = b j 1 + · · · + bj n<br />

pentru j = 1, . . . , n, atunci colecţiile {a 1 , . . . , a n } şi {b 1 , . . . , b n } coincid.<br />

Ori, tocmai asta e ideea: dacă avem două colecţii distincte care produc aceeaşi<br />

colecţie de sume de câte două, atunci numărul elementelor din fiecare colecţie este<br />

o putere a lui 2 (nu am mai spus, dar e aproape evident: două colecţii cu cardinale<br />

diferite nu pot produce aceeaşi colecţie de sume; considerarea aceluiaşi n ca număr<br />

de a-uri şi de b-uri este obligatorie). Iată de ce nu putem obţine la nesfârşit egalităţi<br />

de forma a j 1 + · · · + aj n = b j 1 + · · · + bj n, deci la un moment dat trebuie să avem<br />

f (k) (1) ≠ g (k) (1).<br />

Şi acum chiar că am terminat. Ar mai fi poate de menţionat că exerciţiul 4 este o<br />

consecinţă firească şi aproape imediată a formulelor lui Newton, care pot fi găsite în<br />

orice manual de algebră (v. [1]).<br />

Bibliografie<br />

1. A. Kostrikin - Introduction à l ′ algèbre, Éditions Mir, Moscou, 1981.<br />

2. G. E. Şilov - Analiză matematică. Funcţii de o variabilă, Editura Ştiinţifică şi<br />

Enciclopedică, Bucureşti, 1985.<br />

3. Ioan Tomescu - Probleme de combinatorică şi teoria grafurilor, Editura Didactică<br />

şi Pedagogică, Bucureşti, 1981.<br />

5<br />

i=1


Drepte concurente în conexiune cu punctele I, Γ , N<br />

Temistocle Bîrsan 1<br />

1. Notaţii şi teoreme utilizate. Fie ABC un triunghi oarecare. Notăm cu<br />

D, E, F punctele de tangenţă a cercului înscris C(I, r) la dreptele BC, CA şi respectiv<br />

AB. Cu D a , E a , F a notăm punctele de tangenţă a cercului exînscris C(I a , r a ) cu<br />

aceleaşi drepte; notaţii similare relativ la cercurile exînscrise C(I b , r b ) şi C(I c , r c ).<br />

Mai notăm cu L a , L b , L c picioarele bisectoarelor interioare ale unghiurilor bA,ÒB şi<br />

respectivÒC.<br />

Se ştie că punctul lui Gergonne (notat Γ) este punctul de concurenţă a dreptelor<br />

AD, BE şi CF (numite ceviene Gergonne), iar punctul lui Nagel (notat N) este punctul<br />

de concurenţă a dreptelor AD a , BE b şi CF c (cevienele Nagel). În sfârşit, punctul<br />

Γ a -unul dintre cele trei puncte Gergonne adjuncte–este punctul de intersecţie a<br />

dreptelor AD a , BE a şi CF a .<br />

În prezenta notă vom constata că dreptele ce trec prin picioarele unor ceviene şi<br />

sunt paralele la altele (dintre cele de mai sus) în cele mai multe din cazurile posibile<br />

sunt concurente.<br />

În atingerea scopului, vom prelua neschimbat din [1] Propoziţia 2 şi varianta sa,<br />

Propoziţia 2 ′ :<br />

Teorema 1. Dacă poziţiile punctelor M, N, P, Q, R, S din fig. 1 sunt precizate de<br />

rapoartele m = <strong>MB</strong><br />

MA , n = NC<br />

NA , p = P B<br />

P A , q = QC<br />

QA , r = RB<br />

RA , s = SC<br />

SB , atunci<br />

dreptele MN, P Q, RS sunt concurente dacă şi numai dacă avem<br />

<br />

1 1 1<br />

prs + mq + nr = mrs + np + rq, i.e. m p r =0.<br />

n q rs<br />

Teorema 1 ′ . Dacă punctele M, N, P, Q, R, S din fig. 2 au poziţiile precizate de<br />

rapoartele m = <strong>MB</strong><br />

MA , n = NC<br />

NA , p = P B<br />

P A , q = QC<br />

QA , r = RB<br />

RC , s = SC<br />

SA , atunci<br />

dreptele MN, P Q, RS sunt concurente dacă şi numai dacă avem<br />

<br />

1 1 1<br />

qrs + ms + np = nrs + mq + ps, i.e. m p rs=0.<br />

n q s<br />

Următoarele două rezultate sunt consecinţe ale acestora.<br />

Teorema 2. Fie punctele M, N, P, Q, R, S ca în fig. 3, cu poziţiile date de rapoartele<br />

m = MC<br />

<strong>MB</strong> , n = NA<br />

NC , p = P C<br />

P B , q = QA<br />

QB , r = RC SA şi s =<br />

RB SB . Atunci<br />

dreptele MN, P Q, RS sunt concurente dacă şi numai dacă<br />

<br />

1 1 1<br />

mnp + ms + qr = mnr + mq + ps, i.e. m p r=0.<br />

mn q s<br />

1 Prof. dr., Catedra de matematică, Universitatea Tehnică ”Gh. Asachi”, Iaşi<br />

6


A<br />

A<br />

A<br />

A<br />

R<br />

M<br />

P<br />

Q<br />

N<br />

M<br />

P<br />

S<br />

Q<br />

N<br />

Q<br />

S<br />

N<br />

Q<br />

S<br />

N<br />

B<br />

S<br />

C<br />

B<br />

R<br />

C<br />

B<br />

M<br />

R<br />

P<br />

C<br />

B<br />

M<br />

R<br />

P<br />

C<br />

Fig. 1<br />

Fig. 2<br />

Fig. 3<br />

Fig. 4<br />

Teorema 2 ′ . Fie punctele M, N, P, Q situate ca în fig. 4, cu poziţiile date de<br />

m = <strong>MB</strong><br />

MC , n = NA<br />

NC , p = P B<br />

P C , q = QA<br />

QB , r = RB<br />

RC , s = SA<br />

SC . Atunci dreptele<br />

MN, P Q, RS sunt concurente dacă şi numai dacă<br />

<br />

1 1 1<br />

pqm + ps + nr = pqr + pn + ms, i.e. m p r=0.<br />

n pq s<br />

Aceste teoreme sunt forme mai generale ale teoremei lui Ceva şi reciprocei sale.<br />

Folosirea segmentelor orientate (marcate prin supraliniere) face posibil ca punctele în<br />

discuţie să poată fi situate oriunde pe dreptele suport ale laturilor triunghiului.<br />

2. Paralele la bisectoare. Mai întâi vom antrena bisectoarele interioare.<br />

Propoziţia 1. Paralelele la bisectoarele AL a , BL b , CL c prin punctele de tangenţă<br />

D, E şi respectiv F sunt concurente.<br />

Demonstraţie. Putem presupune, ca în fig. 5, că a < c < b (se procedează la fel<br />

în restul cazurilor). Vom aplica Teorema 2. Avem:<br />

A<br />

m = E′ C<br />

D<br />

E ′ B =− E′ C<br />

ab<br />

E ′ − (p − c)˜<br />

B a + c<br />

= a + c<br />

a − c · p − c<br />

p − b ,<br />

n = EA<br />

EC = −p − a<br />

p − c ,<br />

p = F ′ C<br />

F ′ B = −F L c<br />

F B = −•<br />

= a − b<br />

a + b · p − c<br />

p − b ,<br />

=−<br />

EC<br />

EL b<br />

=−(p − c)Á•<br />

ac<br />

a + b − (p − b)˜Á(p − b)<br />

q = F A<br />

F B = −p − a<br />

p − b , r = DC<br />

DB = −p − c<br />

p − b ,<br />

ac<br />

s = D′ A<br />

D ′ B = −DL a<br />

DB = −•<br />

B<br />

L c<br />

F<br />

E<br />

.<br />

I<br />

.<br />

D<br />

Fig. 5<br />

b + c − (p − b)˜Á(p − b) = − b − c<br />

b + c · p − a<br />

p − b .<br />

Condiţia de concurenţă din Teorema 2 sub formă de determinant revine la<br />

7<br />

L a<br />

L b<br />

E<br />

F<br />

C


1 1 1<br />

a − b<br />

a + b · p − c − p − c<br />

p − b p − b<br />

a + c<br />

a − c · p − c<br />

p − b<br />

− a + c<br />

a − c · p − a<br />

p − b<br />

− p − a<br />

p − b<br />

− b − c<br />

b + c · p − a<br />

p − b<br />

(a − c)(p − b) (a + b)(p − b) (b + c)(p − b)<br />

(a + c)(p − c) (a − b)(p − c) −(b + c)(p − c)<br />

−(a + c)(p − a) −(a + b)(p − a) −(b − c)(p − a)<br />

=0 ⇔<br />

=0,<br />

ceea ce este adevărat, întrucât prima coloană este diferenţa celorlalte două. Aşadar,<br />

dreptele DD ′ , EE ′ şi F F ′ sunt concurente.<br />

Observaţie. Cititorul poate evita calculul cu determinanţi utilizând condiţia<br />

explicitată din Teorema 2.<br />

Propoziţia 2. Paralelele prin D a , E b , F c la bisectoarele interioare AL a , BL b şi<br />

respectiv CL c sunt concurente.<br />

Demonstraţie. Ne limităm, din nou, la cazul a < c < b. Vom aplica Teorema 1 ′<br />

relativ la dreptele D a D ′ , E b E ′ şi F c F ′ . Avem:<br />

m = F cB<br />

F c A = −p − a<br />

p − b ,<br />

n = F ′ C<br />

F ′ A = −F cL c<br />

F c A = −•<br />

= a − b<br />

a + b · p<br />

p − b ,<br />

p = E′ B<br />

E ′ A = −E bL b<br />

E b A = −•<br />

= a − c<br />

a + c · p<br />

p − c ,<br />

q = E bC<br />

E b A = −p − a<br />

p − c ,<br />

s = D′ C<br />

D ′ A = − D aC<br />

D a L a<br />

= −(p − b)Á•<br />

bc<br />

a + b − (p − b)˜Á(p − b)<br />

bc<br />

a + c − (p − c)˜Á(p − c)<br />

ab<br />

b + c<br />

r = D aB<br />

D a C = −p − c<br />

p − b ,<br />

În acest caz, condiţia din Teorema 1 ′ se scrie<br />

<br />

<br />

a − b<br />

a + b ·<br />

F c<br />

A<br />

D<br />

E<br />

L<br />

.<br />

b<br />

. c<br />

L<br />

F b<br />

I E<br />

F<br />

E<br />

.<br />

B D D a<br />

. .<br />

L a<br />

Fig. 6<br />

− (p − b)˜=<br />

c + b<br />

c − b · p − b<br />

p .<br />

1 1 1<br />

− p − a a − c<br />

p − b a + c · p<br />

− c + b<br />

p − c c − b · p − c<br />

p<br />

p<br />

− p − c c + b<br />

p − b p − b c − b · p − b<br />

p<br />

(a + b)(p − b) (a + c)(p − c) (c − b)p<br />

−(a + b)(p − a) (a − c)p −(b + c)(p − c)<br />

(a − b)p −(a + c)(p − a) (b + c)(p − b)<br />

8<br />

=0 ⇔<br />

=0,<br />

C


egalitate adevărată, prima coloană fiind suma celorlalte două (calcule de rutină!).<br />

Deci, dreptele D a D ′ , E b E ′ , F c F ′ sunt concurente.<br />

Relativ la punctul Gergonne adjunct Γ a are loc rezultatul următor:<br />

Propoziţia 3. Paralelele prin punctele D a , E a , F a (de tangenţă a cercului C(I a , r a )<br />

la bisectoarea interioară AI, bisectoarea exterioară BI a şi, respectiv, bisectoarea exterioară<br />

CI a sunt concurente.<br />

Demonstraţie. Vom aplica Teorema 1 ′ . Mai întâi, observăm că m(×E a E ′ F a ) =<br />

m(ÖI a BF a ) = 90 ◦ − B 2 . Ca urmare, m(×AE ′ E a ) =<br />

90 ◦ + B 2 şi m(×AE a E ′ ) = C − A . În △AE ′ E a<br />

2<br />

avem: AE ′ = sin C − A<br />

2<br />

sin C − A<br />

2<br />

·<br />

p<br />

sin(90 ◦ + B 2 ) =<br />

(c − a)p<br />

. Rezultă că BE ′ =<br />

b<br />

p ·<br />

cos B =<br />

2<br />

c − AE ′ (a + c)(p − c)<br />

= (după calcule!). Analog<br />

b<br />

găsim: AF ′ (b − a)p<br />

= şi CF ′ (a + b)(p − b)<br />

= .<br />

c<br />

c<br />

Cu aceste pregătiri, obţinem:<br />

m = E′ B<br />

E ′ A =<br />

F a<br />

B<br />

.<br />

E<br />

A<br />

D<br />

F<br />

I<br />

. .<br />

.<br />

D a<br />

I a<br />

Fig. 7<br />

− E′ B<br />

E ′ A = −c + a<br />

c − a · p − c<br />

p<br />

, n = E aC<br />

E a A = p − b<br />

p<br />

, p = F aB<br />

F a A = p − c<br />

p<br />

, q = F ′ C<br />

F ′ A =<br />

+ a + b<br />

a − b · p − b<br />

p<br />

, r = D aB<br />

D a C = −p − c<br />

p − b , s = D′ C<br />

D ′ A = − D aC<br />

= − b + c<br />

D a L a b − c · p − b<br />

p<br />

. Condiţia<br />

de concurenţă din Teorema 1 ′ se verifică imediat. În concluzie, dreptele D aD ′ , E a E ′<br />

şi F a F ′ sunt concurente.<br />

Dacă rolul bisectoarelor ar fi luat de cevienele Gergonne sau de cele Nagel, nu<br />

obţinem concurenţa paralelelor la acestea decât pentru triunghiuri particulare. Cititorul<br />

poate stabili, utilizând teoremele din secţiunea 1, următoarele rezultate:<br />

Propoziţia 4. Paralelele duse prin picioarele L a , L b , L c ale bisectoarelor interioare<br />

la cevienele Gergonne AD, BE şi respectiv CF sunt concurente dacă şi numai<br />

dacă triunghiul este isoscel.<br />

Propoziţia 5. Paralelele prin punctele L a , L b , L c la cevienele Nagel AD a , BE b şi<br />

respectiv CF c sunt concurente dacă şi numai dacă triunghiul este isoscel.<br />

3. Paralele prin puncte izotomice. Două puncte situate pe dreapta suport a<br />

unui segment [AB] se numesc izotomice dacă sunt simetrice faţă de mijlocul segmentului.<br />

Notăm cu A ′ , B ′ , C ′ mijloacele laturilor triunghiului (A ′ ∈ BC etc.).<br />

Teoremă. Fie AA 1 , BB 1 , CC 1 trei ceviene concurente în punctul X şi A 2 , B 2 , C 2<br />

izotomicele punctelor A 1 , B 1 şi respectiv C 1 .<br />

9<br />

C<br />

.<br />

E a


1) Paralelele prin punctele A 2 , B 2 , C 2 la cevienele AA 1 , BB 1 şi respectiv CC 1 sunt<br />

concurente într-un punct Y .<br />

2) Paralelele prin mijloacele A ′ , B ′ , C ′ la cevienele AA 1 , BB 1 şi respectiv CC 1 sunt<br />

concurente în mijlocul Z al segmentului [XY ].<br />

Demonstraţie. Notăm α = A 1B<br />

A 1 C , β = B 1C<br />

B A , γ = C 1A<br />

; prin ipoteză, αβγ = −1.<br />

C 1 B<br />

Fie a < c < b, ca în fig. 8. Aplicăm Teorema 1 dreptelor A<br />

A 2 A ′ 2, B 2 B 2, ′ C 2 C 2. ′ Cu uşurinţă găsim:<br />

A 2<br />

.<br />

m = C 2B<br />

B<br />

C C 2 2 A = AC 1<br />

= γ,<br />

BC 1 2<br />

Y<br />

B<br />

n = C′ 2 C<br />

2<br />

C C 1<br />

.<br />

B<br />

2 ′ A = C 2C 1<br />

C 2 A = C 2B<br />

+ 1 = AC 1<br />

+ 1 = γ + 1,<br />

BC 1 BC 1 1<br />

X C 2<br />

p = B′ 2 B<br />

B 2 ′ A = B 2B 1<br />

B 2 A = AB 1<br />

+ 1 = 1 CB 1 β + 1,<br />

q = B 2C<br />

B 2 A = AB 1<br />

CB 1<br />

= 1 β ,<br />

r = A′ 2 B<br />

A ′ 2 A = A 2B CA 1<br />

=<br />

A 2 A 1 A 2 C − A 1 C = CA 1<br />

= 1<br />

BA 1 + CA 1 α + 1 ,<br />

s = A 2C<br />

A 2 B = BA 1<br />

CA 1<br />

= α.<br />

B<br />

A 2<br />

A 1<br />

Fig. 8<br />

Excludem cazul degenerat β = 0, iar cazul particular α + 1 = 0 (adică ceviana AA 1<br />

este mediană) se tratează separat la fel. Cum<br />

<br />

1 1 1<br />

1 1 1<br />

<br />

1<br />

γ<br />

m p r=<br />

β + 1 1<br />

1 β α + 1<br />

1<br />

α + 1=<br />

γ β + 1 1 =0<br />

mn q s<br />

1 α β(α + 1)<br />

γ + 1<br />

γ + 1 1 α<br />

β α + 1<br />

(ultima egalitate obţinându-se dezvoltând determinantul), rezultă că afirmaţia punctului<br />

1) este dovedită.<br />

Pentru 2) am putea proceda la fel. Mai simplu, observăm că A ′ , B ′ , C ′ sunt<br />

mijloacele segmentelor [A 1 A 2 ], [B 1 B 2 ] şi [C 1 C 2 ]. Atunci, în fiecare dintre trapezele<br />

AA 1 A 2 A ′ , BB 1 B 2 B ′ , CC 1 C 2 C ′ paralelele la baze prin A ′ , B ′ şi respectiv C ′ vor trece<br />

prin mijlocul segmentului [XY ].<br />

Cititorul poate particulariza acest rezultat considerând diferite triplete de ceviene<br />

remarcabile în triunghi; altfel spus, considerând în locul lui X puncte ca H, O, I etc.<br />

Se ştie că (D, D a ) şi (D b , D c ) sunt perechi de puncte izotomice pe [BC]; analog şi<br />

pe celelalte două laturi (v. [3], p.31). Se obţin direct următoarele:<br />

Propoziţia 6. Dreptele ce trec prin punctele de tangenţă D, E, F (sau prin<br />

mijloacele laturilor A ′ , B ′ , C ′ ) şi sunt paralele cu cevienele Nagel corespunzătoare<br />

AD a , BE b , CF c sunt concurente. Punctul Nagel N şi cele două puncte de concurenţă<br />

rezultate sunt coliniare.<br />

10<br />

C


Propoziţia 7. Paralelele prin punctele D a , E b , F c (sau prin mijloacele A ′ , B ′ , C ′ )<br />

la cevienele Gergonne corespunzătoare sunt concurente. Punctul Gergonne Γ şi punctele<br />

de concurenţă rezultate sunt coliniare.<br />

4. Comentariu. Cu un efort suplimentar, am putea identifica unele puncte de<br />

concurenţă mai sus obţinute şi vedea că ele sunt puncte remarcabile în triunghi. Calea<br />

de urmat poate fi următoarea: se calculează coordonatele triliniare/baricentrice ale<br />

punctelor de concurenţă (ceea ce nu-i greu!) şi apoi se găseşte în lista ,,centrelor” din<br />

[2] cine sunt aceste puncte. De exemplu, punctul de concurenţă dat de Propoziţia 1<br />

este notat X 65 în [2], p.76, şi dintre proprietăţile indicate în acest loc enumerăm: se<br />

află pe dreptele OI şi Γ N, este izogonalul conjugat al punctului lui Schiffler etc.<br />

Bibliografie<br />

1. T. Bîrsan - Generalizări ale teoremei lui Ceva şi aplicaţii, Recreaţii <strong>Matematice</strong>,<br />

4(2002), nr. 2, 10-14.<br />

2. C. Kimberling - Triangle Centers and Central Triangle, Congressus Numerantium<br />

129, Winnipeg, Canada, 1998.<br />

3. T. Lalescu - Geometria triunghiului, Editura Tineretului, Bucureşti, 1958.<br />

MIORIŢA MATEMATICĂ<br />

Pe-un picior de<br />

PLAN EUCLIDIAN,<br />

Iată vin în cale,<br />

TRANSLATÂND la vale,<br />

Trei MULŢIMI de<br />

PUNCTE,<br />

Toate trei DISJUNCTE,<br />

De FUNCŢII păzite<br />

Toate diferite.<br />

Ele sunt tot trei:<br />

Una-i INJECTIVĂ,<br />

Alta-i BIJECTIVĂ<br />

Şi-alta-i SURJECTIVĂ.<br />

Iar cea INJECTIVĂ<br />

Şi cea SURJECTIVĂ,<br />

Mări, se vorbiră<br />

Şi se sfatuiră<br />

Să rămână treze<br />

Pân-o să-nsereze<br />

Şi s-o ANULEZE<br />

Pe cea BIJECTIVĂ,<br />

C-are PRIMITIVĂ<br />

Şi- ASIMPTOTE multe<br />

Câte şi mai câte,<br />

Că e INVERSABILĂ<br />

Şi chiar DERIVABILĂ ...<br />

Dar într-o MULŢIME<br />

Asta s-a aflat<br />

Şi s-au indignat,<br />

C-ale lor cuvinte<br />

Întrec orice LIMITE.<br />

Dar de la f(0)-ncoace<br />

Unui PUNCT nu-i place<br />

Să mai stea-n MULŢIME<br />

Şi de treabă-a se ţine.<br />

BIJECTIVA se-ntrebă:<br />

”PUNCTUL ăsta ce-o<br />

avea”<br />

Şi se duse<br />

Şi îi spuse:<br />

- Dragă PUNCTULEŢUL<br />

meu,<br />

Ce rău oare îţi fac eu<br />

Sau nu-ţi place poate<br />

C-ai COORDONATE<br />

NATURALE toate<br />

Vrei să stai mai jos,<br />

Crezi că-i mai frumos<br />

Nu vrei un‘te-am pus,<br />

Vrei cumva mai sus<br />

- Dragă BIJECTIVĂ,<br />

Eu chiar dimpotrivă,<br />

Mă simt foarte bine,<br />

Dar e rău de tine!<br />

(continuare la pagina 20)<br />

11


Asupra inegalităţii lui Jensen<br />

Florin POPOVICI 1<br />

În această notă stabilim inegalitatea lui Jensen pentru funcţii J-convexe; în raport<br />

cu demonstraţia lui Cauchy, bazată pe schema n → 2 n → n+1, raţionamentul nostru<br />

inductiv este unul direct şi are un suport geometric natural. Ideea din demonstraţia<br />

noastră, prin adaptare, produce demonstraţii simple ale unor inegalităţi clasice; prezentăm<br />

o demonstraţie realmente simplă a inegalităţii mediilor şi una pentru inegalitatea<br />

lui Huygens.<br />

1. Inegalitatea lui Jensen<br />

Teoremă. Fie I ⊆ R un interval. Dacă f : I → R este o funcţie J-convexă<br />

(convexă în sens Jensen), adică<br />

1 + x 2<br />

(1) fx<br />

≤ f(x 1) + f(x 2 )<br />

, (∀)x 1 , x 2 ∈ I,<br />

2 2<br />

atunci pentru orice n ∈ N, n ≥ 2 are loc inegalitatea lui Jensen<br />

1 + . . . + x n<br />

(2) fx<br />

≤ f(x 1) + . . . + f(x n )<br />

, (∀)x 1 , . . . , x n ∈ I.<br />

n n<br />

Demonstraţie. Stabilim (2) prin inducţie. Conform ipotezei, (2) are loc pentru<br />

n = 2.<br />

Fie n ∈ N, n ≥ 2, pentru care are loc (2). Arătăm că (2) are loc şi pentru valoarea<br />

n + 1. Fie a, b ∈ I. Considerăm punctele c =<br />

n<br />

n + 1 a + 1<br />

n + 1 b şi d = 1<br />

n + 1 a + n<br />

n + 1 b.<br />

Deoarece c = n − 1<br />

n a + 1 n d şi d = n − 1<br />

n<br />

b + 1 c, conform ipotezei inductive rezultă că<br />

n<br />

avem<br />

(3) f(c) ≤ n − 1<br />

n f(a) + 1 n − 1<br />

f(d), f(d) ≤<br />

n n<br />

f(b) + 1 n f(c).<br />

Urmează că<br />

(4) f<br />

n<br />

n + 1 a + 1<br />

n + 1 b‹=f(c) ≤<br />

n<br />

n + 1 f(a) + 1<br />

n + 1 f(b).<br />

Fie x 1 , . . . , x n+1 ∈ I. Conform ipotezei inductive şi ţinând cont de (3), obţinem<br />

x 1 + . . . + x n+1 n x 1 + . . . + x n<br />

f<br />

‹=f<br />

+ 1<br />

n + 1<br />

n + 1 n n + 1 x n+1‹≤<br />

n fx 1 + . . . x n<br />

+<br />

n + 1 n<br />

+ 1<br />

n + 1 f(x n+1) ≤<br />

n f(x 1 ) + . . . + f(x n )<br />

+ 1<br />

n + 1 n<br />

n + 1 f(x n+1) = f(x 1) + . . . f(x n+1 )<br />

,<br />

n + 1<br />

deci (2) are loc pentru valoarea n + 1. Conform principiului inducţiei matematice, (2)<br />

are loc pentru orice n ∈ N, n ≥ 2.<br />

Observaţia 1. Geometric, din (3) rezultă că nu poate avea loc contrara inegalităţii<br />

(4), deoarece s-ar contrazice proprietăţile de separare ale planului R 2 .<br />

1 Prof. dr., Colegiul Naţional ”Gr. Moisil”, Braşov<br />

12


2. Inegalitatea mediilor<br />

Teorema 2. Pentru orice n ∈ N, n ≥ 2 are loc inegalitatea mediilor<br />

n<br />

(5) √ x 1 · . . . · x n ≤ x 1 + . . . + x n<br />

, (∀)x 1 , . . . , x n ∈ [0, ∞).<br />

n<br />

Demonstraţie. Fie x 1 , x 2 ∈[0, ∞). Deoarece √ x 1 x 2 ≤ x 1 + x 2<br />

⇔ ( √ x 1 − √ x 2 ) 2 ≥<br />

2<br />

0, rezultă că (5) are loc pentru n = 2.<br />

Fie n ∈ N, pentru care are loc (1). Fie a, b ∈ [0, ∞). Considerăm punctele<br />

c = n+1√ a n b şi d = n+1√ ab n . Deoarece c = n√ a n−1 d şi d = n√ b n−1 c, conform<br />

(n − 1)a + d (n − 1)b + c<br />

ipotezei inductive avem c ≤ , d ≤ . De aici, obţinem succesiv<br />

n<br />

n<br />

n 2 c ≤ n(n − 1)a + nd ≤ n(n − 1)a + (n − 1)b + c, (n 2 − 1)c ≤ n(n − 1)a + (n − 1)b,<br />

(6)<br />

că<br />

n+1 √ a n b ≤ na + b<br />

n + 1 .<br />

Fie x 1 , . . . , x n+1 ∈ [0, ∞). Conform ipotezei inductive şi ţinând cont de (6), rezultă<br />

√ n+1<br />

x 1 · . . . · x n+1 = n+1È( n√ x 1 · . . . · x n ) n x n+1 ≤ n n√ x 1 · . . . · x n + x n+1<br />

n + 1<br />

≤<br />

n x 1+...+x n<br />

n<br />

+ x n+1<br />

= x 1 + . . . x n+1<br />

;<br />

n + 1<br />

n + 1<br />

deci (5) are loc şi pentru valoarea n + 1.<br />

Observaţia 2. În [1] sunt selectate şi prezentate cronologic circa 40 de demonstraţii<br />

ale inegalităţii mediilor, începând cu prima demonstraţie cunoscută, dată de către C.<br />

MacLaurin în 1729.<br />

3. Inegalitatea lui Huygens<br />

Teorema 3. Pentru orice n ∈ N, n ≥ 2 are loc inegalitatea lui Huygens:<br />

(7) 1 + n√ x 1 · . . . · x n ≤ nÈ(1 + x 1 ) · . . . · (1 + x n ), (∀)x 1 , . . . , x n ∈ [0, ∞).<br />

Demonstraţie. Fie x 1 , x 2 ∈ [0, ∞). Deoarece<br />

(1 + √ x 1 x 2 ) 2 = 1 + 2 √ x 1 x 2 + x 1 x 2 ≤ 1 + x 1 + x 2 + x 1 x 2 =È(1 + x 1 )(1 + x 2 ) 2 ,<br />

rezultă că (7) are loc pentru n = 2.<br />

Fie n ∈ N, n ≥ 2 pentru care are loc (7). Fie a, b ∈ [0, ∞). Considerăm punctele<br />

c = n+1√ a n b şi d = n+1√ ab n . Avem c = n√ a n−1 d şi d = n√ b n−1 c. Conform ipotezei<br />

inductive, rezultă că 1 + c ≤ nÈ(1 + a) n−1 (1 + d), 1 + d ≤ nÈ(1 + b) n−1 (1 + c).<br />

De aici, obţinem succesiv<br />

(1 + c) n2 ≤ (1 + a) n(n−1) (1 + d) n ≤ (1 + a) n(n−1) (1 + b) n−1 (1 + c),<br />

13


(1 + c) n2 −1 ≤ (1 + a) n(n−1) (1 + b) n−1 ,<br />

(8) 1 + n+1√ a n b ≤ n+1È(1 + a) n (1 + b).<br />

Fie x 1 , . . . , x n+1 ∈ [0, ∞). Conform ipotezei inductive şi ţinând cont de (8), rezultă<br />

că 1 + n+1√ x 1 · . . . · x n+1 = 1 + n+1È( n√ x 1 · . . . · x n ) n x n+1<br />

≤ n+1È(1 + n√ x 1 · . . . · x n ) n (1 + x n+1 )<br />

≤ n+1qnÈ(1 + x 1 ) · . . . · (1 + x n ) n (1 + x n+1 ) = n+1È(1 + x 1 ) · . . . · (1 + x n+1 );<br />

deci (7) are loc şi pentru valoarea n + 1.<br />

4. Observaţii finale<br />

Posibilitatea conectării inegalităţii lui Jensen cu alte inegalităţi a fost evidenţiată<br />

(cf. [2], pag. 4) de către G. Aumann în anul 1933, prin introducerea conceptului de<br />

funcţie (M, N)-J-convexă.<br />

Fie I 1 , I 2 ⊆ R două intervale. Fie M o medie pe I 1 şi fie N o medie pe I 2 . O<br />

funcţie f : I 1 → I 2 se numeşte (M, N)-J-convexă dacă f(M(x, y)) ≤ N(f(x), f(y)),<br />

(∀)x, y ∈ I 1 . În anumite condiţii asupra mediilor M şi N se obţine inegalitatea lui<br />

Jensen generalizată:<br />

f(M(x 1 , . . . , x n )) ≤ N(f(x 1 ), . . . , f(x n )), (∀)x 1 , . . . , x n ∈ I 1 .<br />

Dacă notăm cu A media aritmetică şi cu G media geometrică, atunci:<br />

1) inegalitatea lui Jensen clasică este inegalitatea lui Jensen generalizată pentru<br />

funcţii (A, A) − J-convexe;<br />

2) inegalitatea mediilor este inegalitatea lui Jensen generalizată pentru funcţia<br />

(G, A) − J-convexă 1 [0,∞) ;<br />

3) inegalitatea lui Huygens este inegalitatea lui Jensen generalizată pentru funcţia<br />

(G, G) − J-convexă x → 1 + x, (∀)x ∈ [0, ∞).<br />

In [3], inegalitatea lui Jensen generalizată este stabilită pentru funcţii (M, N)−Jconvexe,<br />

corespunzător unei clase largi de medii, care include mediile cvasi-aritmetice.<br />

Bibliografie<br />

1. P.S. Bullen, D.S. Mitrinović, P.M. Vasić - Means and Their Inequalities,<br />

D. Reidel Publishing Company, Dordrecht, Holland, 1988.<br />

2. C.P. Niculescu, L.E. Persson - Convex Functions and Their Applications,<br />

A Contemporary Approach. CMS Books in Mathematics, vol. 23, Springer-Verlag,<br />

New York, 2006.<br />

3. C.P. Niculescu, F. Popovici - Inegalitatea lui Jensen pentru funcţii (M, N) − J-<br />

convexe în condiţii generale (va apare).<br />

Vizitaţi noua pagina web a revistei:<br />

http://www.recreatiimatematice.ro<br />

14


O rafinare a inegalităţii lui Euler R ≥ r √ 2<br />

Mihály BENCZE 1<br />

Pentru un patrulater ABCD înscris într-un cerc C(O, R) şi circumscris unui cerc<br />

C(I, r) este valabilă relaţia lui Durrande ([2], p.216), din care decurge inegalitatea de<br />

tip Euler R ≥ r √ 2 (cu egalitate dacă şi numai dacă O şi I coincid, ceea ce revine la<br />

faptul că patrulaterul este pătrat).<br />

Notând cu S şi p aria şi semiperimetrul unui astfel de patrulater, avem sin A =<br />

2S<br />

ad + bc , sin B = 2S (prima formulă rezultă din ad sin A+bc sin C = 2S şi faptul<br />

ab + cd<br />

că A + C = B + D = π). De asemenea au loc relaţiile: S = 1 2<br />

(a + b + c + d)r = pr,<br />

a + c = b + d = p, S 2 = abcd etc. (v. [2]).<br />

Rezultatul următor indică un şir de rafinări ale inegalităţii R ≥ r √ 2.<br />

Teoremă. Fie ABCD un patrulater înscris într-un cerc de rază R şi circumscris<br />

Œ<br />

unui cerc de rază r. Atunci, avem<br />

r √ 2<br />

Ž≤ 1 R<br />

2‚cos A − B + r√ 2<br />

2 R<br />

Ž<br />

+Êr 2 + r √ 4R 2 + r 2<br />

2R 2<br />

≤ 1 2„Êr 2 + r √ 4R 2 + r 2<br />

2R 2 + r√ 2<br />

R<br />

≤<br />

2„cos 1 A − B<br />

‹<br />

2<br />

≤ 1 4 cos A − B + cos B − C + cos C − D + cos D − A<br />

2<br />

2<br />

2<br />

2<br />

2 + sin A + sin B<br />

≤<br />

≤ 1.<br />

4<br />

Demonstraţie. Ţinând seama de formulele mai sus amintite, obţinem<br />

√<br />

1 + sin A sin B<br />

sin A sin B<br />

= p(È(ab + cd)(ad + bc)(ac + bd))<br />

4S 2<br />

Ca urmare, R 2 u 2 − r 2 u − r 2 = 0, unde u = sin A sin B, şi vom avea<br />

= p · 4RS<br />

4S 2 = R r .<br />

sin A sin B = r2 + r √ 4R 2 + r 2<br />

2R 2 .<br />

Revenind la scopul propus, să notăm E = 1 4Pcos A − B<br />

2‹ 2‹<br />

şi să observăm că<br />

2<br />

E = 1 <br />

2 cos A − B + cos B − C ‹= 1 2<br />

2 2 sin A 2 + cos A sin B 2 + cos B<br />

≤ 1 sin A 4 2 + cos A + sin<br />

2‹2<br />

B 2 + cos B 2 + sin A + sin B<br />

≤ 1.<br />

2‹2=<br />

4<br />

1 Profesor, Braşov, e-mail: benczemihaly@yahoo.com<br />

15


Pe de altă parte,<br />

E = 1 2 sin A 2 + cos A 2‹(sin B 2 + cos B 2 ) ≥ 1 2‚rsin A 2 sin B 2 +rcos A Ž<br />

2 cos B 2Œ2<br />

= 1 2 cos A − B<br />

+Êr 2 + r √ 4R 2 + r 2<br />

2<br />

2R 2<br />

Œ<br />

+ r√ 2Œ= 1 R 2‚cos A 2 cos B 2 + sin A 2 sin B 2 + r√ 2<br />

R<br />

+ √ sin A sin B‹= 1 2„cos A − B<br />

2<br />

≥ 1 2‚cos A − B<br />

Œ<br />

2<br />

≥<br />

2‚2rsin 1 A 2 cos A 2 sin B 2 cos B 2 + r√ 2<br />

R<br />

= 1 2„Êr 2 + r √ 4R 2 + r 2<br />

2R 2 + r√ 2Ž≥ r√ 2<br />

R R<br />

(s-a utilizat faptul că din R ≥ r √ 2 decurge cărr 2 + r √ 4R 2 + r 2<br />

2R 2 ≥ r√ 2<br />

R ).<br />

Bibliografie<br />

1. M. Bencze - Inequalities (manuscript), 1982.<br />

2. D. Mihalca, I. Chiţescu, M. Chiriţă - Geometria patrulaterului. Teoreme şi<br />

probleme, Teora, Bucureşti, 1998.<br />

3. D.S. Mitrinović - Analytic Inequalities, Springer-Verlag, 1970.<br />

4. Octogon Mathematical Magazine (1993-2008).<br />

ERRATUM<br />

În articolul Acurateţea limbajului matematic în combinatorică de L. Modan,<br />

apărut în nr. 1 din v. X (2008), în rândul 19 de la pag. 43, în loc de ”se formează în<br />

4n 3 − n + 1 moduri”, se va citi ”se formează în<br />

moduri”.<br />

X<br />

n−1<br />

k=1<br />

C k 2n−1 · C k+1<br />

2n+1 = (4n)!<br />

2 · [(2n)!] 2 − 2n − 1<br />

16


Cercuri tangente la două cercuri date<br />

Geanina HĂVÂRNEANU 1<br />

Fie C 1 şi C 2 două cercuri date într-un plan. Ne propunem să rezolvăm următoarea<br />

problemă:<br />

Să se determine locul geometric al centrelor cercurilor tangente la cercurile C 1 şi<br />

C 2 .<br />

Un prim gând în scopul propus este acela de a folosi metoda coordonatelor. Vom<br />

vedea, însă, că este preferabil să abordăm problema cu mijloacele geometriei sintetice;<br />

vor fi evitate astfel calculele neplăcute (destul de simple, ce-i drept!).<br />

Fie O 1 şi respectiv O 2 centrele cercurilor C 1 , C 2 şi u distanţa centrelor lor. Fără<br />

a restrânge generalitatea, să presupunem că raza cercului C 1 este 1, iar a lui C 2 este<br />

v ≤ 1. Fie C(O, r) un cerc tangent la C 1 , C 2 şi variabil. Este evident faptul că locul<br />

geometric căutat, locul punctului O, depinde de poziţia relativă atât a cercurilor fixe<br />

C 1 , C 2 cât şi a cercului variabil C faţă de cele fixe.<br />

I. C 1 şi C 2 sunt exterioare (u > 1 + v). Distingem<br />

patru familii de cercuri C tangente la C 1 şi C 2 : 1) C este<br />

tangent exterior cercurilor C 1 , C 2 ; 2) C 1 , C 2 sunt tangente<br />

interior cercului C; 3) C este tangent exterior la C 1 şi C 2<br />

este tangent interior la C; 4) C este tangent exterior la C 2<br />

şi C 1 este tangent interior la C.<br />

În cazul 1), avem OO 1 − OO 2 = (1 + r) − (v + r) =<br />

= 1−v =¨const., v < 1<br />

0, v = 1.<br />

O<br />

O<br />

O 1<br />

O 2<br />

În cazul 2), avem O ′ O 2 − O ′ v < 1<br />

O 1 = (r − v) − (r − 1) = 1 − v =¨const.,<br />

0, v = 1.<br />

Pentru v < 1, în ambele cazuri, avem |OO 1 − OO 2 | = 1 − v > 0, adică punctul<br />

O parcurge o hiperbolă H 1 cu focarele O 1 şi O 2 , cu centrul în mijlocul segmentului<br />

[O 1 O 2 ] şi vârfurile V 1 , V 1 ′ ∈ (O 1 O 2 ) precizate de O 1 V 1 = u 2 + 1 − v (după cum rezultă<br />

2<br />

din relaţiile V 1 O 1 − V 1 O 2 = 1 − v şi O 1 O 2 = u) şi O 1 V 1 ′ = u 2 − 1 − v<br />

2 . Ramura<br />

”dreaptă” a hiperbolei H 1 este locul centrelor cercurilor C aflate în cazul 1), căci în<br />

acest caz OO 1 > OO 2 , iar ramura ”stângă” este locul centrelor aflate în cazul 2), căci<br />

în acest caz avem O ′ O 1 < O ′ O 2 .<br />

Pentru v = 1 (adică cercurile C 1 şi C 2 sunt egale), în ambele cazuri se obţine OO 1 =<br />

OO 2 şi locul geometric căutat este mediatoarea segmentului [O 1 O 2 ]. Hiperbola H 1<br />

degenerează în această mediatoare.<br />

Procedăm la fel în cazurile 3) şi 4) (cititorul va face singur figura). În cazul 3)<br />

avem OO 1 − OO 2 = (1 + r) − (r − v) = 1 + v, iar în cazul 4) avem OO 2 − OO 1 =<br />

(r + v) − (r − 1) = 1 + v. Aşadar, în aceste cazuri |OO 1 − OO 2 | = 1 + v, adică locul<br />

1 Profesor, Şcoala generală Horleşti, Horleşti (Iaşi)<br />

17


geometric este o hiperbolă H 2 cu focarele tot punctele O 1 , O 2 şi vârfurile V 2 , V 2 ′ date<br />

de O 1 V 2 = u 2 + 1 + v şi O 1 V 2 ′ = u 2<br />

2 − 1 + v<br />

2 . Ramura ”dreaptă” a hiperbolei H 2 este<br />

parcursă în cazul 3), iar cea ”stângă” în cazul 4). Să mai observăm că situaţa v = 1<br />

nu necesită o tratare distinctă.<br />

În concluzie, locul geometric al centrelor cercurilor tangente la două cercuri C 1 , C 2<br />

exterioare este <strong>format</strong> din hiperbolele H 1 şi H 2 , dacă C 1 , C 2 au raze diferite, sau din<br />

mediatoarea segmentului [O 1 O 2 ] şi H 2 , dacă aceste cercuri au raze egale.<br />

I ′ . C 1 şi C 2 sunt tangente exterior (u = 1 + v), caz limită al celui de mai sus.<br />

Familiile de cercuri 1) şi 2) conduc şi aici la o hiperbolă H 1 (degenerată, dacă v = 1),<br />

iar familiile 3) şi 4) conduc la relaţia |OO 1 − OO 2 | = 1 + v = u, care spune că punctul<br />

mobil O aparţine lui d (d notează linia centrelor O 1 şi O 2 ) şi O /∈ [O 1 O 2 ].<br />

Să observăm, însă, că în acest caz intră în discuţie încă două familii de cercuri C:<br />

5) C este tangent interior la C 1 şi exterior la C 2 ; 6) C este tangent exterior la C 1 şi<br />

interior la C 2 . În aceste cazuri, C va fi tangent la C 1 şi C 2 în punctul T de tangenţă a<br />

acestor din urmă cercuri. În cazul 5), avem OO 1 = 1−r şi OO 2 = v +r, cu 0 ≤ r ≤ 1,<br />

adică O parcurge segmentul [O 1 T ]. În cazul 6), avem OO 1 = 1 + r şi OO 2 = v − r,<br />

cu 0 ≤ r ≤ v, adică O parcurge [T O 2 ].<br />

Cu alte cuvinte, punctul mobil O parcurge dreapta d, dacă cercul C este în cazurile<br />

3)–6); altfel spus, hiperbola H 2 degenerează în dreapta d a centrelor cercurilor fixe.<br />

Rezumând, dacă C 1 , C 2 sunt tangente exterior, locul centrelor cercurilor C tangente<br />

la aceste două cercuri este <strong>format</strong> dintr-o hiperbolă H 1 (care degenerează în<br />

mediatoarea segmentului [O 1 O 2 ] atunci când C 1 , C 2 au raze egale) şi dreapta d.<br />

Observaţie. Vom da o schiţă de rezolvare analitică a problemei enunţate în cazul<br />

cercurilor C 1 şi C 2 exterioare şi de raze diferite.<br />

Considerăm un reper cartezian drept cu originea în O 1 , având axa x-lor dreapta<br />

determinată de O 1 şi O 2 , cu sensul de la O 1 la O 2 şi unitatea de măsură egală cu raza<br />

cercului C 1 . Atunci, avem O 1 (0, 0) şi O 2 (u, 0) şi fie O(x, y).<br />

În cazul familiei 1) de cercuri C se impun condiţiile:<br />

x 2 + y 2 = (r + 1) 2 şi (x − u) 2 + y 2 = (r + v) 2 .<br />

Prin scădere, vom găsi r = u2 − 2ux − v 2 + 1<br />

2(v − 1)<br />

obţinând în cele din urmă ecuaţia locului<br />

cu care eliminăm r din prima ecuaţie,<br />

x − u 22<br />

(1 − v) 2<br />

4<br />

−<br />

y 2<br />

u 2 − (1 − v) 2<br />

adică hiperbola H 1 (ramura ”dreaptă”, căci v < 1).<br />

Aceste calcule, cât şi cele ce trebuie făcute în cazul familiilor de cercuri 2)–4), ne<br />

conving de avantajul abordării sintetice.<br />

4<br />

= 1,<br />

18


II. C 1 şi C 2 sunt secante (1 − v < u < 1 + v). Cercul C tangent la C 1 şi C 2<br />

poate fi în unul dintre următoarele cazuri: 1 ◦ C este tangent interior cercurilor C 1 , C 2 ;<br />

2 ◦ C este tangent exterior cercurilor C 1 , C 2 ; 3 ◦ C 1 , C 2 sunt tangente interior cercului<br />

C; 4 ◦ C este tangent interior la C 1 şi exterior la C 2 ; 5 ◦ C este tangent exterior la C 1 şi<br />

interior la C 2 .<br />

În cazul 1 ◦ , avem OO 1 − OO 2 = (1 − r) − (v − r) = 1 − v şi 0 ≤ r ≤ 1 + u − v ; în<br />

2<br />

cazul 2 ◦ , avem O ′ O 1 − O ′ O 2 = (1 + r) − (v + r) = 1 − v şi v ≥ 0.<br />

Împreună, aceste două cazuri dau, ca făcând parte din<br />

locul geomeric, ramura ”dreapta” a hiperbolei H 1 (cazul<br />

O<br />

1 ◦ dă arcul din partea comună cercurilor C 1 şi C 2 , iar<br />

2 ◦ restul ramurei). În cazul 3 ◦ , avem O ′′ O 2 − O ′′ O 1 =<br />

O<br />

(r − v) − (r − 1) = 1 − v şi r ≥ 1, adică ramura ”stângă”<br />

a hiperbolei H 1 .<br />

O 1<br />

O 2<br />

Cazurile 4 ◦ şi 5 ◦ (cititorul va face singur figura) conduc,<br />

împreună, la elipsa E 1 : OO 1 + OO 2 = 1 + v având<br />

focarele O 1 şi O 2 şi trecând (evident!) prin punctele de<br />

intersecţie a cercurilor C 1 şi C 2 . Într-adevăr, în cazul 4◦ punctul mobil O verifică relaţia<br />

OO 1 + OO 2 = (1 − r) + (v + r) = 1 + v, cu 0 ≤ r ≤ 1 + u − v , deci O parcurge arcul<br />

2<br />

elipsei E 1 aflat în cercul C 1 , iar în cazul 5 ◦ avem OO 1 +OO 2 = (1+r)+(v −r) = 1+v,<br />

cu 0 ≤ r ≤ u + v − 1 , adică O parcurge arcul elipsei E 1 situat în cercul C 2 .<br />

2<br />

Prin urmare, locul centrelor cercurilor tangente la două cercuri secante C 1 şi C 2<br />

este <strong>format</strong> din hiperbola H 1 (care degenerează în mediatoarea segmentului [O 1 O 2 ]<br />

dacă C 1 , C 2 au raze egale) şi elipsa E 1 .<br />

III. C 2 este interior cercului C 1 (0 ≤ u ≤ 1 − v). Avem două familii de cercuri<br />

C: i) C tangent interior la C 1 şi exterior la C 2 ; ii) C tangent interior la C 1 şi C 2<br />

tangent interior la C.<br />

În cazul i) avem OO 1 + OO 2 = (1 − r) + (v + r) = 1 + v, adică O parcurge elipsa<br />

E 1 , iar în cazul ii) avem OO 1 + OO 2 = (1 − r) + (r − v) = 1 − v > 0, adică punctul O<br />

parcurge o elipsă E 2 având ca focare, ca şi E 1 , centrele O 1 şi O 2 .<br />

Dacă C 2 este tangent interior cercului C 1 (u = 1−v), atunci constatăm cu uşurinţă<br />

că elipsa E 1 trece prin punctul de tangenţă a acestora, iar E 2 degenerează în segmentul<br />

[O 1 O 2 ]. Dacă C 1 şi C 2 sunt concentrice (u = 0), vom avea OO 1 = 1 + v<br />

2<br />

şi, respectiv,<br />

OO 1 = 1 − v<br />

2 , adică elipsele E 1 şi E 2 sunt cercuri concentrice cu C 1 , C 2 şi având razele<br />

semisuma şi respectiv semidiferenţa razelor cercurilor C 1 şi C 2 .<br />

Conchidem că, dacă C 2 este interior cercului C 1 , locul centrelor cercurilor tangente<br />

acestor cercuri este <strong>format</strong> din elipsele E 1 şi E 2 (care vor fi cercuri atunci când C 1 şi<br />

C 2 sunt concentrice); elipsa E 2 degenerează în segmentul [O 1 O 2 ] în cazul în care C 2<br />

este tangent interior la C 1 .<br />

Considerăm că problema enunţată, cu toate cazurile ce le comportă, a fost complet<br />

rezolvată.<br />

19


Cazul unui cerc fix degenerat în dreaptă. Modificăm problema rezolvată mai<br />

sus considerând o dreaptă D 1 în locul cercului C 1 . Dintre<br />

cazurile posibile, potrivit cu poziţia dreptei D 1 faţă de cercul<br />

P<br />

O<br />

C 2 , ne limităm la unul singur, cel din figura alăturată.<br />

C 2<br />

Distingem două tipuri de cercuri C tangente la dreapta<br />

D O 1 şi cercul C 2 : j) C şi C 2 sunt tangente exterior; jj) C 2 este<br />

2<br />

tangent interior cercului C. Fie d 1 şi d ′ 1 paralele la dreapta<br />

P O<br />

D 1 situate la distanţa v de aceasta.<br />

d 1 d 1<br />

În cazul j) avem OP = r + v = OO D 2 , deci punctul mobil<br />

1<br />

O descrie parabola P 1 de focar O 2 şi directoare d 1 , iar în<br />

cazul jj) avem O ′ P ′ = r ′ − v = O ′ O 2 , deci O ′ descrie parabola P 2 de focar O 2 şi<br />

directoare d ′ 1.<br />

În concluzie, în cazul considerat, locul centrelor cercurilor tangente la dreapta D 1<br />

şi cercul C 2 este <strong>format</strong> din parabolele P 1 şi P 2 .<br />

(continuare de la pagina 11)<br />

Când o să-nsereze,<br />

Vor să te-ANULEZE<br />

Funcţia INJECTIVĂ<br />

Şi cea SURJECTIVĂ!<br />

- Dacă s-o-ntâmpla<br />

De m-or ANULA,<br />

Să mă-ngropi în zori<br />

În CÂMP DE VECTORI<br />

Într-o VECINĂTATE<br />

Pe-aici, pe-aproape,<br />

Sau chiar în MULŢIME,<br />

Să fiţi tot cu mine!<br />

Iar la cap să-mi pui<br />

CALCUL INTEGRAL,<br />

Ori un MANUAL,<br />

Sau poate-un TRATAT<br />

Cât mai inspirat ...<br />

Şi de l-or citi<br />

Îşi vor aminti<br />

Cei ce au uitat<br />

Că am existat,<br />

Şi voi fi propusă,<br />

În SUBIECTE inclusă,<br />

Pentru OLIMPIADĂ<br />

Sau BALCANIADĂ ...<br />

Şi-n loc de ANULAT,<br />

Să le spui curat<br />

C-am INTERSECTAT<br />

Mândrele ELIPSE,<br />

Că am PUNCTE FIXE,<br />

RĂDĂCINI REALE<br />

Şi IMAGINARE<br />

Şi că am DARBOUX!<br />

Iar dacă-i zări,<br />

Dacă-i întâlni<br />

O SFERĂ bătrână,<br />

Cu un CERC de lână,<br />

Prin SPAŢIU alergând,<br />

De toţi întrebând<br />

Şi la toţi zicând :<br />

≪Cine mi-a văzut<br />

Sau mi-a cunoscut<br />

O FUNCŢIE - AFINĂ,<br />

Cu o PANTĂ lină,<br />

Bine DEFINITĂ<br />

Şi NEMĂRGINITĂ...≫<br />

Să te-nduri de ea<br />

Şi să-i spui aşa :<br />

C-am INTERSECTAT<br />

Mândrele ELIPSE,<br />

20<br />

Că am PUNCTE FIXE,<br />

Rădăcini COMPLEXE<br />

Şi că am DARBOUX ...<br />

Dar nu-i spune tu<br />

De cele REALE,<br />

Că, de-i povesti,<br />

Mult ai s-o mâhneşti<br />

Şi va şti de-ndat<br />

Că m-au ANULAT...<br />

Şi încă te mai rog,<br />

Ca-ntre colegi buni,<br />

Tot ce am avut<br />

Tu să le aduni,<br />

Să le scoţi din SPAŢIUL<br />

Cu trei DIMENSIUNI...<br />

Iar tu, dragul meu,<br />

Să te INTEGREZI,<br />

Să te ANEXEZI<br />

La altă MULŢIME,<br />

Că-i greu fără mine,<br />

Dar îţi va fi bine<br />

şi vei rezista,<br />

Cât va EXISTA<br />

MATEMATICA!


Aplicaţii ale teoremei lui Van Aubel<br />

Omer CERRAHOGLU 1<br />

Rezultatul asupra căruia ne îndreptăm atenţia în această notă este următorul:<br />

Teoremă (Van Aubel). Se consideră triunghiul ABC şi trei ceviene AM, BN<br />

şi CP, concurente în O; atunci AP<br />

P B + AN<br />

NC = AO<br />

OM .<br />

Demonstraţie. Aplicând teorema lui Menelaus în △ABM, cu transversala<br />

P -O-C, obţinem AP<br />

A<br />

P B · BC<br />

CM · MO<br />

AP<br />

= 1, de unde<br />

OA P B = CM<br />

BC · AO<br />

OM .<br />

Analog se arată că AN<br />

NC = BM<br />

BC · AO . Însumând cele două relaţii,<br />

OM P<br />

deducem că<br />

O<br />

N<br />

AP<br />

P B + AN<br />

NC = AO<br />

OM ·<br />

CM<br />

BC + BM<br />

BC‹= AO<br />

OM · BC<br />

BC = AO<br />

OM .<br />

B M C<br />

Această teoremă poate fi aplicată în probleme în care apar ceviene concurente şi<br />

se cunosc anumite rapoarte sau sume de rapoarte.<br />

Problema 1. Se consideră triunghiul ABC şi cevienele AM, BN şi CP concurente<br />

în O. Arătaţi că AP<br />

P B + AN<br />

NC = 1 dacă şi numai dacă A BOC = 1 2 · A ABC.<br />

D. Şt. Marinescu, V. Cornea, Lista scurtă, O.N.M., 2008<br />

Soluţie. Cum A BOC d(O, BC)<br />

=<br />

A ABC d(A, BC) = OM<br />

AM , deducem că A BOC = 1 2 A ABC ⇔<br />

OM<br />

AM = 1 2 ⇔ AO<br />

AO<br />

= 1. Din teorema lui Van Aubel, obţinem că<br />

OM OM<br />

= 1 ⇔<br />

AP<br />

P B + AN = 1, ceea ce încheie soluţia problemei.<br />

NC<br />

Problema 2. Se consideră triunghiul ascuţitunghic ABC, cu laturi de lungimi<br />

diferite. Fie M ∈ (BC), O ∈ (AM), iar X şi Y punctele în care CO, respectiv BO,<br />

intersectează a doua oară cercul circumscris triunghiului. Dacă AX<br />

BX + AY<br />

CY = AO<br />

OM ,<br />

arătaţi că latura BC este cea de lungime mijlocie.<br />

Omer Cerrahoglu<br />

Soluţie. Notăm {N} = BY ∩ AC, {P } = CX ∩ AB. Observăm că AX<br />

BX =<br />

AX · XP · sinÕAXC<br />

BX · XP · sinÕBXC · sinÕBXC<br />

sinÕAXC = 2 · A AXP<br />

· sin A<br />

2 · A BXP sin B<br />

urmare AX<br />

XB = AP<br />

BP · sin A<br />

sin B<br />

. Analog se arată că<br />

AY<br />

Y C = AN<br />

1 Elev, cl. a VII-a, Colegiul Naţional ”Vasile Lucaciu”, Baia Mare<br />

d(X, AB) · AP<br />

=<br />

d(X, AB) · BP · sin A<br />

sin B , prin<br />

CN · sin A . Ţinând seama de<br />

sin C<br />

21


ipoteza problemei şi de teorema lui Van Aubel, obţinem că<br />

(∗)<br />

AP<br />

P B + AN<br />

NC = AP<br />

P B · sin A<br />

sin B + AN<br />

NC · sin A<br />

sin C ,<br />

ambii membri ai acestei identităţi fiind egali cu AO<br />

OM . Pentru<br />

a arăta că BC este latura de lungime mijlocie, ar fi suficient<br />

B M C<br />

să demonstrăm că A ≠ min{A, B, C} şi A ≠ max{A, B, C}.<br />

Presupunem, prin absurd, că A = min{A, B, C}; atunci sin A < sin B şi sin A < sin C,<br />

AP<br />

deci<br />

P B · sin A<br />

sin B < AP<br />

P B şi AN<br />

NC · sin A<br />

sin C < AN , în contradicţie cu relaţia (∗). Analog<br />

NC<br />

se arată că A ≠ max{A, B, C} şi astfel problema este rezolvată.<br />

Problema 3. Fie I centrul cercului înscris în △ABC, {A ′ } = AI ∩ BC, {B ′ } =<br />

BI ∩ AC, {C ′ } = CI ∩ AB. Arătaţi că 1 4 < AI · BI · CI<br />

AA ′ · BB ′ · CC ′ ≤ 8 27 . (O.I.M., 1991)1 .<br />

Soluţie. Folosind teorema lui Van Aubel şi teorema bisectoarei, obţinem că<br />

AI<br />

IA ′ = AC′<br />

BC ′ + AB′<br />

CB ′ = AB<br />

BC + AC AB + AC<br />

= ⇒ AI<br />

BC BC AA ′ = AB + AC<br />

AB + AC + BC .<br />

Scriind încă două relaţii analoage şi folosind notaţiile uzuale într-un triunghi, inegalitatea<br />

din enunţ, devine<br />

1 (a + b)(b + c)(c + a)<br />

<<br />

4 (2p) 3 ≤ 8 27 .<br />

Pentru a demonstra inegalitatea din dreapta, folosim inegalitatea mediilor:<br />

a + b<br />

2p · b + c<br />

2p · c + a ≤•1<br />

2p 3 · a + b<br />

2p<br />

+ b + c<br />

‹<br />

2p<br />

‹ ‹<br />

cu egalitate în cazul triunghiului echilateral. Pentru a demonstra inegalitatea din<br />

stânga, folosim inegalitatea lui Bernoulli generalizată:<br />

a + b<br />

2p · b + c<br />

2p · c + a<br />

2p = 1 8 1 + p − c 1 + p − a 1 + p − b<br />

p<br />

p<br />

p<br />

> 1 8 1 + p − c + p − a + p − b ‹= 1 p p p 4 .<br />

Încheiem prin a propune spre rezolvare, celor interesaţi, două probleme.<br />

X<br />

P<br />

A<br />

O N<br />

+ c + a<br />

2p<br />

‹˜3<br />

= 2 3‹3<br />

= 8 27 ,<br />

Problema 4. Se consideră patrulaterul convex ABCD şi fie {O} = AC ∩ BD,<br />

M mijlocul lui [AO], N mijlocul lui [CO], {R} = DM ∩ AB, {P } = DN ∩ BC. Dacă<br />

AR<br />

RB + CP = 1, demonstraţi că ABCD este paralelogram.<br />

P B<br />

Omer Cerrahoglu<br />

Problema 5. Se consideră triunghul ascuţiunghic ABC, înscris în cercul C.<br />

Bisectoarea unghiului bA intersectează C în S, iar perpendiculara din S pe BC intersectează<br />

a doua oară C în R. Fie M mijlocul lui [RS], {P } = CM ∩ AB, şi<br />

{Q} = BM ∩ AC. Dacă AP<br />

P B + AQ<br />

QC = 1 , demonstraţi că AB = AC.<br />

cos A<br />

Omer Cerrahoglu<br />

1 N.R. Prin problema L36 din RecMat 1/2003, M. Ionescu generalizează acest rezultat, considerând<br />

I ca fiind punct arbitrar în interiorul sau pe laturile triunghiului median al △ABC.<br />

Y<br />

22


Une application de l ′ inversion<br />

Adrien REISNER 1<br />

Etant donné un cercle (C) de centre C et un point A on considère un cercle variable<br />

(S) de centre S passant par A et orthogonal à (C), voir figure. Les deux cercles (C)<br />

et (S) se coupent en P et Q .<br />

Proposition 1. Le lieu géométrique du centre S est une droite.<br />

Démonstration. Soit B le deuxième point commun de la droite CA et du cercle<br />

(S). Les deux cercles (C) et (S) étant orthogonaux la droite CP est tangente en P au<br />

cercle (S). On a – la puissance du point C par rapport au cercle (S) –: CA·CB = CP 2<br />

et par suite le point B est fixe. Le cercle (S) passant par les deux points fixes A et<br />

B le lieu géométrique de son centre S est la droite (∆) perpendiculaire à AB menée<br />

par le milieu I du segment AB.<br />

Proposition 2. La corde P Q commune aux cercles (C) et (S) passe par un point<br />

fixe.<br />

Démonstration. La droite P Q est la polaire du point S par rapport au cercle<br />

(C). Le lieu géométrique du point S étant la droite (∆), la polaire du point S passe<br />

par un point fixe D pôle de (∆) par rapport au cercle (C). Les points A, C, B, D sont<br />

alignés sur une droite perpendiculaire à la droite (∆).<br />

Les droites AP et AQ rencontrent à nouveau le cercle (C) respectivement aux<br />

points P ′ et Q ′ .<br />

Proposition 3. La droite P ′ Q ′ passe par un point fixe.<br />

Démonstration. Considérons l ′ inversion ayant pour pôle le point A et pour<br />

module la puissance du point A par rapport au cercle (C). Ce cercle (C) coïncide<br />

1 Centre de Calcul E.N.S.T., Paris; e-mail: adrien.reisner@enst.fr<br />

23


avec son inverse et les points P ′ et Q ′ sont les inverses des points P et Q. La droite<br />

P ′ Q ′ est l ′ inverse du cercle (AP Q) c ′ est – a – dire du cercle (S). Le cercle (S) étant<br />

orthogonal au cercle (C) on en déduit que la droite P ′ Q ′ passe par le point fixe C.<br />

Remarque. Le cercle (S) passe aussi par le point fixe B. Donc P ′ Q ′ passe par<br />

l ′ inverse du point B et par suite les points B et C sont des points inverses l ′ un de<br />

l ′ autre.<br />

Proposition 4. Le cercle circonscrit au triangle passe par un point fixe.<br />

Démonstration. Le cercle circonscrit au triangle AP ′ Q ′ est l ′ inverse de la droite<br />

P Q. Cette droite P Q passant par le point fixe D, le cercle (AP ′ Q ′ ) passe par le point<br />

fixe D ′ inverse du point D. Démontrons que ce point D ′ est le symétrique de B par<br />

rapport au point C. En effet en considérant les puissances du point C par rapport<br />

aux cercles (S) et (AP ′ Q ′ ) on a:<br />

CA · CB = CP 2 et CA · CD ′ = CP ′ · CQ ′ = −CP 2 .<br />

On en déduit immédiatement que CB = −CD ′ , d ′ où la propriété annoncée.<br />

Références<br />

1. J. Commeau - Géométrie (pages 398–424), Edition Masson.<br />

2. A. Lentin, G. Girard - Géométrie. Mécanique (pages 303–328), Edition Hachette.<br />

1 × 8 + 1 = 9<br />

12 × 8 + 2 = 98<br />

123 × 8 + 3 = 987<br />

1234 × 8 + 4 = 9876<br />

12345 × 8 + 5 = 98765<br />

123456 × 8 + 6 = 987654<br />

1234567 × 8 + 7 = 9876543<br />

12345678 × 8 + 8 = 98765432<br />

123456789 × 8 + 9 = 987654321<br />

1 × 9 + 2 = 11<br />

12 × 9 + 3 = 111<br />

123 × 9 + 4 = 1111<br />

1234 × 9 + 5 = 11111<br />

12345 × 9 + 6 = 111111<br />

123456 × 9 + 7 = 1111111<br />

1234567 × 9 + 8 = 11111111<br />

12345678 × 9 + 9 = 111111111<br />

123456789 × 9 + 10 = 1111111111<br />

24


O demonstraţie simplă a inegalităţii mediilor<br />

Claudiu-Ştefan POPA 1<br />

Ne popunem să prezentăm o cale de introducere a inegalităţii mediilor la nivelul<br />

clasei a VII-a. Considerăm că această metodă are o serie de avantaje: este uşor de<br />

expus de către profesor, facil de înţeles de către elevi, permite recapitularea formulelor<br />

de calcul prescurtat şi deschide copiilor noi orizonturi.<br />

În cele ce urmează, a şi b sunt numere reale pozitive, cu a ≤ b. Mediile uzuale sunt:<br />

M h =<br />

2ab<br />

a + b (media armonică); M g = √ ab (media geometrică); M a = a + b (media<br />

2<br />

aritmetică); M p =ra 2 + b 2<br />

(media pătratică); M ap = a2 + b 2<br />

(media aritmetică<br />

2<br />

a + b<br />

ponderată a numerelor a şi b, cu ponderile a, respectiv b).<br />

Se observă că a ≤ M g ≤ b (prima inegalitate revine la a ≤ √ ab ⇔ a 2 ≤ ab ⇔<br />

a ≤ b, iar a doua la √ ab ≤ b ⇔ ab ≤ b 2 ⇔ a ≤ b). Analog se justifică faptul că<br />

a ≤ M a ≤ b. Reţinem, deci, că media geometrică şi media aritmetică a două numere<br />

sunt cuprinse între numărul mai mic şi cel mai mare.<br />

<br />

Să calculăm acum media aritmetică a numerelor M h şi M ap :<br />

22ab 1<br />

a + b + a2 + b 2<br />

a + b<br />

= 1 2 · a2 + 2ab + b 2<br />

= 1 (a + b)2 · = a + b = M a . Pe de altă parte, este evident că<br />

a + b 2 a + b 2<br />

M h ≤ M ap , deoarece această inegalitate revine la 2ab ≤ a 2 + b 2 ⇔ 0 ≤ (a − b) 2 ,<br />

adevărat. Conform celor de mai sus, obţinem că M h ≤ M a ≤ M ap , cu egalitate când<br />

a = b.<br />

Media geometrică a numerelor M h şi M a ester2ab<br />

a + b · a + b<br />

2<br />

cum M h ≤ M a , deducem că M h ≤ M g ≤ M a , cu egalitate când a = b.<br />

Media geometrică a numerelor M a şi M ap estera + b<br />

2<br />

· a2 + b 2<br />

a + b<br />

= √ ab = M g şi,<br />

=ra 2 + b 2<br />

2<br />

M p . Deoarece M a ≤ M ap , rezultă că M a ≤ M p ≤ M ap , cu egalitate când a = b.<br />

Concluzionăm că, date numerele reale pozitive a şi b, între mediile lor există şirul<br />

de inegalităţi<br />

M h ≤ M g ≤ M a ≤ M p ≤ M ap ,<br />

care se transformă în egalităţi pentru a = b.<br />

=<br />

1 Profesor, Şcoala ”Alecu Russo”, Iaşi<br />

25


O demonstraţie a teoremei a doua a lui Ptolemeu<br />

Gheorghe COSTOVICI 1<br />

În [1], pag. 134–137, se arată echivalenţa afirmaţiilor:<br />

1) (prima teoremă a lui Ptolemeu) dacă un patrulater este inscriptibil, atunci<br />

produsul diagonalelor este egal cu suma produselor laturilor opuse;<br />

2) dacă α + β + γ + δ = π (α, β, γ, δ ∈ R ∗ +), atunci<br />

sin(α + γ) sin(β + γ) = sin α sin β + sin γ sin δ<br />

şi apoi, stabilind această identitate, se deduce justeţea primei teoreme a lui Ptolemeu.<br />

Ne propunem un demers similar în privinţa celei de-a doua teoreme a lui Ptolemeu.<br />

Propoziţie. Dacă α + β + γ + δ = π (α, β, γ, δ ∈ R ∗ +), atunci<br />

(1) sin(α + γ)(sin α sin γ + sin β sin δ) = sin(β + γ)(sin α sin δ + sin β sin γ).<br />

Demonstraţie. Ţinând seama de ipoteză, obţinem:<br />

4 sin(α + γ)(sin α sin γ + sin β sin δ) = 2 sin(α + γ)[cos(α − γ) − cos(α + γ)<br />

+ cos(β − δ) − cos(β + δ)] = 2 sin(α + γ)[cos(α − γ) + cos(β − γ)]<br />

= sin 2α + sin 2γ + sin(α + γ + β − δ) + sin(α + γ − β + δ)<br />

= sin 2α + sin 2β + sin 2γ + sin 2δ.<br />

La acelaşi rezultat ajungem dacă pornim de la membrul doi al identităţii din enunţ<br />

multiplicat cu 4, ceea ce încheie demonstraţia.<br />

Teorema a doua a lui Ptolemeu. Dacă ABCD este un patrulater inscriptibil,<br />

atunci are loc relaţia<br />

(2)<br />

AC AB · AD + CB · CD<br />

=<br />

BD BA · BC + DA · DC .<br />

Demonstraţie.R fiind raza cercului circumscris<br />

patrulaterului, avem: AB = 2R sin α, BC = 2R sin δ,<br />

CD = 2R sin β, DA = 2R sin γ, AC = 2R sin(β + γ)<br />

şi BD = 2R sin(α + γ).<br />

Observând că α + β + γ + δ = π, rezultă că are loc<br />

(1), care, prin trecere la laturi pe baza egalităţilor de<br />

mai sus, conduce la relaţia de demonstrat.<br />

A<br />

B<br />

<br />

<br />

Observaţie. Propoziţia şi Teorema sunt echivalente, în sensul că se implică una<br />

pe alta. Cum s-a văzut că Propoziţia implică Teorema, rămâne de stabilit implicaţia<br />

inversă. Fie date α, β, γ, δ ∈ R ∗ + cu α + β + γ + δ = π. Pe un cerc de rază 1 luăm<br />

arce de lungimi 2α, 2β, 2γ, 2δ, cu suma egaltă cu 2π (vezi figura). Înlocuind în (2)<br />

lungimile AB, BC etc. cu expresiile lor, obţinem imediat relaţia (1).<br />

Bibliografie<br />

1. I.M. Ghelfand, M. Saul - Trigonometry, Birkhäuser, 2001.<br />

1 Conf.dr., Univ. Tehnică ”Gh. Asachi”, Iaşi<br />

<br />

<br />

<br />

<br />

<br />

D<br />

<br />

C<br />

26


Asupra determinării imaginii unei funcţii<br />

de mai multe variabile<br />

Daniel VĂCARU 1<br />

Vom insista asupra unei metode mai speciale de demonstrare a inegalităţilor, care<br />

se pretează la generalizări şi pe care o vom folosi în rezolvarea unor probleme dificile.<br />

Primele două aplicaţii pot fi găsite în aproape orice culegere de probleme de algebră<br />

pentru liceu, iar următoarele au apărut în ultimul timp în reviste de matematică<br />

elementară, ca probleme propuse.<br />

Problema 1. Determinaţi a ∈ R astfel încât<br />

x 2 + y 2 + z 2 + 4x + 2y + 2z − a > 0, ∀x, y, z ∈ R.<br />

Soluţie. Gândim expresia din membrul stâng al inegalităţii din enunţ ca funcţie<br />

de variabila reală x, cu y şi z parametri. Prin urmare, vom considera f : R → R,<br />

f(x) = x 2 +4x+(y 2 +z 2 +2y+2z−a), pe care o vom deriva, obţinând că f ′ (x) = 2x+4.<br />

Cum f ′ (x) < 0, ∀x ∈ (−∞, −2), f ′ (−2) = 0 şi f ′ (x) > 0, ∀x ∈ (−2, +∞), deducem<br />

că x = −2 este punct de minim, iar Imf = [f(−2), +∞], unde f(−2) = y 2 + z 2 + 2y +<br />

2z − a − 4. Dorim ca f(x) > 0, ∀x ∈ R, şi atunci vom impune condiţia f(−2) > 0.<br />

Considerăm funcţia g : R → R, g(y) = y 2 + 2y + (z 2 + 2z − a − 4), cu derivata<br />

g ′ (y) = 2y + 2. Obţinem că y = −1 este punct de minim al lui g, iar valoarea minimă<br />

este g(−1) = z 2 + 2z − a − 5. Vrem ca g(−1) > 0, ∀z ∈ R, fapt care conduce la<br />

h(z) > 0, ∀z ∈ R, unde h : R → R, h(z) = z 2 + 2z − a − 5. Deducem, din nou<br />

cu ajutorul derivatei, că h(z) ≥ h(−1) = a − 6 şi cum trebuie verificată condiţia<br />

h(−1) > 0, obţinem că a ∈ (−∞, −6) este condiţie necesară şi suficientă pentru ca<br />

inegalitatea din enunţ să aibă loc pentru orice x, y, z ∈ R.<br />

Problema 2. Pe mulţimea G = (−1, 1) se consideră operaţia x ∗ y =<br />

x + y<br />

1 + xy ,<br />

∀x, y ∈ G. Demonstraţi că G este parte stabilă faţă de operaţia definită.<br />

Soluţie. Pe aceeaşi idee, considerăm funcţia f : (−1, 1) → R, f(x) =<br />

x + y<br />

1 + xy ,<br />

unde y ∈ (−1, 1) este un parametru. Cum f ′ (x) =<br />

1 − y2<br />

> 0, ∀x, y ∈ (−1, 1),<br />

(1 + xy)<br />

2<br />

deducem că f este strict crescătoare, prin urmare lim f(x) < f(x) < lim f(x),<br />

x↘−1 x↗1<br />

−1 + y<br />

∀x ∈ (−1, 1). Dar lim f(x) = = −1, iar lim<br />

x↘−1 1 − y f(x) = 1 + y = 1, astfel că<br />

x↗1 1 + y<br />

−1 < f(x) < 1, ∀x ∈ (−1, 1), adică x ∗ y ∈ (−1, 1), ∀x, y ∈ (−1, 1).<br />

Problema 3. Demonstraţi că | − 3xy + x + y| ≤ 1, ∀x, y ∈ [0, 1]. (Ovidiu Pop,<br />

Problema VIII.82, RecMat-2/2007, p.151)<br />

Soluţie. Considerăm funcţia f : R → R, f(x) = (−3y + 1)x + y, unde y ∈ [0, 1].<br />

Avem de studiat o funcţie de gradul I pe R. Dacă y = 1 , funcţia este constant egală<br />

3<br />

1 Profesor, Colegiul Naţional ”Zinca Golescu”, Piteşti<br />

27


cu 1 3 , deci f(x) ≤ 1, ∀x ∈ R. Dacă y ∈•0, 1 3‹, atunci f este strict crescătoare, prin<br />

urmare 0 ≤ y = f(0) ≤ f(x) ≤ f(1) = −2y + 1 ≤ 1, ∀x ∈ [0, 1]. În sfârşit, dacă<br />

y ∈<br />

1 3 , 1˜, atunci f este strict descrescătoare, astfel că −1 ≤ −2y + 1 = f(1) ≤<br />

f(x) ≤ f(0) = y ≤ 1, ∀x ∈ [0, 1]. Analiza făcută probează cerinţa problemei.<br />

Problema 4. Demonstraţi că în orice triunghi ABC, cu notaţiile uzuale, are loc<br />

inegalitatea<br />

a<br />

2 · 4r − R<br />

R ≤È(p − a)(p − b) ≤ a 2 .<br />

(Alexandru Roşoiu, Problema 11306, American Mahematical Monthly).<br />

Soluţie. Începem prin a nota x = p−a, y = p−b, z = p−c; evident că x, y, z > 0.<br />

Atunci<br />

r<br />

R = S p : abc<br />

4S = 4S2 4p(p − a)(p − b)(p − c)<br />

4xyz<br />

= =<br />

pabc pabc<br />

(x + y)(y + z)(z + x) .<br />

Inegalitatea din enunţ devine<br />

x + y<br />

2<br />

16xyz<br />

(x + y)(y + z)(z + x) − 1‹≤ √ xy ≤ x + y , ∀x, y, z > 0.<br />

2<br />

Inegalitatea din dreapta rezultă din inegalitatea mediilor, cu egalitate în cazul în<br />

care x = y, i.e. a = b. Pentru a demonstra inegalitatea din stânga, considerăm<br />

funcţia f : R ∗ z<br />

+ → R, f(z) = 8xy ·<br />

(x + z)(y + z) − x + y − √ xy. Avem că f ′ (z) =<br />

2<br />

−z 2 + xy<br />

8xy ·<br />

(x + z) 2 · (y + z) 2 , prin urmare f ′ (z) > 0, ∀z ∈ (0, √ xy), f ′ ( √ xy) = 0 şi<br />

f ′ (z) < 0, ∀z ∈ ( √ xy, +∞). Deducem că f are un maxim în √ xy, iar f( √ xy) =<br />

8xy<br />

( √ x + √ y) − (√ x + √ y) 2<br />

. Inegalitatea f( √ xy) ≤ 0 revine la 16xy ≤ ( √ x + √ y) 4 ,<br />

2 2<br />

care este adevărată conform inegalităţii mediilor. Egalitate se atinge în cazul triunghiului<br />

echilateral.<br />

1 × 1 = 1<br />

11 × 11 = 121<br />

111 × 111 = 12321<br />

1111 × 1111 = 1234321<br />

11111 × 11111 = 123454321<br />

111111 × 111111 = 12345654321<br />

1111111 × 1111111 = 1234567654321<br />

11111111 × 11111111 = 123456787654321<br />

111111111 × 111111111 = 12345678987654321<br />

28


Principiul extremal<br />

Gabriel POPA 1<br />

Ne vom referi în cele ce urmează la o importantă metodă de raţionament, utilă în<br />

soluţionarea unei importante clase de probleme. Ideea este aceea de a ne concentra<br />

asupra celui mai mare/mic element al unei mulţimi asociată problemei şi de a vedea ce<br />

informaţii oferă acest ”element extremal”. Exemplul tipic este cel din demonstraţia<br />

lui Euclid pentru caracterul infinit al mulţimii numerelor prime: se presupune că<br />

mulţimea numerelor prime ar fi finită, rezultă existenţa unui cel mai mare număr<br />

prim p şi construim un element al mulţimii numerelor prime mai mare decât p. Elevii<br />

întâlnesc în şcoală aceeaşi idee atunci când demonstrează iraţionalitatea lui √ 2 sau<br />

existenţa celui mai mare divizor comun cu ajutorul algoritmului lui Euclid.<br />

Precizăm că următoarele două tipuri de mulţimi se pretează la aplicarea principiului<br />

extremal: 1) mulţimile finite de numere reale, care au atât un cel mai mare, cât<br />

şi un cel mai mic element şi 2) submulţimile lui N, care au întotdeauna un cel mai mic<br />

element. În cel de-al doilea caz, principiul îmbracă uneori forma coborârii infinite, pe<br />

care o vom prezenta mai jos.<br />

Există (cel puţin) trei materiale foarte bune şi accesibile ([1], [2, pp.15-16], [3,<br />

pp.53-75]) cu ajutorul cărora se poate realiza familiarizarea cu principiul extremal. Ne<br />

propunem să prezentăm alte câteva probleme, care să vină în sprijinul celor interesaţi<br />

de acest subiect.<br />

Problema 1. Într-un turneu de şah, fiecare dintre jucători dispută câte o partidă<br />

cu fiecare dintre ceilalţi. Ştiind că nu se înregistrează nicio remiză şi că fiecare<br />

participant obţine cel puţin câte o victorie, demonstraţi că există un grup de trei<br />

şahişti A, B, C astfel încât A îl învinge pe B, B îl învinge pe C şi C îl învinge pe A.<br />

(Olimpiadă Irlanda, 2004)<br />

Soluţie (Titu Zvonaru). Fie P 1 , P 2 , . . . , P n şahiştii care participă la turneu<br />

şi notăm cu a i numărul învinşilor jucătorului P i . Din ipoteza problemei, avem că<br />

a i ≥ 1, ∀i = 1, n. Mulţimea finită {a 1 , a 2 , . . . , a n } are un cel mai mic element; putem<br />

presupune că acesta este a 1 şi fie P 2 , P 3 , . . . , P a1 +1 şahiştii pe care îi învinge P 1 . Cum<br />

a 1 ≤ a 2 , există un participant P k învins de către P 2 , unde k ≥ a 1 + 2 . Considerând<br />

A = P 1 , B = P 2 şi C = P k , avem îndeplinită cerinţa problemei.<br />

Problema 2. Spunem că o mulţime M ⊂ R + are proprietatea (P ) dacă orice<br />

element al lui M este media geometrică a două elemente distincte ale lui M.<br />

a) O mulţime cu 2005 elemente poate avea proprietatea (P )<br />

b) Arătaţi că există o infinitate de mulţimi cu proprietatea (P ).<br />

(Gabriel Popa şi Paul Georgescu, Recreaţii <strong>Matematice</strong>-2/2005, p.170)<br />

Soluţie. a) În general, nicio mulţime finită M nu poate avea proprietatea (P ).<br />

Într-adevăr, dacă M este o mulţime finită, atunci M are un cel mai mare element, fie<br />

acesta a. Dacă a ar fi media geometrică a elementelor b, c ∈ M, cu b < c, ar trebui<br />

să avem b < a < c. Astfel, c este un element al lui M strict mai mare decât a şi se<br />

ajunge la o contradicţie.<br />

1 Profesor, Colegiul Naţional, Iaşi<br />

29


) Se observă că pentru orice x ∈ R ∗ +\{1}, mulţimea M x = {x n |n ∈ Z} are<br />

proprietatea (P ), deoarece x n = √ 1 · x 2n , n ∈ Z ∗ , iar 1 = √ x n · x −n .<br />

Problema 3. Spunem că o mulţime A ⊂ N ∗ are proprietatea (P ) dacă ∀a, b ∈ A,<br />

există c ∈ A\{a, b} astfel încât a, b, c să fie lungimile laturilor unui triunghi.<br />

a) N ∗ are proprietatea (P )<br />

b) Demonstraţi că există o infinitate de mulţimi având proprietea (P ).<br />

(Gabriel Popa, Concursul ”Radu Miron”, 2003)<br />

Soluţie. a) Vom lua în considerare cel mai mic element al lui N ∗ , anume pe 1.<br />

Dacă b, c ∈ N ∗ \{1}, b ≠ c, este evident că numerele 1, b, c nu pot fi lungimile laturilor<br />

unui triunghi (dacă b < c, atunci 1+b ≤ c şi este contrazisă inegalitatea triunghiului).<br />

b) Dacă n ∈ N, n ≥ 4, atunci mulţimea A n = {2, 3, . . . , n} are proprietatea (P ).<br />

Într-adevăr, dacă a, b ∈ A şi a ≤ b − 2, putem considera c = b − 1, iar dacă a = b − 1,<br />

luăm convenabil c ∈ {b − 2, b + 1}.<br />

Problema 4. Fie n ∈ N ∗ şi A mulţimea numerelor naturale cel puţin egale cu<br />

2, care sunt relativ prime cu fiecare dintre numerele 1, 2 . . . , n. Demonstraţi că A nu<br />

poate conţine doar numere compuse.<br />

Soluţie. Cum A este o mulţime de numere naturale, va conţine un cel mai mic<br />

element, fie acesta p. Vom demonstra că p este prim, de unde concluzia problemei.<br />

Pentru aceasta, să presupunem prin absurd că p nu ar fi prim; cum p ≥ 2, înseamnă<br />

că este compus, prin urmare există q prim şi k ≥ 2 astfel încât p = q · k. Numărul<br />

prim q nu divide niciunul dintre numerele 1, 2 . . . , n şi atunci q ∈ A. Pe de altă parte,<br />

q < p, ceea ce contrazice faptul că p este cel mai mic element al lui A.<br />

Problema 5. Fie x, y, z trei numere prime distincte. Demonstraţi că 30(xy +<br />

yz + zx) ≤ 31xyz.<br />

(Marius Ghergu, Concursul ”Florica T. Câmpan”, 2005)<br />

Soluţie. Încercăm să folosim buna ordonare a mulţimii numerelor prime. Împărţind<br />

ambii membri prin 30xyz, inegalitatea dată se scrie echivalent sub forma 1 x +<br />

1<br />

y + 1 z ≤ 31 . Cum valorile minime ale numerelor prime distincte x, y, z sunt 2, 3 şi 5,<br />

30<br />

valoarea maximă a sumei 1 x + 1 y + 1 z este 1 2 + 1 3 + 1 5 = 31 , fapt care încheie rezolvarea<br />

30<br />

problemei.<br />

Problema 6. Pe tablă sunt scrise numerele √ 3−1, √ 3+1 şi 2. Se şterg numerele<br />

şi se scriu în locul lor cele trei medii geometrice a câte două dintre numerele iniţiale.<br />

Procedeul se repetă cu noile numere. Este posibil ca, după mai mulţi paşi, să avem<br />

pe tablă 2 − √ 3, 2 + √ 3 şi 4<br />

(Monica Nedelcu, Concursul ”Florica T. Câmpan”, 2007)<br />

Soluţie. Teoretic, asemenea probleme se rezolvă folosind principiul invariantului;<br />

în cazul problemei date, încercările de a găsi un invariant (sau măcar un semiinvariant)<br />

nu dau prea repede roade! Observăm însă că, întrucât media geometrică a două<br />

numere distincte este strict cuprinsă între numărul mai mic şi cel mai mare, numărul<br />

maxim de pe tablă scade la fiecare repetare a operaţiei. Iniţial, acest maxim era<br />

30


√<br />

3 + 1 şi deducem că nu vom putea face în aşa fel încât pe tablă să apară numărul<br />

mai mare 4.<br />

Problema 7. Demonstraţi că nu există triunghiuri dreptunghice având catetele<br />

numere raţionale, iar ipotenuza egală cu √ 2001.<br />

(Constantin Cocea, RecMat-1/2002, p.81)<br />

Soluţia. Pentru a arăta că ecuaţia x2<br />

y 2 + z2<br />

u 2 = 2001 nu are soluţii în N∗ , este<br />

suficient să demonstrăm că ecuaţia m 2 + n 2 = 2001p 2 (∗) nu are soluţii în N ∗ . Vom<br />

utiliza metoda coborârii, o variantă a principiului extremal. Presupunem că ecuaţia<br />

(∗) are soluţii în N ∗ . Cum N ∗ este bine ordonată, putem considera acea soluţie pentru<br />

care p este minim. Observăm că 3|m 2 + n 2 (deoarece 2001 se divide cu 3) şi atunci<br />

m şi n sunt multipli de 3, după cum se poate uşor demonstra. Fie m = 3m 1 , n = 3n 1<br />

cu m 1 , n 2 ∈ N ∗ ; din (∗) obţinem că 3(m 2 1 + n 2 1) = 667p 2 şi, cum (3, 667) = 1, atunci<br />

p = 3p 1 , p 1 ∈ N ∗ . După înlocuire, m 2 1 + n 2 1 = 2001p 2 1, prin urmare (m 1 , n 1 , p 1 ) este o<br />

nouă soluţie a ecuaţiei (∗), cu p 1 < p. Am contrazis astfel minimalitatea asumată a<br />

lui p şi urmează că ecuaţia (∗) nu are soluţii în N ∗ .<br />

Problema 8. Se consideră şase discuri astfel încât frontierele lor au un punct<br />

comun. Demonstraţi că există o pereche de discuri astfel încât unul dintre ele conţine<br />

centrul celuilalt.<br />

Soluţie. Fie O 1 , O 2 , . . . , O 6 centrele celor şase discuri, iar A punctul comun<br />

al frontierelor lor. Considerăm toate unghiurile cu vârful în A şi având ca laturi<br />

semidrepte de forma [AO i , i = 1, 6. Dacă unul dintre aceste unghiuri este nul,<br />

cerinţa problemei este imediată. Dacă toate sunt nenule, considerăm un unghi de<br />

măsură minimă, fie acesta×O 1 AO 2 . Presupunând, fără a restrânge generalitatea,<br />

că AO 2 ≥ AO 1 , vom avea că m(×O 1 AO 2 ) ≤ 60 ◦ , m(×AO 1 O 2 ) ≥ 60 ◦ , prin urmare<br />

O 1 O 2 ≤ AO 2 şi astfel deducem că O 1 aparţine discului de centru O 2 .<br />

Problema 9. Fie M o mulţime finită de puncte din plan cu proprietatea că<br />

orice dreaptă care uneşte două puncte ale lui M conţine cel puţin trei puncte din M.<br />

Demonstraţi că toate punctele din M sunt coliniare. (Sylvester)<br />

Soluţie. Calculăm distanţele de la fiecare punct din M la fiecare dreaptă care<br />

...<br />

uneşte două puncte din M. Să presupunem că există astfel de<br />

C<br />

P D<br />

distanţe nenule; cum numărul lor este finit, există o asemenea B<br />

distanţă care este cea mai mică. Notăm această distanţă cu<br />

d şi să zicem că d = dist(A, BC), cu A, B, C ∈ M. Conform Q<br />

.<br />

ipotezei, mai există un punct D ∈ BC. Notăm cu P piciorul<br />

perpendicularei din A pe BC; măcar două dintre punctele B, C<br />

A<br />

şi D sunt de aceeaşi parte a lui P şi să presupunem că acestea<br />

sunt B şi C, cu C ∈ (P B). În aceste condiţii, dist(C, AB) < d(A, BC) = d, fapt care<br />

contrazice minimalitatea lui d. Rămâne că toate punctele din M sunt coliniare.<br />

Propunem spre rezolvare câteva probleme, bazate pe aceeaşi idee.<br />

31<br />

.<br />

.


Problema 10. Şase unghiuri în jurul unui punct au proprietatea că diferenţa<br />

măsurilor oricăror două unghiuri consecutive este de 2 ◦ . Determinaţi măsurile unghiurilor.<br />

(Dragoş Moldoveanu, O.M., etapa locală, Prahova, 2008)<br />

Problema 11. Interiorul unui pătrat este descompus într-un număr finit de<br />

pătrate mai mici, cu ajutorul unor paralele duse la laturile sale. Demonstraţi că<br />

măcar două dintre pătratele descompunerii au laturile de lungimi egale.<br />

Problema 12. Fie suma S = 1 39 + 1 40 + 1 41 + . . . + 1 . Dacă S se scrie ca<br />

50<br />

fracţie ireductibilă sub forma p q , demonstraţi că p. .89, iar q . .2. (O.M., etapa locală,<br />

Iaşi, 2007)<br />

Problema 13. Fie x, y, z trei numere prime distincte. Să se demonstreze că<br />

3(x + y)(y + z)(z + x) < x 2 y 2 z 2 . (Concursul ”Florica T. Câmpan”, 2005)<br />

Problema 14. Fie n ∈ N ∗ şi A = {1, 2, 3, . . . , 2n}. Demonstraţi că oricum am<br />

alege B ⊂ A cu |B| = n + 1, fie putem selecta trei elemente ale lui B cu proprietatea<br />

că unul dintre ele este egal cu suma celorlaltor două, fie putem selecta două elemente<br />

ale lui B, unul fiind dublul celuilalt.<br />

Problema 15. Rezolvaţi în Z ecuaţia x 2 + y 2 + z 2 + w 2 = x 2 y 2 z 2 .<br />

(Dan Radu, G.M.-A,3/2007)<br />

Problema 16. Determinaţi mulţimea numerelor naturale n pentru care există<br />

n cercuri în plan cu interioarele disjuncte, fiecare dintre ele tangent la cel puţin alte<br />

şase cercuri dintre cele n.<br />

(Luis Funar, Concurs R.M.T., ediţia a V-a)<br />

Problema 17. În sistemul solar Câinele Verde sunt 2001 planete. Pe fiecare<br />

dintre aceste planete este câte un astronom care se uită prin telescop la planeta cea<br />

mai apropiată. Dacă distanţele reciproce dintre planete sunt diferite, demonstraţi că<br />

există o planetă la care nu se uită nimeni.<br />

(Daniel Stretcu, E:13684, G.M. 7-8/2008, p.400; [2], 4.3, p.15)<br />

Nu putem încheia fără a mulţumi domnului Mircea Lascu, la insistenţele căruia<br />

am scris această notă.<br />

Bibliografie<br />

1. , http://math.ournet.md/competitiva/extrem/extrem.html<br />

2. N. Agahanov, O. Podlipsky - Olimpiadele matematice ruseşti, GIL, Zalău, 2004.<br />

3. A. Engel - Probleme de matematică-Strategii de rezolvare, GIL, Zalău, 2006.<br />

4. N.N. Hârţan - Matematică pentru clasa a V-a, Moldova, Iaşi, 1995.<br />

5. E.A. Morozova, I.S. Petrakov, V.A. Skvorţov - Olimpiadele internaţionale de<br />

matematică, Ed. Tehnică, Bucureşti, 1978.<br />

32


Profesorul Constantin E. Popa la şaizeci de ani<br />

Anul trecut un bun coleg şi, în acelaşi timp, bun prieten a împlinit vârsta de<br />

şaizeci de ani.<br />

Folosesc acest rotund prilej spre a supune atenţiei momentele semnificative ale<br />

activităţii unui dascăl deosebit, pentru care munca la catedră a însemnat întotdeauna<br />

dăruire, profesionalism şi, lucru destul de rar în zilele noastre, multă modestie.<br />

S-a născut la 13 octombrie 1948 în comuna Avrămeşti din judeţul Vaslui. A urmat<br />

cursurile şcolii primare şi gimnaziale la Puieşti. Absolvent al Liceului Teoretic din<br />

Puieşti, promoţia 1966, urmează Institutul Pedagogic din Iaşi pe care-l absolvă în<br />

1969 şi apoi cursurile Facultăţii de Matematică din cadrul Universităţii ”Alexandru<br />

Ioan Cuza”, tot din Iaşi.<br />

Preocupat constant de continua sa pregătire metodico-ştiinţifică, profesorul<br />

Constantin Popa promovează cu succes toate examenele de grad (definitivare în<br />

1972, gradul didactic II în 1978 şi gradul didactic I în 1983).<br />

Cartea de muncă a dlui profesor îl menţionează ca profesor la şcoala generală din<br />

Rădeşti în anul şcolar 1969-1970. În anul şcolar următor îl găsim la Liceul de Cultură<br />

Generală din Puieşti, iar în perioada 1971-1975 la Şcoala Generală din Lăleşti.<br />

În 1975 îşi satisface stagiul militar şi apoi este angajat profesor titular la Grupul<br />

Şcolar Industrial Bârlad, ca urmare a promovării concursului de ocupare a catedrelor.<br />

După doi ani este transferat la Liceul ”Gheorghe Roşca Codreanu” - actualul Colegiu<br />

Naţional - unde funcţionează cu succes şi în prezent.<br />

În anul şcolar 1990-1991 este numit director adjunct, iar în perioada 1991-1995<br />

funcţionează ca director. Este eliberat din funcţie la cerere, caz oarecum singular<br />

pentru vremurile în care situaţia cel mai des întâlnită este aceea a omului care ţine<br />

la funcţie. În anul 2002 este numit în funcţia de director adjunct cu delegaţie şi apoi<br />

ocupă postul prin concurs până în 2006.<br />

Cariera didactică a profesorului Popa este marcată de rezultate excepţionale.<br />

Clasele cu care a lucrat au avut întotdeauna rezultate deosebite la concursurile şcolare,<br />

la examenele de absolvire (bacalaureat şi capacitate) şi la examenele de admitere la<br />

obiectul matematică.<br />

Pot afirma cu certitudine că cele mai bune rezultate obţinute în ultimii 25 de ani<br />

de către elevii Colegiului Naţional ”Gheorghe Roşca Codreanu” au fost realizate sub<br />

îndrumarea profesorului Constantin Popa. În ultimii zece ani patru dintre foştii săi<br />

elevi, absolvenţi ai facultăţilor de matematică din ţară, au obţinut titlul de doctor în<br />

matematică; este vorba despre Cătălin Trenchea (doctorat obţinut la Universitatea<br />

”Al. I. Cuza” din Iaşi), Cristian Mardare (doctorat obţinut la Sorbona), Constantin<br />

C. Popa (fiul dlui profesor; doctorat obţinut la Institutul Weissman din Israel) şi<br />

Marius Tărnăuceanu (doctorat la Universitatea ”Ovidius” din Constanţa). Şi, tot<br />

ca o dovadă a talentului profesorului Popa de a genera profesionişti în domeniu, să<br />

33


mai spunem că opt dintre elevii unei clase care a absolvit acum câţiva ani sunt astăzi<br />

profesori de matematică. De asemenea, elevul Andrei Carp, absolvent din 2006, actualmente<br />

student la Facultatea de Automatizări din Universitatea Politehnică din<br />

Bucureşti se află abia la începutul drumului: el a participat la Balcaniada de Matematică<br />

a studenţilor. Şi să nu uităm un alt rezultat excepţional: ca elev, Marius<br />

Tărnăuceanu a fost component al lotului olimpic lărgit al României pentru Olimpiada<br />

Internaţională de Matematică.<br />

Fără îndoială că mai există profesori cu asemenea palmares. Eu vreau însă să<br />

subliniez bucuria de a întâlni şi de a lucra cu un om dăruit de Dumnezeu cu mult har<br />

şi cu deosebită inteligenţă, pe care a ştiut să le folosească, ”înmulţind talanţii”.<br />

Nu este deloc lipsit de importanţă să semnalăm şi una din cele mai mari realizări<br />

ale domnului C. Popa ca director: reparaţiile capitale la care a fost supusă clădirea<br />

străveche a şcolii noastre s-au realizat în cea mai mare parte sub conducerea domniei<br />

sale. Sper ca în felul acesta să înlăturăm o mare nedreptate, pentru că la festivităţile<br />

care au avut loc cu ocazia împlinirii a 150 de ani de la înfiinţarea liceului, când dealtfel<br />

s-au finalizat lucrările de consolidare, nici măcar nu i-a fost pomenit numele.<br />

Personal am avut dintotdeauna o relaţie specială cu domnul profesor Popa. Îmi<br />

amintesc cu mare bucurie de discuţiile noastre despre matematică, dar şi despre viaţă.<br />

În anul 1979, proaspăt absolvent al Facultăţii de Matematică, am fost repartizat la<br />

Liceul ”Gheorghe Roşca Codreanu”, ca profesor de Rezistenţa Materialelor şi Organe<br />

de Maşini (!), deşi erau ore de matematică disponibile. Sfătuit de profesorul C. Popa,<br />

am plecat la Ministerul Învăţământului, unde am prezentat situaţia. Ca urmare a<br />

acestui demers şi conform legii, în următorul an şcolar am fost titularizat pe o catedră<br />

de matematică.<br />

Mărturisesc că am apreciat încă de la început calităţile domnului profesor<br />

Constantin Popa şi că, pentru mine, domnia sa a fost mereu un model de seriozitate,<br />

conştiinciozitate şi profesionalism; nu pot decât să regret că nu sunt suficiente<br />

cuvintele pentru a exprima în profunzime acest lucru.<br />

Portretul domnului profesor nu poate fi complet dacă nu evidenţiem armonia din<br />

viaţa sa de familie; poate că multe din cele menţionate mai sus nu s-ar fi putut realiza<br />

fără sprijinul discret şi permanent al soţiei domniei sale, doamna Mariana Popa. S-au<br />

completat atât de firesc de-a lungul timpului şi sunt părinţii a doi copii cu educaţie<br />

aleasă: Constantin Popa - doctor în matematică (aşa cum am mai spus) şi Elena Popa<br />

- medic.<br />

Închei aici - deşi ar mai fi destule de povestit - mai spunând că sunt bucuros şi<br />

fericit că am lucrat şi lucrez la Colegiul Naţional ”Gheorghe Roşca Codreanu” cu<br />

oameni deosebiţi; doar câteva nume, pe lângă cel al profesorului Popa: Bernard Perl,<br />

Constantin Iancu, Petru Sava, Ion Luchian, Răzvan Ionescu, Vasile Ţugulea, Rodica<br />

Popovici, Gabi Ghidoveanu, Marian Tetiva, Dănuţ Mihai, Anişoara Creţu.<br />

Cu mulţumiri colegei Gabi Ghidoveanu pentru adăugiri, limpeziri şi organizarea<br />

materialului.<br />

Dumitru MIHALACHE<br />

Colegiul Naţional ”Gh. Roşca Codreanu”, Bârlad<br />

34


Concursul ”Recreaţii <strong>Matematice</strong>”<br />

Ediţia a VI-a, Muncel (Iaşi), 26 august 2008<br />

Clasa a VI-a<br />

1. Determinaţi ultimele două cifre ale numărului A = 7·19·31·...·1999·2011.<br />

(Numărul A reprezintă produsul tuturor numerelor naturale mai mici decât 2012 care<br />

dau restul 7 la împărţirea cu 12.)<br />

2. Se consideră şirul de numere naturale 1, 1, 2, 5, 12, 27, 58, ... Calculaţi suma<br />

primilor 100 de termeni ai şirului.<br />

3. Pe o masă sunt mai multe bomboane, iar în jurul mesei sunt aşezaţi mai mulţi<br />

elevi. Primul elev ia de pe masă 1<br />

15 din numărul de bomboane; al doilea ia 1 15 din<br />

numărul bomboanelor rămase şi încă 1 15<br />

din numărul de bomboane luate de primul<br />

elev. Al treilea elev ia 1<br />

1<br />

15<br />

din numărul de bomboane rămase şi încă<br />

15<br />

din numărul<br />

de bomboane luate de primul şi al doilea elev împreună etc. Procedeul continuă până<br />

când ultimul elev reuşeşte să ia ultimele bomboane de pe masă, după regula de mai<br />

sus. Aflaţi numărul de elevi.<br />

Clasa a VII-a<br />

1. Determinaţi p∈N pentru care numerele p, p + 12, p + 22, p + 52, p + 72, p + 102<br />

şi p + 132 sunt prime.<br />

2. Alegeţi în faţa fiecăruia dintre numerele 1, 2, 3, ..., 2009 unul dintre semnele<br />

+ sau – astfel încât numărul A = |±1 ± 2 ± ... ± 2009| să ia cea mai mică valoare<br />

posibilă.<br />

3. Fie unghiul ∢AOB având măsura de 120 ◦ , iar P un punct pe bisectoarea sa<br />

astfel încât OP = OA + OB. Să se arate că triunghiul PAB este echilateral.<br />

Clasa a VIII-a<br />

1. Rezolvaţi în mulţimea numerelor întregi ecuaţia x 4 + 4y 4 = 2z 4 + 8u 4 .<br />

2. Determinaţi numerele întregi a, b, c, d pentru care ac + bd = 1 iar ad + bc = 2.<br />

3. Fie triunghiul ABC cu m(∢A) = 90 o şi D ∈ (BC ) astfel încât m(∢CAD) = 30 o<br />

şi CD = AB = 1. Să se calculeze lungimea segmentului [BC ], ştiind că aceasta este<br />

un număr natural nenul.<br />

Clasa a IX-a<br />

1. Pentru n ∈ N*, considerăm A = {1 2 , 2 2 , 3 2 , . . . , n 2 }. Determinaţi n, ştiind că<br />

există o funcţie f : A → A astfel încât f (x) − f (y) = √ x − √ y, ∀x, y ∈ A.<br />

2. Arătaţi că x + y + z − xy − yz − zx ≤ 1, ∀x, y, z ∈ [0; 1] .<br />

3. Fie M ∈ Int(∆ABC ). Dacă P este perimetrul △ABC, demonstraţi că<br />

P<br />

2<br />

< MA + <strong>MB</strong> + MC < P.<br />

35


Clasa a X-a<br />

1. Să se rezolve în N ecuaţia<br />

n −hn<br />

2i−hn<br />

3i+hn<br />

12i+hn<br />

18i=11 ·hn<br />

36i.<br />

(s-a notat cu [x] partea întreagă a numărului real x).<br />

2. Fie punctul M interior triunghiului ABC şi fie AM ∩ BC = {A ′ }, BM ∩ AC =<br />

{B ′ }, CM ∩ AB = {C ′ }. Se notează cu S 1 , S 2 , S 3 , S 4 , S 5 şi S 6 ariile triunghiurilor<br />

MA ′ B, MA ′ C, <strong>MB</strong> ′ C, <strong>MB</strong> ′ A, MC ′ A, respectiv, MC ′ B. Să se demonstreze că<br />

S 1<br />

S 2<br />

+ S 3<br />

S 4<br />

+ S 5<br />

S 6<br />

= 3<br />

dacă şi numai dacă M este centrul de greutate al triunghiului ABC .<br />

3. Fie f : R → R o funcţie surjectivă şi strict crescătoare. Să se determine funcţiile<br />

g : R → R pentru care g (f (x)) ≤ x ≤ f (g (x)) , ∀x ∈ R. Apoi, să se determine g dacă<br />

f (x) = x 3 + 3x 2 + 3x, x ∈ R.<br />

Clasa a XI-a<br />

1. Să se rezolve în mulţimea R ecuaţia |3 −x − 4| −x 3 − 3=a x 3 − 3 −x + 1,<br />

unde a > 1, fixat.<br />

2. Fie n puncte pe un cerc, cu proprietatea că oricare două coarde cu extremităţile<br />

în cele n puncte nu sunt paralele şi oricare trei nu sunt concurente într-un punct situat<br />

în interiorul cercului. Să se determine<br />

nX<br />

numărul de regiuni din interiorul cercului<br />

delimitate de laturile şi diagonalele poligonului convex care are ca vârfuri cele n<br />

puncte date, n ≥ 4.<br />

1<br />

3. Să se calculeze suma S (n) =<br />

2 k Ck n+k.<br />

k=0<br />

Clasa a XII-a<br />

1. Fie şirul (x n ) n≥1<br />

, unde x 1 > 1, x n+1 = x n − 1<br />

, ∀n ∈ N*. Calculaţi lim<br />

ln x n<br />

n→∞ xn n.<br />

2. Cercetaţi dacă există A, B ∈ M n (Z) astfel încât det(A + 2005B) = 0 şi<br />

det(A + 2007B) = 2009.<br />

3. Se consideră o funcţie f : R → R astfel încât:<br />

(i) funcţia f are limite laterale în orice punct a ∈ R şi<br />

f (a − 0) ≤ f (a) ≤ f (a + 0) ;<br />

(ii) pentru orice a, b ∈ R, a < b, avem f (a − 0) < f (b − 0).<br />

Să se arate că f este strict crescătoare.<br />

36


Concursul omagial ”Recreaţii Ştiinţifice”<br />

Rezultatul concursului<br />

Amintim punctajul pe baza căruia s-au acordat premiile:<br />

- fiecare problemă este notată maxim cu 10 puncte;<br />

- se acordă câte 2 puncte în plus pentru alte soluţii, generalizări etc.;<br />

- se depunctează soluţiile incomplete sau redactate neîngrijit.<br />

A treia problemă a concursului a apărut enunţată în nr. 2/2008, p.182, cu o<br />

greşeală de tipărire în formula de stabilit; formula corectă este<br />

tgÔ IOI ′ 2 |sin B − sin C|<br />

= ,<br />

2 cos A − 1<br />

cum este dată în Recreaţii Ştiinţifice, t.VI(1888), p.96, şi nu cu numitorul 2 cos B − 1,<br />

cum apare în locul menţionat.<br />

Ca urmare, această problemă a fost retrasă, în concurs rămânând numai patru<br />

probleme. Ne cerem scuze pentru acest incident regretabil.<br />

Doi concurenţi au îndeplinit condiţiile de premiere:<br />

- CĂPĂŢÂNĂ Roxana, cl. a IX-a, Colegiul Naţional din Iaşi;<br />

- TIBA Marius, cl. a X-a, Liceul Internaţional de In<strong>format</strong>ică, Bucureşti,<br />

amândoi cu acelaşi punctaj: 38 puncte din 40 posibile.<br />

Comisia de concurs a hotărât ca, în această situaţie de egalitate a punctajului<br />

obţinut, să acorde premiul I acestor concurenţi (iniţial era prevăzut un singur premiu<br />

I). Premiul I constă dintr-o diplomă şi suma de 200 lei.<br />

Rezolvarea problemelor<br />

1. Ion şi Constantin merg la cumpărături cu soţiile lor, Maria şi Elena. Fiecare<br />

din aceste patru persoane cumpără un număr de obiecte ce le plăteşte pe fiecare cu<br />

atâţia lei câte obiecte a cumpărat. Ion cumpără nouă obiecte mai mult decât Elena<br />

şi fiecare soţ cheltuieşte cu 21 lei mai mult decât soţia sa. Care este soţia lui Ion şi<br />

care este a lui Constantin Care este numărul de obiecte cumpărate de fiecare dintre<br />

aceste persoane Care este suma cheltuită de fiecare dintre ele<br />

Recreaţii Ştiinţifice, I(1883)-Problema 50, p.279<br />

Soluţie (ibidem, p.279). Notăm cu x numărul obiectelor cumpărate de un bărbat<br />

şi cu y cel cumpărat de soţia lui. Din enunţul problemei rezultă că<br />

x 2 − y 2 = 21 sau (x + y)(x − y) = 21,<br />

ecuaţie echivalentă cu următoarele două sisteme:<br />

¨x + y = 7<br />

¨x + y = 21<br />

şi .<br />

x − y = 3 x − y = 1<br />

Primul sistem are soluţia x = 5, y = 2, iar al doilea x = 11, y = 10. Aşadar, un bărbat<br />

a cumpărat 5 obiecte şi celălalt 11 obiecte, iar o femeie a cumpărat 2 şi cealaltă 10.<br />

Cum, din enunţ, Ion cumpără 9 obiecte mai mult decât Elena, rezultă că Ion<br />

cumpără 11 obiecte, Elena 2 obiecte, Constantin 5 şi Maria 10.<br />

Ca urmare, Ion a cheltuit 11 2 = 121 lei, Constantin 5 2 = 25 lei, Elena 2 2 = 4 lei<br />

şi Maria 10 2 = 100 lei.<br />

37


În sfârşit, din condiţiile problemei deducem că Maria este soţia lui Ion şi Elena a<br />

lui Constantin.<br />

2. Să se rezolve sistemul de ecuaţii<br />

(x + 2y) (x + 2z) = a,<br />

(y + 2x) (y + 2z) = b,<br />

(z + 2x) (z + 2y) = c (0 < a < b < c) .<br />

8>:t − (y − z) 2 = a<br />

t − (z − x) 2 = b<br />

t − (x − y) 2 = c<br />

Recreaţii ştiinţifice, IV(1886)-Problema 209, p.144<br />

Soluţie (Recreaţii Ştiinţifice, t.V, p.18 – schiţă de soluţie). Utilizând identitatea<br />

+ n2<br />

− n2<br />

mn =m<br />

−m<br />

şi notând t = (x+y+z) 2 , putem scrie sistemul în forma<br />

2 2<br />

8>:|y − z| = √ t − a<br />

sau |z − x| = √ t − b<br />

|x − y| = √ t − c,<br />

cu t > c (t = c ar implica a = b). Explicitarea valorilor absolute conduce la opt cazuri<br />

(+ inseamnă<br />

II8<<br />

că expresia ”dintre bare” rămâne neschimbată, iar − spune că aceasta<br />

se ia cu semn schimbat):<br />

:+ :+<br />

+ , + , III8<<br />

IV8< V8< I8< II8< :+ :+ :− :− :−<br />

− , − , + , V + , V − , V III8< :−<br />

−<br />

+ − + − + −<br />

+<br />

−<br />

Cazurile I şi VIII. Adunând<br />

√<br />

cele √ trei ecuaţii,<br />

√<br />

suntem conduşi la ecuaţia în t<br />

t − a + t − b + t − c = 0, t > c,<br />

I8<<br />

.<br />

care nu are soluţii pe mulţimea t > c. Ca urmare, sistemele I şi VIII nu au soluţii.<br />

Cazurile II şi VII. În acelaşi mod, obţinem ecuaţia<br />

√ √ √<br />

t − a + t − b = t − c, t > c,<br />

care nu are soluţii, căci, din 0 < a < b < c, avem √ t − c < √ t − a.<br />

Cazurile III şi VI. Se obţine ecuaţia<br />

√<br />

t − a +<br />

√<br />

t − c =<br />

√<br />

t − b, t > c,<br />

şi, ca în cazul precedent, se consată că nu avem soluţii.<br />

Cazurile IV şi V. Obţinem, pe mulţimea t > c, ecuaţia<br />

√<br />

t − b +<br />

√<br />

t − c =<br />

√<br />

t − a sau 2<br />

√<br />

t − b −<br />

√<br />

t − c = b + c − a − t,<br />

care, pentru t verificând condiţiile c < t < b + c − a, este echivalentă cu<br />

4(t − b)(t − c) = t 2 − 2(b + c − a)t + (b + c − a) 2<br />

sau<br />

3t 2 − 2(a + b + c)t + 4bc − (b + c − a) 2 = 0<br />

(menţionăm că nu putem avea t = b + c − a fără a fi în contradicţie cu 0 < a < b < c).<br />

Această ecuaţie de gradul al doilea are rădăcinile<br />

38


t ± = 1 3 [(a + b + c) ± 2√ a 2 + b 2 + c 2 − ab − bc − ca].<br />

Vom vedea că numai t + verifică condiţiile c < t < b + c − a. Într-adevăr, avem<br />

t ± > c ⇔ ±2pa 2 + b 2 + c 2 − ab − bc − ca > 2c − a − b<br />

şi, cum 2c − a − b > 0, urmează că rădăcina t − trebuie exclusă. În privinţa lui t +,<br />

ridicând la pătrat şi operând reduceri, obţinem a 2 + b 2 > 2ab, care este adevărată,<br />

căci a < b. Deci t + > c şi rămâne de văzut că are loc şi inegalitatea t + < b + c − a:<br />

t + < b+c−a ⇔ √ a 2 + b 2 + c 2 − ab − bc − ca < b+c−2a ⇔ . . . ⇔ (b−a)(c−a) > 0,<br />

ceea ce este adevărat.<br />

V8><<br />

Să revenim la rezolvarea sistemelor IV şi V, adică<br />

:y − z = √ t − a<br />

z − x = − √ :y − z = − √ t − a<br />

t − b<br />

x − y = − √ şi z − x = √ t − b<br />

t − c<br />

x − y = √ t − c,<br />

IV8>< ><br />

cu 3t = (a + b + c) + 2 √ a 2 + b 2 + c 2 − ab − bc − ca.<br />

Mai întâi, amintindu-ne că am notat (x+y +z) 2 = t, deducem că x+y +z = ± √ t.<br />

Pentru a rezolva sistemul IV procedăm astfel: aflăm x înlocuind în această ultimă<br />

egalitate pe y şi z daţi de ultimele două ecuaţii ale sistemului IV, iar y şi z se găsesc<br />

în mod similar; obţinem următoarele două soluţii:<br />

3x = ± √ t+ √ t − b− √ t − c, 3y = ± √ t+ √ t − a+ √ t − c, 3z = ± √ t− √ t − a− √ t − b.<br />

Analog, sistemul V are soluţiile:<br />

3x = ± √ t− √ t − b+ √ t − c, 3y = ± √ t− √ t − a− √ t − c, 3z = ± √ t+ √ t − a+ √ t − b,<br />

cu 3t = (a + b + c) + 2 √ a 2 + b 2 + c 2 − ab − bc − ca.<br />

În concluzie, sistemul dat are patru soluţii, două câte două opuse (adică, dacă<br />

(x 0 , y 0 , z 0 ) este soluţie, atunci şi (−x 0 , −y 0 , −z 0 ) este soluţie a sistemului).<br />

3. Fie O, I, I ′ centrele cercului circumscris triunghiului ABC, cercului înscris<br />

acestuia şi, respectiv, al cercului exînscris tangent laturii BC. Să se demonstreze că<br />

tgÔIOI ′ 2 |sin B − sin C|<br />

= .<br />

2 cos A − 1<br />

Recreaţii Ştiinţifice, VI(1888)-Problema 285, p.96<br />

Soluţie (ibidem, p.273). Dacă O se află pe bisectoarea unghiuluibA, atunci △ABC<br />

este isoscel (cu vârful în A). Excludem acest caz banal. De asemenea, cu teorema lui<br />

Pitagora şi calcule de rutină, avem: m(ÔIOI ′ ) = 90 ◦ ⇔ m(bA) = 60 ◦ ⇔ 2 cos A − 1 = 0<br />

şi formula are loc. Apoi, dacăÒC >ÒB, atunci O se află în semiplanul determinat de<br />

><br />

dreapta II ′ care nu conţine vârful C (chiar dacă bA sauÒC<br />

este obtuz).<br />

În △OII ′ , cu teorema cosinusului, avem:<br />

cosÔ IOI ′ = OI2 + OI ′2 − II ′2<br />

2OI · OI ′ .<br />

Pe de altă parte, cu teorema sinusurilor aplicată △OII ′ şi<br />

△OI ′ A, avem:<br />

sinÔ IOI ′ = II′ · sinÔOI ′ I<br />

OI<br />

= R · II′ · sinÔOAI<br />

OI · OI ′ .<br />

39<br />

B<br />

O<br />

. .<br />

A<br />

I<br />

I<br />

C


Ca urmare, obţinem că<br />

(1) tgÔIOI ′ = 2R · II′ · sinÔOAI<br />

OI 2 + OI ′2 − II ′2 .<br />

În △AOI şi △AOI ′ avem relaţiile: OI 2 = R 2 + AI 2 − 2R · AI · cosÔOAI şi OI ′2 =<br />

R 2 + AI ′2 − 2R · AI ′ · cosÔOAI, care, înlocuite în (1), conduc la<br />

(2) tgÔIOI ′ =<br />

R · II ′ · sinÔOAI<br />

R 2 + AI · AI ′ − R(AI + AI ′ ) cosÔOAI .<br />

Pentru AI, AI ′ şi II ′ se cunosc sau se stabilesc uşor formulele următoare:<br />

(3) AI = p − a<br />

cos A , AI ′ = p<br />

2<br />

cos A , II ′ = a<br />

2<br />

cos A .<br />

2<br />

Să mai observăm că, dacăÒC >ÒB, vom avea<br />

m(ÔOAI) = A 2 − m(ÕOAB) = A 2 − 1 C − B<br />

(π − 2C) = ,<br />

2 2<br />

iar, dacăÒB >ÒC, vom obţine m(ÔOAI) = B − C . Aşadar, în ambele cazuri, avem<br />

2<br />

(4) m(ÔOAI) =<br />

|B − C|<br />

.<br />

2<br />

Nu mai rămâne decât să înlocuim în (2) expresiile mărimilor ce intervin, date de<br />

(3) şi (4), şi să facem calcule de rutină pentru a ajunge la rezultatul dorit.<br />

Notă. Această problemă a fost rezolvată de Vasile Cristescu, viitorul fondator şi<br />

unul dintre cei patru ”stâlpi” ai Gazetei <strong>Matematice</strong>.<br />

4. Să se taie o sferă cu un plan astfel încât diferenţa volumelor conurilor drepte<br />

ce au ca baze secţiunea planului cu sfera şi vârfurile pe sferă să fie maximă.<br />

Recreaţii Ştiinţifice, IV(1886)-Problema 227, p.288<br />

Soluţie. (Recreaţii Ştiinţifice, t. V, pp.89, 113, 277). Să notăm cu x raza bazei<br />

conurilor şi cu y distanţa centrului sferei la planul de secţiune. A<br />

Înălţimile conurilor fiind R + y şi R − y, unde R notează raza<br />

C x E<br />

sferei, diferenţa V a a volumelor conurilor este dată de formula<br />

D<br />

y<br />

V = 1 3 πx2 (R + y) − 1 3 πx2 (R − y) = 2 3 πx2 y.<br />

Având în vedere că x 2 = (R + y)(R − y) (teorema înălţimii),<br />

obţinem că<br />

V = 2 3 πy(R2 − y 2 ).<br />

Valoarea y pentru care V este maxim este aceeaşi cu cea pentru care este maxim<br />

produsul y(R 2 −y 2 ) sau pătratul acestuia y 2 (R 2 −y 2 ) 2 . Cum suma factorilor ultimului<br />

40<br />

.<br />

B<br />

O<br />

R


produs este y 2 + (R 2 − y 2 ) = R 2 =constant, rezultă că el va fi maxim pentru valorile<br />

y ce satisfac condiţia<br />

y 2<br />

1 = R2 − y 2<br />

⇔ 3y 2 = R 2 ,<br />

2<br />

adică y = R√ 3<br />

3 . Obţinem că V are valoarea maximă egală cu 4√ 3<br />

27 πR3 .<br />

Secţiunea cu un plan perpendicular pe un diametru AB în punctul E al acestuia,<br />

cu OE = R√ 3<br />

, va satisface cerinţele problemei.<br />

2<br />

Notă. Vasile Cristescu, elev al Licelui Naţional din Iaşi, dă două soluţii acestei<br />

probleme.<br />

5. Fie M un punct exterior cercului C de centru O şi raza R. Notăm cu T 1 , T 2<br />

punctele de contact ale tangentelor duse din M la C şi cu A punctul de intersecţie a<br />

dreptei OM cu cercul C care verifică condiţia A /∈ [OM]. Să se determine mulţimea<br />

punctelor M pentru care se poate construi un triunghi cu segmentele [MT 1 ], [MT 2 ]<br />

şi [MO], dar nu se poate construi un triunghi cu [MT 1 ], [MT 2 ] şi [MA].<br />

Recreaţii <strong>Matematice</strong>, 1/2009-Problema L156, p.78<br />

Soluţie. Notăm cu d distanţa între M şi O şi cu t lungimile tangentelor din M.<br />

Condiţia necesară şi suficientă de existenţă a unui triunghi<br />

M<br />

având laturile [MT 1 ], [MT 2 ] şi [MO] este d < 2t, iar cea de<br />

non-existenţă a unui triunghi cu laturile [MT 1 ], [MT 2 ] şi [MA]<br />

T<br />

este d + R ≥ 2t. Mulţimea X căutată este<br />

1<br />

X = {M|d < 2t} ∩ {M|d + R ≥ 2t}<br />

= {M|d < 2 √ d 2 − R 2 } ∩ {M; d + R ≥ 2 √ d 2 − R 2 }<br />

= {M|3d 2 > 4R 2 } ∩ {M|3d 2 − 2Rd − 5R 2 ≤ 0}<br />

= {M|d > 2√ 3<br />

3 R} ∩ {M|3(d + R)(d − 5 R) ≤ 0}<br />

3<br />

= {M| 2√ 3<br />

R<br />

< d ≤ 5 3 R},<br />

adică X este coroana de centru O şi raze 2√ 3<br />

3<br />

şi pe cel mic.<br />

O<br />

A T 2<br />

şi 5 R incluzând cercul mare, dar nu<br />

3<br />

9 × 9 + 7 = 88<br />

98 × 9 + 6 = 888<br />

987 × 9 + 5 = 8888<br />

9876 × 9 + 4 = 88888<br />

98765 × 9 + 3 = 888888<br />

987654 × 9 + 2 = 8888888<br />

9876543 × 9 + 1 = 88888888<br />

98765432 × 9 + 0 = 888888888<br />

41


Soluţiile problemelor propuse în nr. 1 / 2008<br />

Clasele primare<br />

P.144. Elevii clasei I intră în clasă în rând câte unul. Câţi elevi sunt în clasă,<br />

dacă Matei este al 12-lea când se numără începând din faţă şi al 16-lea când se<br />

numără începând din spate<br />

(Clasa I )<br />

Inv. Elena Porfir, Iaşi<br />

Soluţie. (12 − 1) + 16 = 11 + 16 = 27.<br />

P.145. Un fluture zboară din floarea 1 în floarea 3, apoi din aceasta în floarea 5<br />

şi aşa mai departe (figura 1). După câte zboruri ajunge în<br />

floarea de pe care a plecat<br />

(Clasa I )<br />

Evelina Zaporojanu, elevă, Iaşi<br />

Soluţie. Formăm şirul de numere: 1 3 5 7 9 2 4 6 8 1.<br />

Fluturele ajunge în floarea de pe care a plecat după 9 zboruri.<br />

P.146. După ce fratele meu mi-a dat un sfert din merele<br />

sale, le-am amestecat cu cele 6 ale mele şi le-am aşezat pe două<br />

farfurii, cu câte 5 mere fiecare. Câte mere avea fratele meu<br />

(Clasa a II-a)<br />

Inst. Elena Nuţă, Iaşi<br />

Soluţie. Pe cele două farfurii au fost aşezate 5 + 5 = 10 (mere). Un sfert din<br />

merele fratelui înseamnă 10 − 6 = 4 (mere). Fratele avea 4 + 4 + 4 + 4 = 16 (mere).<br />

P.147. Dacă numerele ar fi puse corect în cele trei cercuri,<br />

atunci am avea aceeaşi sumă a celor aflate în fiecare dintre cercuri<br />

(figura 2). În câte moduri pot fi aşezate corect aceste numere<br />

(Clasa a II-a)<br />

Cătălina Istrate, elevă, Iaşi<br />

Soluţie. 1+2+3+4+5+6+7+8+9 = 45; 45 = 15+15+15.<br />

Avem cazurile:<br />

1) 1+9+5=15; 2+6+7=15; 3+4+8=15;<br />

2) 1+8+6=15; 2+4+9=15; 3+5+7=15.<br />

P.148. Aflaţi valoarea a ştiind că 100−99 : 99−98 : 98−97 : 97−· · ·−a : a = 11.<br />

(Clasa a III-a)<br />

Mariana Nastasia, elevă, Iaşi<br />

Soluţie. Fiecare împărţire are rezultatul 1; 100 − b = 11, b = 100 − 11, b = 89. Pe<br />

1 trebuie să-l scădem de 89 ori. De la a la 99, trebuie să avem 89 numere consecutive;<br />

99 − a + 1 = 89, 99 − a = 88, a = 99 − 88 = 11.<br />

P.149. Irina îi spune Mioarei:<br />

– Dă-mi 2 lei ca să am şi eu cât tine!<br />

Mioara îi răspunde:<br />

– Dă-mi tu 2 lei, să am o sumă de 2 ori mai mare decât suma ce-ţi rămâne ţie!<br />

Ce sumă a avut la început fiecare fată<br />

(Clasa a III-a)<br />

Inst. Maria Racu, Iaşi<br />

42


Soluţie. Mioara are cu 2 + 2 = 4 (lei) mai mult ca Irina. Suma Irinei, micşorată<br />

cu 2 lei, este 2 + 2 + 2 + 2 = 8 (lei). La început Irina a avut 8 lei +2 lei = 10 lei, iar<br />

Mioara a avut 10 lei +4 lei = 14 lei.<br />

P.150. Romanul Harry Potter are 7 volume. Ştiind că fiecare volum, începând cu<br />

al doilea, are cu 144 pagini mai puţin decât dublul numărului de pagini al volumului<br />

precedent, iar al treilea volum are 176 de pagini, aflaţi câte pagini are întregul roman.<br />

(Clasa a III-a)<br />

Robert Vicol, elev, Iaşi<br />

Soluţie. Volumul al doilea are (176 + 144) : 2 = 160 (pagini), iar primul volum<br />

are (160 + 144) : 2 = 152 pagini. Volumele IV, V, V I, V II au, respectiv, 208 pagini,<br />

272 pagini, 400 pagini, 656 pagini. Însumând, obţinem că întregul roman are 2024 de<br />

pagini.<br />

P.151. Descoperiţi regula de formare a şirului 1, 3, 6, 10, 15, 21, . . . şi scrieţi<br />

numărul de pe locul 2008.<br />

(Clasa a IV-a)<br />

Petru Asaftei, Iaşi<br />

Soluţie. 1 = 1; 3 = 1 + 2; 6 = 1 + 2 + 3; 10 = 1 + 2 + 3 + 4; . . .<br />

Termenul de pe locul 2008 este 1 + 2 + 3 + . . . + 2008 = 2017036.<br />

P.152. În şirul de pătrate egale,<br />

. . . , fiecare<br />

pătrat este împărţit în pătrate mai mici, după o anumită regulă.<br />

a) Arătaţi că nu există în acest şir un pătrat împărţit în 23 pătrate mai mici;<br />

b) Arătaţi că există în şir un pătrat împărţit în 2008 pătrate mai mici.<br />

(Clasa a IV-a)<br />

Ana Tăbăcaru, elevă, Iaşi<br />

Soluţie. Primul pătrat are 3 × 1 + 1 pătrate mai mici, al doilea are 3 × 2 + 1<br />

pătrate mai mici, al treilea are 3 × 3 + 1 pătrate mai mici etc.<br />

a) 23 = 3 × 7 + 2, deci nu există un pătrat în şir cu 23 pătrate mici;<br />

b) 2008 = 3 × 669 + 1; pătratul de pe locul 669 este împărţit în 2008 pătrate mai<br />

mici.<br />

P.153. O veveriţă transportă nişte alune la scorbura sa în 6 ore, iar alta face<br />

acelaşi lucru în 3 ore. În câte ore cele două veveriţe ar transporta alunele împreună<br />

(Clasa a IV-a)<br />

Alexandru-Dumitru Chiriac, elev, Iaşi<br />

Soluţie. Dacă prima veveriţă transportă, în 6 ore, x alune, a doua va transporta,<br />

în 6 ore, 2x alune. Împreună, ele vor transporta, în 6 ore, 3x alune, deci în 2 ore vor<br />

transporta x alune.<br />

Clasa a V-a<br />

V.88. O secvenţă de numere este <strong>format</strong>ă din multipli consecutivi ai lui 4, astfel<br />

încât suma dintre primul şi ultimul număr este 280, iar suma ultimelor două numere<br />

este 508. Arătaţi că media aritmetică a tuturor numerelor este termen al secvenţei<br />

considerate.<br />

Mirela Marin, Iaşi<br />

Soluţie. Dacă a 1 , a 2 , . . . , a n sunt numerele date, atunci a n−1 + a n = 508 şi, cum<br />

a n = a n−1 +4, deducem că a n = 256. Apoi, a 1 = 280−a n = 24, iar n = a n − a 1<br />

+1 =<br />

4<br />

43


59. Media aritmetică a numerelor va fi 140 şi va fi termen al secvenţei, deoarece 140 . .4,<br />

iar 24 ≤ 140 ≤ 256.<br />

V.89. Determinaţi cifrele x, y, z pentru care xy 2 + xz 2 = 168x.<br />

Ioan Săcăleanu, Hârlău<br />

Soluţie. Dacă x ≥ 3, atunci xy 2 + xz 2 ≥ 30 2 + 30 2 = 1800 > 168x. Dacă x = 1,<br />

atunci xy 2 + xz 2 ≤ 19 2 + 19 2 = 722 < 168x. Rămâne că x = 2 şi, datorită simetriei,<br />

putem presupune că y ≤ z. Dacă z < 9, atunci 2y 2 + 2z 2 ≤ 28 2 + 28 2 = 1568 < 1682.<br />

Deducem că z = 9 şi, în acest caz, 2y 2 +29 2 = 1682 ⇔ 2y 2 = 841 ⇔ 2y = 29 ⇔ y = 9.<br />

În concluzie, (x, y, z) = (2, 9, 9).<br />

V.90. Fie E(n) = 3 n + 5 n , n ∈ N. Aflaţi ultimele două cifre ale numerelor E(10)<br />

şi E(2008).<br />

Mihaela Bucătaru, Iaşi<br />

Soluţie. Observăm că 3 8 = 3 4 ·3 4 = 81·81 = . . . 61, prin urmare 3 10 = . . . 61·9 =<br />

. . . 49. Apoi, 5 10 = . . . 25, deci E(10) = . . . 74.<br />

Avem că 3 20 = . . . 49 · . . . 49 = . . . 01, de unde 3 2000 = (3 20 ) 100 = . . . 01 şi astfel<br />

3 2008 = 3 2000 · 3 8 = . . . 01 · . . . 61 = . . . 61. În concluzie, E(2008) = . . . 61 + . . . 25 =<br />

. . . 86.<br />

V.91. Să se arate că 61 n , n ∈ N ∗ , se poate scrie atât ca sumă, cât şi ca diferenţă<br />

de două pătrate perfecte nenule.<br />

Alexandru Negrescu, student, Iaşi<br />

Soluţie. Dacă n = 2k + 1, k ∈ N, atunci:<br />

61 n = 61 2k · 61 = 61 2k · (5 2 + 6 2 ) = (61 k · 5) 2 + (61 k · 6) 2 ;<br />

61 n = 61 2k · 61 = 61 2k · (31 2 − 30 2 ) = (61 k · 31) 2 − (61 k · 30) 2 .<br />

Dacă n = 2k + 2, k ∈ N, atunci:<br />

61 n = 61 2k · 3721 = 61 k (60 2 + 11 2 ) = (61 k · 60) 2 + (61 k · 11) 2 ;<br />

61 n = 61 2k · 3721 = 61 2k (1861 2 − 1860 2 ) = (61 k · 1861) 2 − (61 k · 1860) 2 .<br />

V.92. Demonstraţi că 22<br />

1 + 32<br />

2 + 42<br />

3 + · · · + 172<br />

16 > 171. Petru Asaftei, Iaşi<br />

n 2<br />

Soluţie. Observăm că<br />

n − 1 = n2 − n + n − 1 + 1<br />

= n + 1 + 1 , prin urmare<br />

n − 1<br />

n − 1<br />

2 2<br />

1 + 32<br />

2 + 42<br />

3 + . . . + 172<br />

16 = 4 + 4 + 1 5 +<br />

2‹+<br />

3‹+. 1 . . + 18 +<br />

16‹=169 1 + 1 2 +<br />

1<br />

3 + 1 4‹+ 1 5 + . . . + 8‹+ 1 1 9 + . . . + 16‹> 1 169+ 1 2 + 1 4 + 1 4‹+ 1 8 + . . . + 1 8‹+<br />

1<br />

16 + . . . + 16‹=169 1 + 1 2 + 1 2 + 1 2 + 1 2 = 171.<br />

V.93. Fie A = {2, 3, 4, . . . , 50, 52, 53, . . . , 100}. Folosind fiecare element al lui<br />

A câte o singură dată, fie ca numărător, fie ca numitor, se scriu 49 de fracţii.<br />

Demonstraţi că măcar una dintre aceste fracţii este reductibilă.<br />

Gabriel Popa, Iaşi<br />

44


Soluţie. Mulţimea A conţine 50 de elemente pare; conform principului cutiei,<br />

dintre cele 49 de fracţii, măcar una va avea şi numărătorul şi numitorul pare, deci se<br />

va simplifica prin 2.<br />

V.94. Fie A mulţimea acelor numere naturale cel mult egale cu 2008, care se divid<br />

cu 2, dar nu se divid cu 6. Dacă scriem elementele lui A în ordine descrescătoare,<br />

care este al 322-lea număr<br />

Enache Pătraşcu, Focşani<br />

Soluţie. Scriem multiplii lui 2 în ordine descrescătoare, grupându-i câte trei:<br />

(2008, 2006, 2004), (2002, 2000, 1998),. . .. Multiplii lui 6 apar în fiecare grupă pe al<br />

treilea loc; numărul căutat se află în a 161-a grupă, pe a doua poziţie şi acesta va fi<br />

1046.<br />

Clasa a VI-a<br />

VI.88. Fie a, b, c, d numere raţionale pozitive astfel încât a b = c d şi a + 1<br />

b + 1 =<br />

c + 1<br />

a + x<br />

. Să se arate că<br />

d + 1 b + x = c + x<br />

d + x , ∀x ∈ Q +.<br />

Claudiu-Ştefan Popa, Iaşi<br />

Soluţie. Toţi numitorii care apar sunt strict pozitivi, deci nenuli. Din ipoteză<br />

obţinem că ad = bc şi (a + 1)(d + 1) = (b + 1)(c + 1), prin urmare a + d = b + c. Însă<br />

a + x<br />

b + x = c + x<br />

d + x ⇔ ad + x(a + d) + x2 = bc + x(b + c) + x 2<br />

şi cum ultima egalitate este adevărată, rezultă concluzia.<br />

VI.89. Arătaţi că numărul N = 1 2007 + 2 2007 + · · · + 2008 2007 se divide cu 2009.<br />

Cătălin Calistru, Iaşi<br />

Soluţie. Dacă n este impar şi a, b ∈ N, se ştie că a n + b n = M(a + b). Grupând<br />

câte doi termenii sumei şi observand că 1 2007 + 2008 2007 = M2009, 2 2007 + 2007 2007 =<br />

M2009, . . . , 1004 2007 + 1005 2007 = M2009, obţinem concluzia problemei.<br />

VI.90. Să se determine numerele naturale cu proprietatea că atât ele cât şi<br />

răsturnatele lor se scriu ca produs de doi factori primi, fiecare factor având două<br />

cifre şi fiind răsturnatul celuilalt.<br />

Temistocle Bîrsan, Iaşi<br />

Soluţie. Să determinăm numerele N care îndeplinesc condiţiile: 1. N = p · q cu<br />

p, q prime, 2. p = ab şi q = ba şi 3. N, răsturnatul lui N, verifică 1 şi 2.<br />

Din aceste ipoteze rezultă că a şi b nu pot fi pare şi nici 5, adică a, b ∈ {1, 3, 7, 9}.<br />

I. p = q, ceea ce este echivalent cu a = b. Avem p ∈ {11, 33, 77, 99} şi numai<br />

p = 11 este număr prim. Obţinem o soluţie a problemei: N = 121 = 11 · 11.<br />

II. p ≠ q şi putem presupune a < b. Avem p ∈ {13, 17, 19, 37, 39, 79} şi cum<br />

p = 19 conduce la q = 91 care este compus, iar p = 39 este compus, urmează că<br />

p ∈ {13, 17, 37, 79}. Vom arăta că nu avem soluţii ale problemei pentru niciuna dintre<br />

aceste patru valori ale lui p. Într-adevăr, dacă p = 13, avem N = 13 · 31 = 403<br />

şi N = 304 = 16 · 19 nu verifică 1. Dacă p = 17, atunci N = 17 · 71 = 1207 şi<br />

N = 7031 = 7 · 17.59 nu convine. Pentru p = 37, avem N = 37 · 73 = 2701 şi<br />

45


N = 1072 = 16 · 67 nu verifică. În sfârşit, p = 79 implică N = 79 · 97 = 7663 şi<br />

N = 3667 = 19 · 193, care nu verifică 2.<br />

În concluzie, singura soluţie a problemei este numărul 121.<br />

|{z}<br />

n−1<br />

VI.91. Considerăm numerele scrise în baza 8: a 1 = 0, 0 (4) (8)<br />

; a 2 = 0, 0 (04) (8)<br />

;<br />

. . . ; a n = 0, 0(00 . . . 0 4) (8) . Să se arate că numărul N = 1 + 1 + · · · + 1 este<br />

a 1 a 2 a n<br />

divizibil cu 14 (10) .<br />

Vasile Chiriac, Bacău<br />

Soluţie. Trecem de la fracţii zecimale la fracţii ordinare; rolul pe care îl are 9 în<br />

baza 10 este jucat de 7 în baza 8. Avem:<br />

a 1 =<br />

a 2 =<br />

a n =<br />

4<br />

4<br />

=<br />

70‹(8) 7 · 8 + 0 = 1 14 ;<br />

4<br />

4<br />

=<br />

770‹(8) 7 · 8 2 + 7 · 8 + 0 = 1<br />

14(8 + 1) ; . . . ;<br />

4<br />

4<br />

=<br />

77 . . . 70‹(8) 7 · 8 n + . . . + 7 · 8 + 0 = 1<br />

14(8 n−1 + . . . + 8 + 1) ,<br />

de unde concluzia este evidentă (acolo unde nu am precizat baza de numeraţie, aceasta<br />

este 10).<br />

VI.92. De o parte şi de alta a unei drepte AB se consideră punctele M şi N<br />

astfel încât △ABM ≡ △ABN, m(ÖMAN)+m(Ö<strong>MB</strong>N) = 180 ◦ , iar [AB]∩[MN] = ∅.<br />

Să se arate că B este ortocentrul △AMN.<br />

Petru Asaftei, Iaşi<br />

Soluţie. Cum AM = AN şi <strong>MB</strong> = BN, înseamnă că punctele<br />

A şi B se află pe mediatoarea segmentului [MN], prin<br />

M<br />

urmare AB⊥MN. Să arătăm că <strong>MB</strong>⊥AN; ne plasăm<br />

în cazul în care m(ÖMAN) < 90 ◦ , celălalt caz tratânduse<br />

analog. Notăm {P } = <strong>MB</strong> ∩ AN şi, observând<br />

A<br />

B<br />

că P ∈ (AN) (deoarece [AB] ∩ [MN] = ∅), obţinem:<br />

m(ÕP BN) + m(ÕP NB) = [180 ◦ − m(Ö<strong>MB</strong>N)] + [90 ◦ −<br />

m(ÕNAB) − m(ÖBNM)] = 270 ◦ − [180 ◦ − m(ÖMAN)] −<br />

1<br />

P<br />

2 m(ÖMAN) − 1 2 [180◦ − (180 ◦ − m(ÖMAN))] = 90 ◦ +<br />

N<br />

m(ÖMAN) − 1 2 m(ÖMAN) − 1 2 m(ÖMAN) = 90 ◦ . Astfel, m(ÕBP N) = 90 ◦ şi rezolvarea<br />

este încheiată.<br />

VI.93. Fie ABCD un patrulater convex cu m(bA) = m(ÒB) = 80 ◦ şi AB = CD =<br />

DA şi astfel încât există F ∈ (BC) pentru care m(ÕBAF ) = 20 ◦ .<br />

a) Demonstraţi că △AF D este echilateral.<br />

b) Determinaţi măsurile unghiurilorÒC şiÒD.<br />

Cristian Lazăr, Iaşi<br />

46


Soluţie. a) Avem că m(ÕAF B) = 180 ◦ − m(ÕBAF ) − m(ÒB) = 80 ◦ , deci △ABF<br />

este isoscel, cu AB = AF. Atunci AD = AF şi, cum m(ÕDAF ) = 80 ◦ − 20 ◦ = 60 ◦ ,<br />

deducem că △ADF este echilateral.<br />

b) Din a) şi ipoteză obţinem că DF = DC, deci △DF C este isoscel. Pe de altă<br />

parte, m(ÕDF C) = 180 ◦ − m(ÕDF A) − m(ÕAF B) = 40 ◦ . Rezultă că m(ÒC) = 40 ◦ , iar<br />

m(ÒD) = m(ÕADF ) + m(ÕF DC) = 60 ◦ + 100 ◦ = 160 ◦ .<br />

VI.94. Un joc are trei beculeţe. Primul se aprinde la fiecare două secunde. Al<br />

doilea se aprinde prima dată la o secundă după aprinderea primului, apoi la fiecare<br />

trei secunde. Al treilea se aprinde prima dată la a două aprindere a primului, apoi la<br />

fiecare 5 secunde. În primele 10 minute de funcţionare, de câte ori cele trei beculeţe<br />

sunt aprinse simultan<br />

Gabriel Popa, Iaşi<br />

Soluţie. Dacă secunda zero este cea în care se aprinde prima dată primul beculeţ,<br />

atunci în secunda n sunt aprinse simultan toate beculeţele dacă şi numai dacă n =<br />

M2 = M3 + 1 = M5 + 2. Cel mai mic număr cu aceste proprietăţi este n = 22. Apoi,<br />

cum n − 22 = M2 − 22 = M3 − 21 = M5 − 20, deducem că n − 22 se divide simultan<br />

cu 2, 3 şi 5, deci cu 30. Începând cu secunda 22, din 30 în 30 de secunde vor fi aprinse<br />

simultan toate beculeţele; în primele 10 minute de funcţionare, acest fapt se petrece<br />

de 20 ori.<br />

Clasa a VII-a<br />

VII.88. Fie x, y, z numere reale distincte, iar a = (x−y) (y−z), b = (y−z) (z−x),<br />

c = (z−x) (x−y). Să se arate că exact două dintre numerele a, b, c sunt negative,<br />

iar al treilea este pozitiv.<br />

Ovidiu Pop, Satu Mare<br />

Soluţie. Să presupunem că x < y < z, celelalte situaţii discutându-se analog;<br />

atunci x − y < 0, y − z < 0, deci a > 0; y − z < 0, z − x > 0, deci b < 0, iar<br />

z − x > 0, x − y < 0, prin urmare c < 0.<br />

VII.89. Determinaţi cifrele x, y, z pentru careÈ14xyzx5 ∈ Q.<br />

Damian Marinescu, Târgovişte<br />

Soluţie. Evident, trebuie ca 14xyzx5 să fie pătrat perfect; cum acest număr are<br />

ultima cifră 5, penultima cifră va fi 2, deci x = 2. Aplicând algoritmul de extragere<br />

a rădăcinii pătrate pentruÈ142yz25, obţinem că 1192 2 < 142yz25 ≤ 1195 2 şi astfel<br />

numărul dorit va fi 1428025, prin urmare y = 8, z = 0.<br />

VII.90. Rezolvaţi în numere întregi ecuaţia 4 x = 5y + 4.<br />

Ion Vişan, Craiova<br />

Soluţie. Nu putem avea x < 0, deoarece în acest caz 0 < 4 x < 1, iar 5y + 4 ∈ Z,<br />

imposibil. Căutăm soluţii cu x ∈ N; atunci 4 x = (5 − 1) x = M5 + (−1) x şi cum<br />

4 x = 5y + 4, în mod necesar trebuie că x să fie impar. Pentru orice x = 2k + 1, k ∈ N,<br />

avem y = 4(42k − 1)<br />

5<br />

∈ Z, deoarece 4 2k − 1 = 16 k − 1 = M(16 − 1) = M5. În<br />

,k ∈ N.<br />

concluzie, soluţiile ecuaţiei sunt2k + 1, 4(42k − 1)<br />

5<br />

47


VII.91. Fie a ∈ N ∗ 1<br />

, a ≤ 98, iar n =<br />

a (a + 1) + 1<br />

(a + 1) (a + 2) + · · · + 1<br />

98 · 99 .<br />

Demonstraţi că n nu poate fi pătratul unui număr raţional.<br />

Gheorghe Iurea, Iaşi<br />

Soluţie. Cu procedeul uzual de calcul pentru sume telescopice, obţinem că n =<br />

1<br />

a − 1 99 = 99 − a<br />

9 · 11 · a . Presupunem prin absurd că √ n ∈ Q; atunci 99 − a va fi pătratul<br />

11 · a<br />

unui număr raţional. Cum 11 este prim, avem fie că 11|99 − a, fie că a = 11b, b ∈ N ∗ .<br />

În oricare dintre situaţii, a va fi multiplu de 11, fie a = 11k, k ∈ {1, 2, . . . , 8}, iar<br />

99 − a<br />

11a<br />

= 9 − k , k ∈ {1, 2, . . . , 8}. Verificând fiecare dintre cele 8 situaţii, nu obţinem<br />

11k<br />

niciodată pătrat al unui număr raţional, de unde concluzia problemei.<br />

VII.92. În trapezul ABCD cu baza mare [CD], diagonala BD este bisectoarea<br />

unghiuluiÕABC. Perpendiculara în B pe diagonala BD intersectează dreapta AD în<br />

E. Să se arate că dreapta CE trece prin mijlocul laturii [AB].<br />

Dan Nedeianu, Drobeta-Tr. Severin<br />

Soluţie. Din ÕABD ≡ ÕBDC (alterne interne) şi ÕABD ≡ ÕDBC (BD<br />

bisectoare), deducem că ÕDBC ≡ ÕBDC, prin urmare<br />

BC = CD. Dacă {F } = BE ∩ CD, atunci<br />

E<br />

A B<br />

ÕCBF ≡ÕCF B (au complementele congruenteÕDBC,<br />

respectivÕBDF , deci BC = CF. Astfel, C este mijlocul<br />

lui [DF ] şi mediana EC în △EDF va trece<br />

prin mijlocul segmentului [AB] paralel cu baza.<br />

D C F<br />

VII.93. Pe latura [AB] a triunghiului ABC se consideră punctul M şi notăm<br />

m = AM, n = BM. Paralela prin M la AC taie BC în N, iar paralela prin N la<br />

AB taie AC în P . Fie S 1 = A BMN , S 2 = A CNP , S = A ABC , iar x = m . Să se<br />

n<br />

exprime raportul S 1 + S 2<br />

în funcţie de x şi să se afle x pentru care acest raport este<br />

S<br />

minim.<br />

Adrian Corduneanu, Iaşi<br />

Soluţie. Notăm p = BN, q = CN; atunci S 1 = 1 2 np sinÒB, S 2 = 1 2 mq sinÕP NC,<br />

= 1 2 mq sinÒB, S = 1 2 (m + n)(p + q) sinÒB, deci<br />

S 1 + S 2<br />

S<br />

=<br />

=<br />

=<br />

np + mq<br />

(m + n)(p + q)<br />

np + mq (m + n)(p 1 1‹˜<br />

+ q)<br />

:<br />

nq<br />

nq<br />

p<br />

q + m n‹:•m<br />

n + p<br />

q +<br />

= x + 1 x‹:•(x + 1)<br />

1<br />

x + 1‹˜= x2<br />

48<br />

B<br />

n<br />

+ 1<br />

(x + 1) 2 .<br />

m<br />

M<br />

p<br />

A<br />

m<br />

N<br />

P<br />

q<br />

C


Se demonstrează imediat că x2 + 1<br />

(x + 1) 2 ≥ 1 , cu egalitate când x = 1, prin urmare<br />

2<br />

valoarea minimă a raportului este 1 , atinsă când M este mijlocul laturii [AB].<br />

2<br />

VII.94. Determinaţi poligoanele regulate care au proprietatea că oricare trei<br />

vârfuri ale lor determină un triunghi isoscel.<br />

Gheorghe Iurea, Iaşi<br />

Soluţie. Fie A 1 A 2 . . . A n un poligon regulat cu proprietatea din enunţ. Evident<br />

că n = 3 convine; fie n ≥ 4. Cum m(ùA 1 A 2 ) = 360◦<br />

n<br />

△A 1 A 2 A 4 vom avea m(bA 1 ) = 360◦<br />

n<br />

, m(bA 4 ) = 180◦<br />

n<br />

clar că m(bA 1 ) ≠ m(bA 4 ); putem avea m(bA 1 ) = m(bA 2 ) când 360◦<br />

n<br />

5 şi m(bA 2 ) = m(bA 4 ) când 180◦ = 180◦ − 540◦<br />

, m(ùA 2 A 4 ) = 2 · 360◦ , atunci în<br />

n<br />

, iar m(bA 2 ) = 180 ◦ − 540◦<br />

n<br />

. Este<br />

= 180◦ − 540◦<br />

n ⇔ n =<br />

⇔ n = 4. Se verifică uşor că pătratul<br />

n<br />

n<br />

şi pentagonul regulat au proprietatea cerută, ele adăugându-se astfel triunghiului<br />

echilateral.<br />

Clasa a VIII-a<br />

VIII.88. Fie A = {1, 3, 5, . . . , 2n − 1}, iar S 1 şi S 2 reprezintă suma elementelor<br />

lui A, respectiv suma pătratelor elementelor lui A. Să se determine n ∈ N ∗ pentru<br />

care S 2 − 3 · |A| ≥ S 1<br />

nX<br />

nX<br />

.<br />

Laurenţiu Modan, Bucureşti<br />

Soluţie. Avem că |A| = n, S 1 = (2k − 1) = n 2 , iar S 2 = (2k − 1) 2 =<br />

n(4n 2 − 1)<br />

. Atunci:<br />

3<br />

k=1<br />

k=1<br />

S 2 − 3 · |A| ≥ S 1 ⇔ n(4n 2 − 1) − 9n ≥ 3n 2 ⇔ n(n − 10) ≥ 0,<br />

fapt care se întâmplă când n ∈ N, n ≥ 10.<br />

VIII.89. Demonstraţi că √ n 2 + 1 + √ n 2 + 2 + · · · + √ n 2 + 2n < 4n2 + 2n + 1<br />

,<br />

2<br />

∀n ∈ N ∗ .<br />

Lucian Tuţescu, Craiova<br />

Soluţie. Obţinem imediat că √ n 2 + a < n + a<br />

2n , ∀n ∈ N∗ , ∀a > 0; dând lui a<br />

valorile 1, 2, . . . , 2n şi sumând inegalităţile obţinute, rezultă inegalitatea dorită.<br />

VIII.90. Demonstraţi că mulţimea A =x | x = 27 6n+2 + 3 12n+5 + 1, n ∈ N nu<br />

conţine numere prime.<br />

Damian Marinescu, Târgovişte<br />

Soluţie. Observăm că x = (3 6n+2 ) 3 + 3(3 6n+2 ) 2 + 3 · 3 6n+2 + 1 − 3 · 3 6n+2 =<br />

(3 6n+2 + 1) 3 − 3 6n+3 = (3 6n+2 − 3 2n+1 + 1)[(3 6n+2 + 1) 2 + 3 2n+1 · (3 6n+2 + 1) + 3 4n+2 ].<br />

Evident că ambii factori sunt diferiţi de 1 şi atunci x nu poate fi număr prim.<br />

VIII.91. Se consideră funcţia f : {1, 2, . . . , 2008} → N care asociază unui element<br />

n al domeniului, numărul divizorilor săi naturali.<br />

49


a) Determinaţi n pentru care f (n) = 7.<br />

b) Aflaţi valoarea maximă a funcţiei.<br />

c) Dacă f (n) + f (m) + f (p) = 33, arătaţi că măcar unul dintre numerele n, m<br />

sau p este pătrat perfect.<br />

Monica Nedelcu, Iaşi<br />

Soluţie. a) Dacă f(n) = 7, atunci n = p 6 , cu p prim. Convin situaţiile n = 2 6 =<br />

64 şi n = 3 6 = 729.<br />

b) Utilizând formula care dă numărul de divizori ai unui număr natural şi făcând<br />

o analiză simplă, găsim valoarea maximă 40, atinsă pentru n = 2 4 · 3 · 5 · 7 = 1680.<br />

c) Măcar unul dintre numerele f(n), f(m) şi f(p) va fi impar, iar un număr care<br />

are un număr impar de divizori va fi pătrat perfect.<br />

VIII.92. Să se arate că pentru orice număr întreg impar n, există numerele<br />

naturale a şi b astfel încât a (a + 2n) = b (b + n).<br />

Constantin Apostol, Rm. Sărat<br />

Soluţie. Avem succesiv:<br />

a(a + 2n) = b(b + n) ⇔ 4a 2 + 8an + 4n 2 − 4n 2 = 4b 2 + 4bn + n 2 − n 2<br />

⇔ (2a + 2n) 2 − (2b + n) 2 = 3n 2 ⇔ (2a − 2b + n)(2a + 2b + 3n) = 3n 2 .<br />

Cum avem libertatea de a căuta a şi b, încercăm să avem 2a + 2b + 3n = 3n 2 ,<br />

2a − 2b + n = 1; găsim atunci a = 1 4 (3n2 − 4n + 1), b = 1 4 (3n2 − 2n − 1). Ar mai<br />

trebui să avem a, b numere naturale; însă n = 2k + 1, k ∈ Z şi astfel a = 3k 2 + k ∈ Z,<br />

b = 3k 2 + 2k ∈ Z. Observăm şi că numerele a = k(3k + 1) şi b = k(3k + 2) sunt<br />

pozitive pentru k ∈ Z, ceea ce încheie rezolvarea problemei.<br />

VIII.93. Fie ABCDA ′ B ′ C ′ D ′ un paralelipiped oarecare şi O, O ′ punctele de<br />

intersecţie a diagonalelor bazelor. Se notează cu G A şi G A ′ centrele de greutate ale<br />

△BCD şi, respectiv, △B ′ C ′ D ′ şi cu A 1 mijlocul segmentului [G A G A ′]. Notaţiile G B ,<br />

G B ′ şi B 1 etc. se introduc în mod similar. Arătaţi că dreptele AA 1 , BB 1 , CC 1 , DD 1<br />

sunt concurente într-un punct P situat pe OO ′ şi precizaţi poziţia lui P pe [OO ′ ].<br />

Temistocle Bîrsan, Iaşi<br />

Soluţie. Din O, G A ∈ AC şi O ′ , G A ′ ∈ A ′ C ′ rezultă că segmentele [OO ′ ]<br />

şi [G A G A ′] aparţin planului ACC ′ A ′ . Se constată uşor<br />

D<br />

C<br />

că aceste segmente sunt paralele şi egale. Ca urmare,<br />

O.<br />

.<br />

G A<br />

există {P } = AA 1 ∩ OO ′ . Din △AOP ∼ △AG A A 1 şi A<br />

B<br />

faptul că AG A = AO + OG A = AO + 1 3 AO = 4 3 AO,<br />

avem<br />

A 1<br />

P.<br />

.<br />

OP<br />

= AO = 3 G A A 1 AG A 4 ⇔ OP = 3 4 G AA 1 ⇔ OP = 3 8 OO′<br />

D<br />

(∗)<br />

C<br />

. .<br />

ceea ce precizează poziţia lui P pe OO ′ .<br />

G<br />

A O A<br />

B<br />

În mod analog se arată că BB 1 , CC 1 şi DD 1 intersectează<br />

OO ′ în acelaşi punct P precizat de (∗).<br />

50


Notă. Problema se poate uşor extinde la cazul în care A 1 împarte [G A G A ′] în<br />

raportul k ≠ 1 2 şi Û<br />

analog pentru B 1, C 1 , D 1 .<br />

VIII.94. Fie V A 1 A 2 . . . A n o piramidă regulată; notăm cu P poligonul A 1 A 2 . . . A n<br />

şi fie U =nm( V M, (A 1 A 2 A 3 )) | ÛM ∈ Po. Demonstraţi că max U < 2 min U.<br />

Claudiu-Ştefan Popa, Iaşi<br />

Soluţie. Evident că m( V M, (A 1 A 2 A 3 )) = m(ÖV MO), unde O este centrul<br />

lui P. Din considerente de simetrie, este suficient să luăm M<br />

pe segmentul [A 1 N], unde N este mijlocul lui [A 1 A 2 ]. Fie N ′<br />

V<br />

şi M ′ pe [OA 1 ] astfel încât ON ′ = ON, OM ′ = OM; din<br />

congruenţe imediate de triunghiuri, avem căÖV N ′ O ≡ÕV NO,<br />

ÖV M ′ O ≡ÖV MO. Notăm u = m(ÖV N ′ O), v = m(ÖV A 1 O) şi<br />

se arată imediat că M ′ ∈ [N ′ A 1 ], prin urmare u = max U,<br />

v = min U.<br />

Rămâne să demonstrăm că u < 2v; cum u = v+m(ÔN ′ V A 1 ),<br />

ar trebui să avem m(×N ′ V A 1 ) < v, adică N ′ A 1 < N ′ V (∗). u x v<br />

Notăm R = OA 1 , a = ON, h = V O şi se vede uşor că O N M A 1<br />

a = R cos 180◦<br />

n<br />

, N ′ A 1 = R − a = R(1 − cos 180◦<br />

n<br />

), iar N ′ V = √ h 2 + a 2 . Atunci:<br />

(∗) ⇔ R 2 1 − cos 180◦<br />

‹2<br />

< h 2 + a 2 ⇔ R 2 − 2R 2 cos 180◦<br />

n n + R2 cos 2 1802<br />

n<br />

< h 2 + R 2 cos 2 180◦<br />

n ⇔ h2 > R 2 1 − 2 cos<br />

‹⇔ 180◦ h > 1 − 2 cos<br />

n R‹2<br />

180◦<br />

n .<br />

Aceasta din urmă inegalitate este adevărată pentru n ≥ 3, întrucât membrul stâng<br />

este pozitiv, iar cel drept negativ.<br />

Clasa a IX-a<br />

IX.86. Fie O mijlocul ipotenuzei [BC] a triunghiului dreptunghic ABC, r raza<br />

cercului înscris, iar R 1 şi R 2 razele cercurilor circumscrise triunghiurilor AOB, respectiv<br />

AOC. Să se demonstreze că √ R 1 R 2 ≥<br />

a2<br />

2a + 4r .<br />

D. M. Bătineţu-Giurgiu, Bucureşti<br />

bc<br />

Soluţie. Utilizând formulele uzuale, avem că r =<br />

a + b + c , R 1 = a2<br />

4b , R 2 = a2<br />

4c .<br />

Cum a + 2r = a +<br />

2bc<br />

a + b + c = a2 + 2bc + a(b + c)<br />

a + b + c<br />

a2<br />

=<br />

(b + c)(a + b + c)<br />

a + b + c<br />

= b + c,<br />

inegalitatea de demonstrat revine la<br />

4bc ⇔ 2(b + c) ⇔ (√ b − √ c) 2 ≥ 0, evident<br />

adevărată. Egalitatea se atinge pentru b = c, deci când △ABC este dreptunghic<br />

isoscel.<br />

IX.87. Demonstraţi că într-un triunghi ascuţitunghic, cu notaţiile uzuale, are loc<br />

inegalitatea<br />

a<br />

b 4 + c 4 + b<br />

c 4 + a 4 + c<br />

a 4 + b 4 < 3<br />

4Rrp .<br />

Gheorghe Molea, Curtea de Argeş<br />

51<br />

a2


Soluţie. Pentru x, y > 0, avem succesiv:<br />

(x − y) 2 (x 2 + y 2 + xy) ≥ 0 ⇔ (x 2 + y 2 − 2xy)(x 2 + y 2 + xy) ≥ 0<br />

⇔ x 4 + y 4 ≥ x 3 y + xy 3 ⇔ x4 + y 4<br />

≥ x 2 + y 2 .<br />

xy<br />

Aplicând această inegalitate pentru laturile unui triunghi ascuţitunghic, obţinem:<br />

a 4 + b 4<br />

≥ a 2 + b 2 > a 2 + b 2 − 2ab cos C = c 2<br />

ab<br />

⇔ a4 + b 4<br />

> c ⇔ a4 + b 4<br />

> abc ⇔ c<br />

abc<br />

c<br />

a 4 + b 4 < 1<br />

abc .<br />

Considerând relaţiile analoage şi sumându-le, obţinem concluzia dacă observăm că<br />

abc = 4SR = 4Rrp.<br />

IX.88. Demonstraţi că 15 · 25 n + 32 · n 2 + 120n − 15 . . 128, ∀n ∈ N.<br />

Lucian Tuţescu, Craiova<br />

Soluţie. Notând A n = 15 · 25 n + 32n 2 + 120n − 15, vom arăta că A n<br />

. .128, ∀n ∈<br />

N, prin inducţie matematică. Avem că A 0 = 0 . .128; presupunem că A k<br />

. .128 şi să<br />

demonstrăm că A k+1<br />

. .128. Cum Ak+1 = A k + 360 · 25 k + 64k + 152, 360 = 256 + 104 şi<br />

152 = 182 + 24, rămâne să arătăm că 104 · 25 k + 64k + 24 . .128, ceea ce revine la faptul<br />

că B k = 13 · 25 k + 8k + 3 . .16. Această din urmă afirmaţie se probează uşor printr-o<br />

nouă inducţie matematică.<br />

IX.89. Să se arate că pentru orice n ∈ N ∗ are loc inegalitatea<br />

1 + 1<br />

2 √ 2‚1<br />

2 2 + 1 5 2 + · · · + 1<br />

(3n − 1) 2Œ> 1<br />

n + 1 + 1<br />

n + 2 + · · · + 1<br />

3n + 1 .<br />

Soluţie. Din inegalitatea mediilor obţinem că<br />

2 √ 2 · (3k − 1) 2 = 2 · (3k − 1) ·È1 · (3k − 1) ·È2(3k − 1)<br />

< 2(3k − 1) · 1 + 3k − 1<br />

2<br />

· 2 + 3k − 1<br />

2<br />

=<br />

Titu Zvonaru, Comăneşti<br />

(3k − 1) · 3k · (3k + 1)<br />

, k ∈ N ∗ ,<br />

2<br />

inegalitatea fiind strictă deoarece nu putem avea simultan 3k − 1 = 1 şi 3k − 1 = 2.<br />

1<br />

Rezultă că<br />

2 √ 2(3k − 1) > 2<br />

2 (3k − 1)3k · (3k + 1) = 1<br />

3k − 1 − 2<br />

3k + 1<br />

3k + 1 = 1<br />

3k − 1 −<br />

1<br />

3k + 1<br />

3k + 1 − 1 . Dând valori lui k şi sumând inegalităţile obţinute, găsim tocmai<br />

k<br />

inegalitatea dorită.<br />

IX.90. Să se determine toate şirurile de numere reale (a n ) n≥0<br />

cu proprietatea că<br />

a n+m + a n−m = a 3n + n, ∀n, m ∈ N.<br />

I. V. Maftei, Bucureşti şi Mihai Haivas, Iaşi<br />

52


Soluţie. Dacă notăm b n = a n + n, relaţia din enunţ se scrie sub forma b n+m +<br />

b n−m = b 3n , ∀n, m ∈ N. Pentru n = m = 0, obţinem că b 0 = 0. Pentru n = m,<br />

deducem că b 2n = b 3n , ∀n ∈ N. Luând acum m = 0, găsim că 2b n = b 3n , ∀n ∈ N(1),<br />

de unde b 2n = 2b n , ∀n ∈ N. În sfârşit, facem n = 2m şi rezultă că b 3m + b m = b 6m =<br />

b 2·3m = 2b 3m , ∀m ∈ N, prin urmare b m = b 3m , ∀m ∈ N(2). Din (1) şi (2) obţinem că<br />

b n = 0, ∀n ∈ N, deci a n = −n, ∀n ∈ N.<br />

Clasa a X-a<br />

X.86. Aflaţi x, y ∈ R ∗ + pentru care lg 2 x<br />

y = 3 lg x 3 lg 3 y . A. V. Mihai, Bucureşti<br />

Soluţie. Ecuaţia se scrie succesiv:<br />

lg 2 x − 2 lg x lg y + lg 2 y + 3(lg x lg y − lg x lg 3 − lg 3 lg y + lg 2 3) = 0<br />

⇔ (lg x − lg 3) 2 + (lg y − lg 3) 2 + (lg x + lg y − 2 lg 3) 2 = 0 ⇔<br />

⇔ lg x = lg y = lg 3 ⇔ x = y = 3.<br />

| {z }<br />

n funcţii<br />

X.87. Fie A ⊂ N şi f : A → A o funcţie injectivă; notăm f n = f ◦ f ◦ . . . ◦ f.<br />

Determinaţi f, ştiind că există p, q ∈ N ∗ numere prime între ele astfel încât f p (x) +<br />

f q (x) = 2x, ∀x ∈ A.<br />

Romeo Ilie, Braşov<br />

Soluţie. Fie A = {x 1 , x 2 , x 3 , . . .}, cu x 1 < x 2 < x 3 < . . . . Avem că f p (x 1 ) ≥ x 1 ,<br />

f q (x 1 ) ≥ x 1 şi, cum f p (x 1 ) + f q (x 1 ) = 2x 1 , înseamnă că f p (x 1 ) = f q (x 1 ) = x 1 .<br />

Inductiv se demonstrează că f p (x k ) = f q (x k ) = x k , ∀k ∈ N ∗ (folosim faptul că f p şi<br />

f q sunt funcţii injective), prin urmare f p (x) = f q (x) = x, ∀x ∈ A. Din f(f p−1 (x)) =<br />

x, ∀x ∈ A, deducem că f este surjectivă, deci bijectivă.<br />

Ştim că (p, q) = 1; atunci există u, v ∈ Z pentru care up + vq = 1 şi astfel<br />

| {z }<br />

−k funcţii<br />

f(x) = f up+vq (x) = (f up ◦ f vq )(x) = x, ∀x ∈ A (unde f k = f −1 ◦ f −1 ◦ . . . ◦ f −1 ,<br />

pentru k ∈ Z, k < 0). În concluzie, există o singură funcţie cu proprietatea dată,<br />

anume funcţia identică.<br />

X.88. Fie ABCD un paralelogram, iar M şi N mijloacele laturilor (BC), respectiv<br />

(CD). Dacă AM = BN şi AM ⊥ BN, arătaţi că ABCD este pătrat.<br />

Gheorghe Iurea, Iaşi<br />

Soluţie. Fie a = AB, b = AD; deoarece −−→ AM = −→ 1 −→<br />

AB + AD, −−→ BN = −→ 1 −→<br />

AD − AB<br />

2<br />

2<br />

şi AM⊥BN, obţinem că −−→ AM · −−→ BN = 0 ⇔ −→ −→ 2<br />

AB · AD =<br />

3 (a2 − b 2 ). Apoi, cum<br />

AM = BN, vom avea că −−→ AM 2 = −−→ BN 2 , de unde −→ −→ 3<br />

AB · AD =<br />

8 (b2 − a 2 ). Rezultă că<br />

2<br />

3 (a2 − b 2 ) = 3 8 (b2 − a 2 −→ −→<br />

), deci a = b şi apoi AB · AD = 0. Astfel, ABCD va fi un<br />

pătrat.<br />

Dl. Titu Zvonaru, Comăneşti, observă că ipoteza poate fi slăbită, în sensul că<br />

este suficient ca punctele M şi N să împartă laturile [BC], respectiv [CD], într-un<br />

53


acelaşi raport, adică BM<br />

MC = CN . Domnia sa oferă o demonstraţie analitică, însă<br />

ND<br />

poate fi uşor adaptată şi soluţia de mai sus.<br />

X.89. În planul complex se consideră punctele A (3i), B (4), iar M este un punct<br />

variabil de modul 1.<br />

a) Determinaţi locul geometric al punctului N cu proprietatea că triunghiurile<br />

AOB şi AMN sunt asemenea şi la fel orientate.<br />

b) Găsiţi punctele N 1 , N 2 ale locului ce se plasează pe segmentele [BA], respectiv<br />

[BO], precum şi punctele M 1 , M 2 din care provin.<br />

Dan Brânzei, Iaşi<br />

Soluţie. a) Asemănarea revine la condiţia z N − z A<br />

= z B − z A<br />

, care se<br />

z N − z M z B<br />

rescrie 3z N = 12 + (3 + 4i)z M (∗) ⇔ 3(z N − z B ) =<br />

(3 + 4i)z M . Trecând la module în această relaţie, y<br />

A .<br />

deducem că 3NB = 5 şi aceasta este singura<br />

condiţie pentru N, deoarece egalarea argumentelor<br />

conduce la argumentul variabil al lui M. În concluzie,<br />

locul geometric cerut este cercul C B, 5 3‹.<br />

M N 1<br />

. . 1 M2<br />

.<br />

. .<br />

N 2 . B . N x<br />

b) Punctul N 1 este pe paralela prin M 1 la O<br />

OB, unde M 1 este punctul în care OA taie cercul<br />

unitate; găsim z M1 = i, z N1 = 8 + i. Dacă<br />

.<br />

M<br />

3<br />

= cos α + i sin α şi N 2 este punctul de pe Ox corespunzător, anulând partea<br />

z M2<br />

imaginară a lui z N2 în (*) obţinem condiţia 4 cos α + 3 sin α = 0, deci tg α = − 4 3 şi<br />

pentru un astfel de α găsim z N2 = 7 3 .<br />

X.90. Fie X 1 , X 2 , . . . , X n variabile aleatoare independente, fiecare dintre ele<br />

luând valorile −1 şi 1 cu probabilităţile p, respectiv q. Considerăm Y = X 1 + X 2 +<br />

· · · + X n .<br />

a) Să se calculeze media şi dispersia lui Y .<br />

b) Să se precizeze care este valoarea luată de Y cu probabilitate maximă.<br />

Petru Minuţ, Iaşi<br />

Soluţie. a) Notăm cu Z numărul de variabile X i (din suma care defineşte Y )<br />

care primesc valoarea −1; atunci Y = n − 2Z. Deoarece P (Z = k) = C k n · p p · q n−k ,<br />

k = 0, n, variabila aleatoare Z are media M(Z) = np şi dispersia D(Z) = npq, prin<br />

urmare M(Y ) = n − 2np, iar D(Y ) = 4npq.<br />

b) Avem că P (Y = n − 2k) = P (Z = k) = C k n · p k · q n−k . Acest număr este maxim<br />

pentru k ∈ [np − q, np + q]. Valoarea cea mai probabilă a lui Y este n − 2k 0 , unde<br />

k 0 = [np − q] + 1, dacă np − q /∈ N şi k 0 ∈ {np − q, np + q} pentru np − q ∈ N.<br />

Clasa a XI-a<br />

XI.86. Fie n ∈ 2N ∗ şi A ∈ M m (R); arătaţi că numerele det A n+1 − I mşi<br />

det (A − I m ) au acelaşi semn.<br />

Romanţa Ghiţă şi Ioan Ghiţă, Blaj<br />

54


Soluţie. Are loc egalitatea<br />

A n+1 − I m = (A − I m )(A n + A n−1 +<br />

pY<br />

. . . + A + I m ).<br />

Fie n = 2p, iar ε 1 , ε 1 , ε 2 , ε 2 , . . . , ε p , ε p rădăcinile complexe nereale de ordin n + 1<br />

ale unităţii: atunci A n + A n−1 + . . . + A + I m = (A − ε i I m )(A − ε i I m ). Cum<br />

det(B · B) = det B · det B = det B · det B = | det B| 2 ≥ 0, ∀B ∈ M m (C), rezultă că<br />

det(A n + A n−1 + . . . + I m ) ≥ 0 şi atunci det(A n+1 − I m ) şi det(A − I m ) au acelaşi<br />

semn.<br />

XI.87. Studiaţi convergenţa şirurilor (a n ) n≥1 şi (b n ) n≥1<br />

, unde<br />

a n =<br />

i=1<br />

2008 + cos √ n<br />

2008 + cos √ n + 1 , b n = 2009 + cos √ n<br />

2008 + cos √ n + 1 , ∀n ∈ N∗ .<br />

Liviu Smarandache, Craiova<br />

Soluţie. Vom arăta că a n converge spre 1, în timp ce b n este un şir fără limită.<br />

Avem:<br />

√ √ √ √<br />

a n = 1 + cos √ n − cos √ n + 1 + n n + 1 − n<br />

n + 1 2 sin<br />

sin<br />

2008 + cos √ n + 1 = 1 + 2<br />

2<br />

2008 + cos √ =<br />

n + 1<br />

√ √ √ √ n + 1 − n<br />

n + 1 + n<br />

sin<br />

= 1 + √ 2<br />

√ n + 1 − n<br />

2<br />

√ √ n + 1 − n 2 sin<br />

·<br />

·<br />

2<br />

2<br />

2008 + cos √ n + 1 .<br />

Al doilea factor al produsului tinde către 0, primul către 1, iar al treilea este mărginit;<br />

deducem că limita produsului este 0, deci a n → 1.<br />

1<br />

Observăm apoi că b n = a n +<br />

2008 + cos √ şi cum al doilea termen la sumei<br />

n + 1<br />

nu are limită, rezultă că b n este şir divergent.<br />

XI.88. Fie şirul (x n ) n≥1<br />

definit prin: x 1 ∈0, π 2, x n+1 = 2 tg x n+1 − x n ,<br />

∀n ∈ N ∗ . Studiaţi existenţa limitelor lim<br />

n→∞ x n şi lim<br />

n→∞ nx n.<br />

Dan Popescu, Suceava<br />

Soluţie. Fixăm a ∈x 1 , π 2şi definim f : [0, a] → [0, 2 tg a − a], f(t) = 2 tg t − t;<br />

evident că f este bijectivă şi strict crescătoare şi are loc inegalitatea f −1 (a) < a<br />

(aceasta fiind echivalentă cu f(a) > a ⇔ 2 tg a > 2a, adevărată pentru a ∈0, π 2.<br />

Observăm că x n = f(x n+1 ) şi prin inducţie se arată că x n ∈ (0, a), ∀n ≥ 1, deci<br />

(x n ) n≥1 este şir mărginit. Apoi, aceeaşi inegalitate tg u > u, u ∈0, π 2, impune că<br />

x n+1 > 2x n+1 − x n , de unde x n+1 < x n , ∀n ≥ 1. Conform teoremei lui Weierstrass,<br />

şirul (x n ) n≥1 va fi convergent şi trecând la limită în relaţia de recurenţă găsim că<br />

lim<br />

n→∞ x n = 0.<br />

55


1<br />

Considerăm şirul =<br />

nx<br />

n‹<br />

1<br />

x n<br />

, n ≥ 1, şi îi aplicăm criteriul Stolz-Cesàro:<br />

n n<br />

1<br />

lim − 1<br />

1<br />

1<br />

= lim −<br />

n→∞ x n+1 x n→∞ x n+1 2 tg x n+1 − x<br />

2 tg x n+1 − 2x n+1<br />

= lim<br />

n+1‹<br />

n→∞ x n+1 (2 tg x n+1 − x n+1 ) .<br />

Cum 1 > 1 , n ≥ 1 şi lim<br />

x n+1 x n t→0<br />

lim nx n = +∞.<br />

n→∞<br />

n→∞…<br />

1<br />

2 tg t − 2t<br />

= 0, rezultă că lim<br />

t(2 tg t − t) n→∞<br />

1<br />

nx n<br />

= 0, prin urmare<br />

XI.89. Calculaţi lim<br />

1 2 sin 1 + 1<br />

1<br />

2 2 sin 1 + · · · +<br />

n<br />

2<br />

2 sin 1 .<br />

n<br />

Silviu Boga, Iaşi<br />

Soluţia autorului. Mai general, vom demonstra următoarea<br />

Propoziţie. Fie (a n ) n≥1 un şir de numere reale astfel încât există (b n ) n≥1 ⊂ R<br />

b n (a n+1 − a n )<br />

şir strict monoton şi nemărginit cu lim<br />

= L ∈ R ∗ . Atunci (a n ) n≥1<br />

n→∞ b n+1 − b n<br />

este strict monoton de la un rang încolo şi lim a n = +∞ · (sgnL).<br />

n→∞<br />

b n<br />

Într-adevăr, vom avea că<br />

> 0 de la un rang încolo; cum L ≠ 0, şirul<br />

b n+1 − b n<br />

(a n ) n≥1 va fi strict monoton începând cu acel rang, ceea ce arată şi existenţa limitei<br />

L ′ = lim a n. Vom aplica Stolz-Cesàro pentru a n = a nb n<br />

: dacă există L ′′ = lim d n,<br />

n→∞ n→∞<br />

unde d n = a n+1b n+1 − a n b n<br />

, atunci L ′′ = L ′ .<br />

b n+1 − b Însă<br />

n<br />

d n = a n+1(b n+1 − b n ) + b n (a n+1 − a n )<br />

b n+1 − b n<br />

= a n+1 + b n(a n+1 − a n )<br />

b n+1 − b n<br />

,<br />

ceea ce justifică existenţa lui L ′′ ; în plus, vom avea că L ′ = L ′ + L, egalitate ce nu<br />

poate avea loc decât dacă L ′ = ±∞. În concluzie, L′ = +∞ · (sgn L).<br />

Revenim la problemă: dacă (a n ) este şirul din enunţ, iar b n = n 2 , obţinem că<br />

b n (a n+1 − a n )<br />

= n2<br />

b n+1 − b n 2n + 1 · 1<br />

→ 1 . Deducem că lim<br />

(n + 1) 2 1 2 a n = +∞.<br />

n→∞<br />

sin<br />

n + 1<br />

XI.90. Există funcţii polinomiale p : R → R care să aibă exact n puncte fixe<br />

distincte a 1 , a 2 , . . . , a n ∈ R şi astfel încât pentru fiecare 1 ≤ j ≤ n, ecuaţia p (x) = a j<br />

să aibă soluţie reală unică<br />

Marian Tetiva, Bârlad<br />

Soluţie. Există astfel de funcţii pentru n impar şi nu există pentru n par.<br />

Pentru n = 1, un exemplu ar fi p(x) = a 1 + α(x − a 1 ), cu α ≠ 0; să considerăm<br />

n ≥ 3 impar. Pentru j ∈ {1, . . . , n} oarecare, fie m j valoarea minimă a funcţiei q j (x) =<br />

56<br />

b n


Y(x−a i ) şi fie k un număr real pozitiv fixat astfel încât k < − 1 , ∀j = 1, n (se vede<br />

m j<br />

i≠j<br />

uşor că fiecare m j există şi este negativ deoarece funcţia ia şi valori negative). Arătăm<br />

că funcţia polinomială definită prin p(x) = x + k(x − a 1 ) . . . (x − a n ) are proprietăţile<br />

cerute. Faptul că p(a j ) = a j , j = 1, n, este imediat. Apoi, dacă presupunem că<br />

există j şi un b ≠ a j astfel ca p(b) = a j , obţinem că (b − a j )(1 + kq j (b)) = 0, deci<br />

1 + kq j (b) = 0, ceea ce conduce la contradicţia 0 = 1 + k · q j (b) ≥ 1 + km j > 0.<br />

Fie acum n par şi să presupunem că a 1 , . . . , a n sunt singurele puncte fixe ale<br />

funcţiei polinomiale p. Trebuie să avem p(x) − x = f(x)(x − a 1 ) . . . (x − a n ), cu f<br />

funcţie polinomială de grad par (întrucât nu are zerouri reale); atunci şi p va avea<br />

gradul par, deci limitele sale spre +∞ şi −∞ vor fi ambele +∞ sau ambele −∞.<br />

Tratăm doar primul caz, al doilea fiind analog. Să zicem că a 1 < a 2 < . . . < a n ;<br />

atunci ecuaţia p(x) = a n va avea sigur o soluţie în intervalul (−∞, a 1 ), deoarece<br />

lim (p(x) − a n) = +∞ şi p(a 1 ) − a n = a 1 − a n < 0. Această soluţie este clar diferită<br />

n→−∞<br />

de a n , astfel că p nu îndeplineşte toate condiţiile enunţului în acest caz.<br />

Clasa a XII-a<br />

XII.86. Fie c∈R ∗ , iar f, g :[a, b]→R ∗ + funcţii continue astfel încât f(a+b−x) =<br />

g (x), ∀x ∈ [a, b]. Să se determine y ∈ [a, b] pentru care<br />

Zb<br />

a<br />

[f (x)] g(x)<br />

[g (x)]<br />

dx = cZa+b−y<br />

f(x)<br />

[f (x)] g(x) f(x)<br />

+ [g (x)] y [f (x)] g(x) + [g (x)]<br />

f(x)<br />

dx.<br />

D. M. Bătineţu-Giurgiu, Bucureşti<br />

Soluţie. Cu schimbarea de variabilă a + b − x = t, obţinem că<br />

I =Zb<br />

a<br />

[f(x)] g(x)<br />

[g(x)]<br />

dx =Zb<br />

f(x)<br />

dx.<br />

[f(x)] g(x) + [g(x)]<br />

f(x)<br />

a [f(x)] g(x) + [g(x)]<br />

f(x)<br />

b − a<br />

Adunând cele două integrale, deducem că 2I =Rb<br />

a<br />

dx = b − a, deci I =<br />

2 .<br />

Analog, notând cu J integrala din membrul drept al relaţiei din enunţ, vom găsi că<br />

J = a + b − y. Din egalarea lui I cu J aflăm y = a + b<br />

+ a − b<br />

2c .<br />

2<br />

2<br />

XII.87. a) Fie f : [0, a] → R de clasă C 1 , astfel încât f (a) = 0. Să se arate că<br />

Za<br />

f (t) dt‹2<br />

[f ′ (t)] 2 dt. Pentru ce funcţii se realizează egalitatea<br />

0<br />

≤ a3<br />

3Za<br />

0<br />

b) Fie f : [0, a] → R de clasă C 2 , astfel încât f (a) = f ′ (a) = 0. Să se arate că<br />

Za<br />

f (t) dt‹2<br />

≤<br />

20Za<br />

a5<br />

(f ′′ (t)) 2 dt. Pentru ce funcţii se realizează egalitatea<br />

0<br />

0<br />

Adrian Corduneanu, Iaşi<br />

Soluţie. a) Folosim formula de integrare prin părţi şi inegalitatea Cauchy-Schwarz:<br />

Za<br />

0<br />

tf ′ (t)dt = tf(t)a<br />

⇒<br />

−Za<br />

0 0<br />

(Za<br />

0<br />

f(t)dt = −Za<br />

f(t)dt<br />

0<br />

f(t)dt) 2 = (Za<br />

tf ′ (t)dt) 2 ≤Za<br />

t 2 dt ·Za<br />

[f ′ (t)] 2 dt,<br />

0<br />

57<br />

0<br />

0


de unde inegalitatea dorită.<br />

Egalitatea se atinge dacă f ′ (t) = λt, cu λ ∈ R, deci<br />

pentru f(t) = λ 2 t2 + C. Cum f(a) = 0, obţinem că f(t) = λ 2 (t2 − a 2 ), cu λ ∈ R<br />

constantă arbitrară.<br />

b) Procedăm întru totul analog: vom avea căZa<br />

t 2 f ′′ (t)dt = 2Za<br />

f(t)dt (două<br />

integrări prin părţi), apoi aplicăm Cauchy-Schwarz şi rezultă inegalitatea dorită. Vom<br />

avea egalitate când f ′′ (t) = λt 2 şi f(a) = f ′ (a) = 0, deci pentru f(t) = λ 12 (t4 −4a 3 t+<br />

3a 4 nX<br />

), cu λ ∈ R constantă arbitrară.<br />

XII.88. Să se arate că şirul (a n ) n≥1<br />

este convergent şi să se afle limita sa, unde<br />

a n = 1 k 2 − n 2<br />

n n 2 ln k + n , ∀n ∈ N ∗ .<br />

k<br />

k=1<br />

Laurenţiu Modan, Bucureşti<br />

Soluţie. Şirul (a n ) n≥1 este şirul sumelor Riemann asociat funcţiei f : (0, 1] → R,<br />

f(x) = (x 2 − 1) ln<br />

0<br />

1 + 1 x‹, diviziunii cu puncte echidistante 0 < 1 n < 2 n < . . . < n n şi<br />

sistemului de puncte intermediare k , k = 1, n. Evident, avem de-a face cu o integrală<br />

n<br />

improprie, despre care vom arăta însă că este convergentă şi îi vom calcula valoarea:<br />

Z1<br />

0<br />

(x 2 − 1) ln 1 + 1 x‹dx =x 3<br />

= − 2 3 ln 2 − A + 1 3Z1<br />

= − 1 6 − A − 4 ln 2,<br />

3<br />

unde<br />

0<br />

3 − xln 1 + 1 x‹<br />

x 2 − 3<br />

x + 1 dx = −2 3 ln 2 − A + 1 3<br />

x 3<br />

3 − x·<br />

x 2<br />

− x − 2 ln(x + 1)1<br />

2<br />

1<br />

−Z1<br />

0 0<br />

A = lim<br />

x↘0x 3<br />

3 − xln 1 +<br />

x‹= 1 3<br />

3[ln(x + 1) − ln x]<br />

lim<br />

= lim<br />

x↘0 1<br />

x↘0<br />

x 3 − 3x 2<br />

= lim<br />

x↘0<br />

x 2 (x − 3) 2<br />

(x + 1)(x − 2) = 0.<br />

0<br />

−1<br />

x(x + 1) dx<br />

0<br />

x‹<br />

1<br />

x + 1 − 1<br />

−3x 2 + 6x<br />

x 4 (x − 3) 2<br />

În concluzie, şirul (a n ) n≥1 este convergent spre − 1 6 − 4 ln 2.<br />

3<br />

XII.89. Să se arate că polinomul f = 200X 5 + 196X 4 − 49X 3 + 35X 2 + 14X + 63<br />

este ireductibil peste Q.<br />

Mihai Haivas, Iaşi<br />

Soluţie. Vom folosi criteriul de ireductibilitate al lui Eisenstein şi teorema lui<br />

Gauss. Într-adevăr, numărul prim 7 divide 63, 14, 35, −49 şi 196; 7 nu divide 200, iar<br />

7 2 nu divide 63. Deducem (Eisenstein) că f este ireductibil peste Z, de unde (Gauss)<br />

f va fi ireductibil şi peste Q.<br />

58


XII.90. Fie H corpul cuaternionilor, iar i, j, k unităţile cuaternionice (i 2 =<br />

j 2 = k 2 = −1, ij = −ji = k, jk = −kj = i, ki = −ik = j). Definim<br />

K =¨a + b + c√ 3<br />

2<br />

i + b√ 3 − c + 2d √ 3<br />

4<br />

j + 3b − c√ 3 − 2d<br />

k | a, b, c, d ∈ R«,<br />

4<br />

pe care considerăm operaţiile uzuale între cuaternioni. Să se arate că în acest mod<br />

obţinem un corp necomutativ, izomorf cu corpul cuaternionilor.<br />

Dumitru Mihalache, Bârlad<br />

Soluţie. Notăm u = 1 √<br />

3<br />

2 i + 4 j + 3 √<br />

3<br />

4 k, v = 2 i − 1 √ √<br />

3 3<br />

4 j − 4 k, w = 2 j − 1 2 k.<br />

Se verifică prin calcul că u 2 = v 2 = w 2 = −1, uv = −vu = w, vw = −wv = u,<br />

wu = −uw = v, iar orice element al lui K se scrie sub forma a + bu + cv + dw.<br />

Funcţia f : H → K, f(a + bi + xj + dk) = a + bu + cv + dw este bijectivă şi realizează<br />

un transport de structură între H şi K, de unde concluzia.<br />

IMPORTANT<br />

• În scopul unei legături rapide cu redacţia revistei, pot fi utilizate următoarele<br />

adrese e-mail: t birsan@yahoo.com şi profgpopa@yahoo.co.uk . Pe<br />

această cale colaboratorii pot purta cu redacţia un dialog privitor la materialele<br />

trimise acesteia, procurarea numerelor revistei etc. Sugerăm colaboratorilor<br />

care trimit probleme originale pentru publicare să le numeroteze<br />

şi să-şi reţină o copie xerox a lor pentru a putea purta cu uşurinţă o discuţie<br />

prin e-mail asupra acceptării/neacceptării acestora de către redacţia revistei.<br />

• La problemele de tip L se primesc soluţii de la orice iubitor de matematici<br />

elementare (indiferent de preocupare profesională sau vârstă). Fiecare dintre<br />

soluţiile acestor probleme - ce sunt publicate în revistă după un an - va fi<br />

urmată de numele tuturor celor care au rezolvat-o.<br />

• Adresăm cu insistenţă rugămintea ca materialele trimise revistei<br />

să nu fie (să nu fi fost) trimise şi altor publicaţii.<br />

• Rugăm ca materialele tehnoredactate să fie trimise pe adresa redacţiei<br />

însoţite de fişierele lor (de preferinţă în L A TEX).<br />

• Pentru a facilita comunicarea redacţiei cu colaboratorii ei, autorii materialelor<br />

sunt rugaţi să indice adresa e-mail.<br />

59


Soluţiile problemelor pentru pregătirea<br />

concursurilor propuse în nr. 1/2008<br />

A. Nivel gimnazial<br />

G136. Determinaţi numerele reale x, y, z, pentru care<br />

2 −x + 3 · 2 −y + 2 −z = 2 x + 3 · 2 y+2 + 2 z+2 = 9.<br />

Andrei Nedelcu, Iaşi<br />

Soluţia 1 (a autorului). Este binecunoscută inegalitatea a2<br />

x + b2<br />

y + c2<br />

z<br />

(a + b + c) 2<br />

x + y + z , ∀a, b, c ∈ R, ∀x, y, z ∈ (0, ∞), cu egalitate când a x = b y = c z . Atunci:<br />

9 = 2 −x + 3 · 3 −y + 2 −z = 12<br />

2 x + 62<br />

3 · 2<br />

y+2<br />

+<br />

22<br />

2 z+2 ≥ 9 2<br />

2 x + 3 · 2 y+2 + 2 z+2 = 9<br />

şi deducem că 1<br />

2 x = 1<br />

2 y+1 = 1 = t. Obţinem că t + 6t + 2t = 9, prin urmare t = 1<br />

2z+1 şi astfel x = 0, y = −1, z = −1.<br />

Soluţia 2 (Paul Georgescu). Egalităţile din enunţ se pot rescrie sub forma<br />

2 −x + 6 · 2 −(y+1) + 2 · 2 −(z+1) = 2 x + 6 · 2 y+1 + 2 · 2 z+1 = 9. Din inegalitatea CBS<br />

urmează că<br />

9 · 9 = (2 −x + 6 · 2 −(y+1) + 2 · 2 −(z+1) )(2 x + 6 · 2 y+1 + 2 · 2 z+1 ) ≥<br />

≥ ( √ 2 −x · √2 x + √ 6 · 2 −(y+1) · √6<br />

· 2 y+1 + √ 2 · 2 −(z+1) · √2<br />

· 2 z+1 ) 2 = 9 2 .<br />

Cum se atinge efectiv egalitatea, obţinem că 2 x = 2 y+1 = 2 z+1 şi, după înlocuire,<br />

deducem că x = 0, y = −1, z = −1.<br />

G137. Fie a, b, c ∈ Q ∗ + şi λ = 2√ a + √ b − √ c<br />

√ √ √ . Să se exprime în funcţie de a, b,<br />

a + b + c<br />

√ √ √ a − b + c<br />

c şi λ numărul real µ =<br />

2 √ a + √ b + √ c .<br />

I. V. Maftei, Bucureşti şi Mihai Haivas, Iaşi<br />

Soluţie. Amplificăm succesiv fracţia ce defineşte λ cu √ a, √ b, √ √ √ √<br />

c; notând x =<br />

ab, y = bc, z = ca, obţinem:<br />

≥<br />

2a + x − z = λ(a + x + z); 2x + b − y = λ(x + b + y); 2z + y − c = λ(z + y + c) ⇔<br />

(1 − λ)x − (1 + λ)z = a(λ − 2); (2 − λ)x − (1 + λ)y = b(λ − 1);<br />

(1 − λ)y + (2 − λ)z = c(λ + 1).<br />

Din acest sistem putem afla x, y, z în funcţie de a, b, c şi λ. Cum µ · √a = a − x − z<br />

2a + x + z ,<br />

deducem că µ = √ 1 a · A · C − B · C + A · D<br />

· a 2a · A · C + B · C + A · D , unde A = 2(2 − 3λ + λ2 ), B =<br />

c(1+λ) 2 −b(1−λ) 2 −a(2−λ) 2 , C = 2(2+λ−λ 2 ), D = c(1+λ) 2 −b(1−λ) 2 +a(2−λ) 2 .<br />

60


G138. a) Numerele reale pozitive a, b, c sunt astfel încât 4abc = a + b + c + 1. Să<br />

se arate că b2 + c 2<br />

+ c2 + a 2<br />

+ b2 + a 2<br />

≥ 2 (ab + bc + ca).<br />

a b c<br />

b) Numerele reale pozitive a, b, c sunt astfel încât b2 + c 2<br />

+ c2 + a 2<br />

+ b2 + a 2<br />

≤<br />

a b c<br />

2 (ab + bc + ca). Să se arate că a + b + c + 1 ≤ 4abc.<br />

Andrei Laurenţiu Ciupan, elev, Bucureşti<br />

Soluţie. a) Conform inegalităţii CBS, avem căb 2 a + c2 a + a2(a + a + 1) ≥<br />

(a + b + c) 2 şi încă două relaţii similare; prin adunare, obţinem că<br />

Xb 2 + c 2<br />

+Xa 2 ≥ (a + b + c) 2 ·<br />

a<br />

Din condiţia 4abc = a + b + c + 1 rezultă imediat că<br />

folosind (1), urmează că<br />

1<br />

2a + 1 + 1<br />

2b + 1 + 1<br />

(1)<br />

2c + 1‹.<br />

1<br />

2a + 1 + 1<br />

2b + 1 + 1<br />

2c + 1 = 1 şi,<br />

Xb 2 + c 2<br />

+ (a 2 + b 2 + c 2 ) ≥ (a + b + c) 2 ⇒Xb 2 + c 2<br />

a<br />

a<br />

b) Folosind (1) şi ipoteza, găsim că<br />

≥ 2(ab + bc + ca).<br />

1<br />

2a + 1 ⇒<br />

·X<br />

(a + b + c) 2 ≥Xb 2 + c 2<br />

+Xa 2 ≥ (a + b + c) 2<br />

a<br />

1<br />

2a + 1 + 1<br />

2b + 1 + 1<br />

2c + 1 ≤ 1 ⇒X(2a + 1)(2b + 1) ≤ (2a + 1)(2b + 1)(2c + 1)<br />

⇒ 4Xab + 4Xa + 3 ≤ 4Xab + 2Xa + 8abc + 1 ⇒ a + b + c + 1 ≤ 4abc.<br />

G139. Denisa scrie pe tablă numerele 1, 2, 3, . . . , 2008. Ea alege două numere,<br />

le şterge de pe tablă şi scrie în loc modulul diferenţei lor, repetând această operaţie<br />

până când pe tablă rămâne un singur număr. Poate proceda Denisa în aşa fel încât<br />

numărul rămas să fie 2007 Dar 2008<br />

Iulieta Grigoraş, Iaşi<br />

Soluţie. Se constată uşor că paritarea numărului de numere impare de pe tablă<br />

este un invariant. Cum iniţial sunt 1004 numere impare, în final trebuie să avem un<br />

număr par de numere impare şi acest număr va fi zero, prin urmare pe tablă nu poate<br />

rămâne 2007.<br />

La a doua întrebare, răspunsul este afirmativ. Un procedeu de a obţine 2008 ar<br />

putea fi următorul: Denisa înlocuieşte numerele 2 şi 3 cu 1, 4 şi 5 cu 1, . . . , 2006 şi 2007<br />

cu 1 şi rămâne astfel cu 1004 de 1 şi un 2008. Alegând perechi (1, 1) şi înlocuindu-le<br />

cu 0, rămâne cu 502 de 0 şi un 2008. Indiferent ce va face în continuare, în final<br />

rămâne pe tablă numărul 2008.<br />

G140. Un poligon cu n laturi este împărţit în n−2 triunghiuri cu ajutorul a n−3<br />

diagonale ale sale care nu se intersectează în puncte interioare (o astfel de împărţire<br />

se numeşte triangulaţie a poligonului). Notăm cu T 0 numărul triunghiurilor ale căror<br />

61


laturi sunt toate diagonale ale poligonului şi cu T 2 numărul triunghiurilor care au câte<br />

două laturi care sunt laturi şi pentru poligon, iar a treia latură diagonală a poligonului.<br />

Să se arate că T 2 = T 0 + 2.<br />

Marian Tetiva, Bârlad<br />

Soluţie. Mai considerăm şi triunghiurile care au exact o latură care este şi latură<br />

a poligonului şi notăm cu T 1 numărul lor. Avem că n = 2T 2 + T 1 (numărând în<br />

două feluri laturile poligonului) şi că n − 2 = T 2 + T 1 + T 0 (numărând în două<br />

moduri triunghiurile). Egalând cele două expresii ale lui n, obţinem că 2T 2 + T 1 =<br />

T 2 + T 1 + T 0 + 2, de unde T 2 = T 0 + 2.<br />

G141. Se consideră o reţea de drepte care formează prin intersecţii pătrate congruente.<br />

Marcăm 2n + 1 vârfuri ale unor astfel de pătrate, n ≥ 2, astfel încât orice<br />

dreaptă din reţea să conţină cel mult un punct marcat. Să se arate că există măcar<br />

două puncte marcate care sunt separate atât pe orizontală, cât şi pe verticală, de câte<br />

un număr impar de drepte ale reţelei.<br />

Petru Asaftei, Iaşi<br />

Soluţie. Asociem nodurilor reţelei coordonate întregi. Fie N 1 (a 1 , b 1 ), N 2 (a 2 , b 2 ),<br />

. . . , N j (a j , b j ), j ≥ n + 1, numărul maxim de puncte marcate pentru care abscisele<br />

a 1 , a 2 , . . . , a j au aceeaşi paritate; sumele a 1 + a 2 , a 1 + a 3 , . . . , a j−1 + a j vor fi toate<br />

pare. Dintre numerele b 1 , b 2 , . . . , b j , cel puţin j 2 sau•j<br />

2˜+1, funcţie de paritatea lui j,<br />

au aceeaşi paritate. Cum min{ j 2 ,•j<br />

2˜+1} ≥ 2, înseamnă că cel puţin o sumă dintre<br />

b 1 + b 2 , b 1 + b 3 , . . . , b j−1 + b j este pară; fie aceasta b p + b q . Vom arăta că punctele<br />

N p (a p , b p ) şi N q (a q , b q ) îndeplinesc cerinţa din enunţ.<br />

Avem că a p + a q<br />

, b p + b q<br />

∈ Z, deci mijlocul O al segmentului N p N q este nod al<br />

2 2<br />

reţelei. Fie d 0 , d p , d q dreptele orizontale ale reţelei care trec prin O, N p , respectiv N q .<br />

Numărul dreptelor din reţea cuprinse între d 0 şi d p este acelaşi cu cel al dreptelor<br />

cuprinse între d 0 şi d q ; fie k acest număr. Atunci N p şi N q sunt separate pe orizontală<br />

de 2k + 1 drepte (incluzând şi pe d 0 ). Analog se judecă pe verticală.<br />

G142. Spunem că vârful A al triunghiului ABC are proprietatea (P ) dacă AX <<br />

BC, ∀X ∈ (BC). Să se arate că dacă fiecare vârf al △ABC are proprietatea (P ),<br />

atunci triunghiul este echilateral.<br />

Doru Buzac, Iaşi<br />

Soluţie. Arătăm întâi că dacă △ABC este echilateral, fiecare vârf are proprietatea<br />

(P ). Fie X ∈ (BC); atunci m(ÕAXB) > m(ÕACX) (proprietate a unghiului<br />

exterior), deci m(ÕAXB) > m(ÕABX) şi atunci AB > AX, prin urmare AX < BC şi<br />

astfel vârful are proprietatea (P ). Analog se procedează pentru celelalte vârfuri.<br />

În continuare, demonstrăm că dacă △ABC nu este echilateral, vârful care se<br />

opune laturii mai scurte nu are proprietatea (P ). Să zicem că c este latura cea<br />

mai scurtă; raţionamentul funcţionează şi dacă sunt două laturi de lungime c. Notăm<br />

{M} = (BC)∩C(C, c), {N} = (AC)∩C(C, c) şi fie Y un punct oarecare al arculuiøMN,<br />

interior triunghiului (evident că există astfel de puncte). Dacă {X} = CY ∩ (AB),<br />

atunci CX > CY = c = AB, prin urmare vârful C nu are proprietatea (P ).<br />

62


G143. Considerăm triunghiul ABC, iar D, D ′ sunt puncte pe dreapta BC astfel<br />

încâtÕCAD ≡ÕABC, iarÖBAD ′ ≡ÕACB. Bisectoarele interioare ale unghiurilorÕBAD<br />

şiÖCAD ′ taie dreapta BC în E, respectiv F . Să se arate că cercul circumscris △AEF<br />

şi cercul înscris în △ABC sunt concentrice.<br />

Neculai Roman, Mirceşti (Iaşi)<br />

Soluţie.<br />

puncte D şi D ′ se află pe segmentul [BC]; vom<br />

face justificarea în situaţia din figură, în rest<br />

judecându-se analog.<br />

Deoarece m(ÕAF B) = m(ÕACB) + m(ÕCAF ) =<br />

m(ÖBAD ′ ) + m(ÖD ′ AF ) = m(ÕBAF ), înseamnă că<br />

△ABF este isoscel şi analog △ACE va fi tot isoscel.<br />

Mediatoarele segmentelor [AF ] şi [AE] sunt<br />

bisectoarele unghiurilorÒB, respectivÒC şi de aici<br />

urmează concluzia dorită.<br />

Sunt de considerat mai multe cazuri, după cum unul sau ambele<br />

B<br />

A<br />

. . . .<br />

E D D F<br />

G144. Fie ABCD un patrulater cu AB = BC. Să se arate că m(ÕBAD) +<br />

m(ÕBCD) = 90 ◦ dacă şi numai dacă AB 2 · CD 2 + AD 2 · BC 2 = AC 2 · BD 2 .<br />

Ioan Săcăleanu, Hârlău<br />

Soluţie. Fie T, S ∈ (BD astfel încât ÕAT B ≡ ÕBAD, iar ÕCSB ≡ ÕBCD.<br />

Din asemănarea △ABT ∼ △DBA (U.U.),<br />

obţinem că AB<br />

BD = BT<br />

AB = AT , prin urmare<br />

DA<br />

BT = AB2<br />

AB · AD<br />

, iar AT = . Analog, din<br />

D<br />

BD BD<br />

△DBC ∼ △CBS obţinem că BS = BC2<br />

BD , iar<br />

BC · CD<br />

CS = . Cum AB = BC, rezultă că A<br />

BD<br />

BT = BS, deci T = S. Atunci:<br />

B<br />

m(bA) + m(ÒD) = 90 ◦ ⇔ m(ÕAT C) = 90 ◦ ⇔ AT 2 + T C 2 = AC 2 ⇔<br />

AB 2 · AD 2<br />

BD 2 + BC2 · CD 2<br />

BD 2 = AC 2 ⇔ BC 2 · AD 2 + AB 2 · CD 2 = AC 2 · BD 2 .<br />

G145. Se consideră triunghiul isoscel ABC cu AB = AC, iar pe arcul deschis<br />

⌢<br />

BC care nu-l conţine pe A al cercului circumscris triunghiului se ia un punct M. Să<br />

se arate că<br />

√<br />

<strong>MB</strong> · MC < MA <<br />

√<br />

<strong>MB</strong> · MC +<br />

T<br />

AB · AC<br />

√<br />

<strong>MB</strong> · MC<br />

.<br />

Gheorghe Costovici, Iaşi<br />

Soluţie. Fie 2α = m(ÕBAC); evident că BC = 2AB · sin α. Aplicând prima<br />

teoremă a lui Ptolemeu patrulaterului inscriptibil ABMC, obţinem:<br />

AM · BC = AB · MC + AC · <strong>MB</strong> ⇔ 2AM · AB · sin α = AB(<strong>MB</strong> + MC) ⇒<br />

<strong>MB</strong> + MC 1<br />

⇒ MA = ·<br />

2 sin α ≥ √ 1<br />

<strong>MB</strong> · MC ·<br />

sin α > √ <strong>MB</strong> · MC,<br />

63<br />

C<br />

C


cu inegalitate strictă deoarece α < 90 ◦ . Aplicând acum a doua teoremă a lui Ptolemeu,<br />

avem:<br />

MA<br />

BC<br />

AB · AC + <strong>MB</strong> · MC<br />

=<br />

AB · <strong>MB</strong> + AC · MC ⇒ MA<br />

2AB · sin α = AB2 + <strong>MB</strong> · MC<br />

AB(<strong>MB</strong> + MC) ⇒<br />

⇒ MA = AB2 + <strong>MB</strong> · MC<br />

<strong>MB</strong> + MC<br />

2<br />

de unde concluzia problemei.<br />

· sin α < AB2 + <strong>MB</strong> · MC<br />

√<br />

<strong>MB</strong> · MC<br />

· 1,<br />

B. Nivel liceal<br />

L136. Fie A, B, C trei puncte pe sfera S de centru O, iar M 1 şi M 2 două<br />

puncte exterioare sferei astfel încât OM 1 şi OM 2 să intersecteze planul (ABC) în<br />

două puncte interioare △ABC. Dacă M 1 A ≥ M 2 A, M 1 B ≥ M 2 B şi M 1 C ≥ M 2 C,<br />

să se arate că M 1 O ≥ M 2 O.<br />

Cătălin Ţigăeru, Suceava<br />

Soluţie. Vom demonstra întâi un rezultat ajutător:<br />

Lemă. Se consideră segmentul ST, iar X, Y sunt puncte în spaţiu astfel<br />

încât XS ≥ Y S, XT ≥ Y T. Dacă Q ∈ [ST ], atunci<br />

XQ ≥ Y Q.<br />

X.<br />

Vom nota −→ a 1 = −→ XS,<br />

−→ a 2 = −−→ −→<br />

XT , b 1 = −→ Y S,<br />

−→ b 2 = −→ Y T , λ = SQ<br />

−−→<br />

∈ [0, 1]; avem că XQ = .<br />

Y<br />

ST<br />

(1 − λ) −→ a 1 + λ −→ −→<br />

a 2 , Y Q = (1 − λ) −→ b 1 + λ −→ b 2 , iar<br />

−→ a . .<br />

2 − −→ a 1 = −→ b 2 − −→ b 1 . Considerăm funcţia f :<br />

.<br />

[0, 1] → R, f(λ) = |(1 − λ) −→ a<br />

T<br />

1 + λ −→ a 2 | 2 − |(1 − S Q<br />

λ) −→ b 1 + λ −→ b 2 | 2 = 2λ · [ −→ a 1 · −→ a 2 − −→ b 1 · −→ b 2 − | −→ a 1 | 2 + | −→ b 1 | 2 ] + | −→ a 1 | 2 − | −→ b 1 | 2 .<br />

Avem de-a face cu o funcţie liniară, cu f(0) = | −→ a 1 | 2 − | −→ b 1 | 2 ≥ 0 şi f(1) = | −→ a 2 | 2 −<br />

| −→ b 2 | 2 ≥ 0; deducem că f(λ) ≥ 0, ∀λ ∈ [0, 1] şi astfel lema este demonstrată.<br />

Revenim la problemă: fie {P } = M 1 O ∩ (ABC),<br />

{P 1 } = AP ∩ BC, unde P 1 ∈ [BC] şi P ∈ [AP 1 ], conform<br />

ipotezei. Aplicând de două ori lema precedentă,<br />

.<br />

M 1<br />

obţinem că M 1 P 1 ≥ M 2 P 1 , apoi că M 1 P ≥ M 2 P. Atunci .<br />

M<br />

M 2<br />

2 O ≤ M 2 P +P O ≤ M 1 P +P O = M 1 O, ceea ce încheie<br />

C<br />

soluţia.<br />

A. . .<br />

P .<br />

L137. Considerăm △ABC înscris în cercul C şi fie<br />

P<br />

.<br />

1<br />

C 1 cercul de centru O 1 , tangent la AB, BC şi la cercul<br />

B<br />

C în M, K, respectiv L. Paralela prin B la MK intersectează<br />

dreptele LM şi LK în R, respectiv S. Să se<br />

O<br />

arate că unghiulÖRO 1 S este ascuţit.<br />

Neculai Roman, Mirceşti (Iaşi)<br />

Soluţie. Fie Q punctul de pe semidreapta [BO 1 pentru care unghiulÕRQS<br />

64


este drept. Cum O 1 B⊥MK şi MK ∥ RS, rezultă că<br />

O 1 B⊥RS. Conform teoremei înălţimii, avem că QB 2 =<br />

RB ·BS şi atunci concluzia problemei revine la a demonstra<br />

că O 1 B 2 > RB · RS(∗).<br />

Avem că ÕBKS ≡ ÕLKC ≡ ÖLMK ≡ ÖMRB, iar<br />

m(Ö<strong>MB</strong>R) = m(ÕKBS) = 90 ◦ − 1 2 m(ÒB) ([BO 1 fiind bisectoare<br />

pentruÕABC). Rezultă că △MRB ∼ △SKB, de<br />

unde <strong>MB</strong><br />

SB = RB<br />

KB ⇔ <strong>MB</strong>2 = RB · SB. Însă O 1B > <strong>MB</strong><br />

şi astfel deducem că (∗) este adevărată, ceea ce încheie<br />

rezolvarea.<br />

L138. Fie ABC un triunghi cu AB ≠ AC, m(bA) < 90 ◦ , unghiulbA fiind cel mai<br />

mare al triunghiului. Notăm cu M mijlocul lui [BC] şi T punctul de intersecţie al<br />

simedianei din A cu mediatoarea lui [BC]. Să se arate că 2AM < AT .<br />

Titu Zvonaru, Comăneşti şi Cristian Pravăţ, Iaşi<br />

Soluţie. Fie {D} = AT ∩ BC şi α = m(ÕCBT ) = m(ÕBCT ). Cum AD este<br />

simediană, avem:<br />

c 2<br />

b 2 = BD<br />

CD = S ABD<br />

= S BT D<br />

= S ABD + S BT D<br />

=<br />

S ACD S CT D S ACD + S CT D<br />

= S ABT<br />

S ACT<br />

=<br />

⇒ c b<br />

AB · BT · sin(B + α)<br />

AC · CT · sin(C + α)<br />

=<br />

sin(B + α)<br />

sin(C + α) .<br />

R<br />

B<br />

B<br />

S<br />

A<br />

M<br />

. . M<br />

D<br />

Folosind teorema sinusurilor şi faptul că B ≠ d, deducem<br />

că<br />

T<br />

sin B sin(B + α) = sin C sin(C + α) ⇔ cos(2B + α) = cos(2C + α) ⇔<br />

⇔ 2B + α = 360 ◦ − 2C − α ⇔ A = α.<br />

Ţinând cont că BT =<br />

△ABT obţinem:<br />

a<br />

2 cos A şi AM 2 = 1 2 (b2 + c 2 ) − 1 4 a2 , cu teorema cosinusului în<br />

AT 2 = c 2 +<br />

a2<br />

4 cos 2 A − 2c a · cos(A + B) ⇔<br />

2 cos A<br />

⇔ (2T cos A) 2 = c 2 (2 cos A) 2 + a 2 + ac · (2 cos A)(2 cos C) ⇔<br />

⇔ (2AT cos A) 2 = c2 (b 2 + c 2 − a 2 )<br />

b 2 c 2 + a 2 + ac · b2 + c 2 − a 2<br />

· b2 − c 2 + a 2<br />

⇔<br />

bc<br />

ab<br />

⇔ (2AT cos A) 2 = 2b 2 + 2c 2 − a 2 ⇔ (2AT cos A) 2 = 4AM 2 ⇔ AM = AT cos A.<br />

ÎnsăbA este cel mai mare unghi, deci m(bA) > 60 ◦ şi astfel cos A < cos 60 ◦ = 1 2 , de<br />

unde concluzia problemei.<br />

65<br />

A<br />

O 1<br />

K<br />

L<br />

C<br />

C


L139. Fie A 1 A 2 · · · A n un poligon regulat, iar M un punct variabil în interiorul<br />

sau pe laturile poligonului. Să se determine maximul produsului f (M) = MA 1 ·MA 2 ·<br />

· · · · MA n , precum şi punctele M care realizează acest maxim, în fiecare din cazurile:<br />

a) n = 3; b) n = 6.<br />

Dumitru Mihalache şi Marian Tetiva, Bârlad<br />

Soluţie. a) Raportăm planul la un reper cartezian astfel încât vârfurile triunghiului<br />

echilateral să aibă afixele A 1 (O), A 2 (1), A 3 ( 1 √<br />

3<br />

2 + i ). Dacă z este afixul lui M,<br />

2<br />

atunci g(z) = z(z − 1)(z − 1 √<br />

3<br />

2 − i ) este o funcţie olomorfă pentru z în domeniul<br />

2<br />

plan delimitat de triunghi. Conform principiului maximului modulului, maximul lui<br />

f(M) = |g(z)| se realizează pe frontiera domeniului, iar datorită simetriei triunghiului<br />

echilateral este suficient 2" să căutăm acest maxim pentru M ∈ [A 1 A 2 ]. Aceasta<br />

înseamnă că trebuie să găsim maximul lui |g(z)| pentru z = x ∈ [0, 1], unde<br />

|g(x)| 2 = x 2 (1 − x) x −<br />

2‹2<br />

1 3<br />

+‚√Œ2#=x 2 (1 − x) 2 (x 2 − x + 1) ≤ 3 2<br />

64 .<br />

Într-adevăr, după calcule, inegalitatea anunţată se dovedeşte a fi echivalentă cu (2x −<br />

1) 2 [4x(x−1)(4x 2 −4x+3)−3] ≤ 0, iar aceasta este clară pentru x ∈ [0, 1]. Egalitatea<br />

se atinge doar pentru x = 1 √<br />

3<br />

2 . În concluzie, maximul produsului f(M) este şi este<br />

8<br />

atins când M este unul dintre mijloacele laturilor triunghiului.<br />

b) Procedăm<br />

√<br />

ca mai înainte: alegem un reper cartezian în raport cu care A 1 (0, 0),<br />

3<br />

A 2 (1, 0), A 3‚3<br />

Œ,<br />

2 , A 4 (1, √ 3), A 5 (0, √ 3); A 1 √<br />

3<br />

6‚−<br />

Œşi<br />

2<br />

2 , folosind principiul<br />

2<br />

maximului modulului şi considerentele de simetrie, vom considera că M ∈ [A 1 A 2 ],<br />

deci M(x, 0), cu x ∈ [0, 1]. Atunci<br />

[f(M)] 2 = x 2 (1 − x) 2 (x 2 − 3x + 3)(x 2 − 2x + 4)(x 2 + 3)(x 2 + x + 1)<br />

şi maximul acestei expresii poate fi determinat cu inegalitatea mediilor:<br />

x 2 (1 − x) 2 · x2 − 3x + 3<br />

7<br />

· x2 − 2x + 4<br />

13<br />

· x2 + 3<br />

13<br />

· x2 + x + 1<br />

7<br />

≤1<br />

6 (2x(1 − x) + x2 − 3x + 3<br />

+ x2 − 2x + 4<br />

+ x2 + 3<br />

6<br />

13 13<br />

˜6<br />

− x) + 101<br />

=•142x(1<br />

6 · 91<br />

≤…71<br />

2 + 101<br />

6 · 91<br />

6<br />

= 1 4 6 .<br />

≤<br />

+ x2 + x + 1<br />

)6<br />

=<br />

7<br />

La urmă am folosit inegalitatea x(1 − x) ≤ 1 (faptul că x ∈ [0, 1] asigură că produsul<br />

x(1 − x) este nenegativ). Egalitatea are loc pentru x(1 − x) = x2 − 3x + 3<br />

4<br />

=<br />

7<br />

66


x 2 − 2x + 4<br />

= x2 + 3<br />

= x2 + x + 1<br />

şi x(1 − x) = 1 13 13 7<br />

4 , deci dacă şi numai dacă x = 1 2 ,<br />

91<br />

caz în care M este mijlocul laturii [A 1 A 2 ]. În concluzie, f(M) ≤ şi maximul se<br />

64<br />

atinge în mijloacele laturilor hexagonului.<br />

L140. Fie a, b, c ∈ R ∗ + astfel încât (a+b) 2 +(b+c) 2 +(c+a) 2 +(a+b)(b+c)(c+a)= 4.<br />

Să se arate că a bc + b<br />

ca + c ab ≥ a + b + b + c + c + a .<br />

c a b<br />

Andrei Vrăjitoarea, elev, Craiova<br />

Soluţia 1 (Marius Olteanu, Rm. Vâlcea). Din identitatea a 3 + b 3 + c 3 =<br />

(a+b+c)[(a 2 +b 2 +c 2 )−(ab+bc+ca)]+3abc, rezultă că (a+b+c)(ab+bc+ca)−3abc =<br />

(a + b + c)(a 2 + b 2 + c 2 ) − (a 3 + b 3 + c 2 ). Ţinând seama de acest fapt, inegalitatea<br />

dată se scrie succesiv:<br />

a 2 + b 2 + c 2 ≥ ab(a + b) + bc(b + c) + ac(a + c) = (a + b + c)(a 2 + b 2 + c 2 ) − 3abc<br />

⇔ a 2 + b 2 + c 2 ≥ (a + b + c)(a 2 + b 2 + c 2 ) − (a 3 + b 3 + c 3 ) ⇔<br />

⇔ a 3 + b 3 + c 3 + a 2 + b 2 + c 2 ≥ (a + b + c)(a 2 + b 2 + c 2 ). (1)<br />

Din inegalitatea lui Cebîşev, avem că a 3 +b 3 +c 3 ≥ 1 3 (a+b+c)(a2 +b 2 +c 2 ) şi atunci,<br />

pentru a demonstra (1), ar fi destul să arătăm că<br />

1<br />

3 (a + b + c)(a2 + b 2 + c 2 ) + (a 2 + b 2 + c 2 ) ≥ (a + b + c)(a 2 + b 2 + c 2 ) ⇔<br />

⇔ a 2 + b 2 + c 2 ≥ 2 3 (a + b + c)(a2 + b 2 + c 2 ) ⇔ a + b + c ≤ 3 2 , (2)<br />

deoarece a, b, c ∈ R ∗ + ⇒ a 2 + b 2 + c 2 > 0.<br />

Notăm x = a + b<br />

2 , y = b + c<br />

2 , z = c + a<br />

2 ; condiţia din enunţ devine x2 + y 2 +<br />

z 2 + 2xyz = 1. Conform problemei 19, pg. 10, din Old and New Inequalities, autori<br />

T. Andreescu, V. Cîrtoaje, G. Dospinescu, M. Lascu, apărută la Editura GIL, Zalău,<br />

2002, rezultă că x + y + z ≤ 3 şi, de aici, urmează (2). Astfel, soluţia problemei este<br />

2<br />

încheiată.<br />

Soluţia 2 (a autorului). Vom demonstra că orice ecuaţie de forma α 2 +β 2 +γ 2 +<br />

αβγ = 4 are soluţiile pozitive de forma (2 cos A, 2 cos B, 2 cos C), unde A, B, C sunt<br />

unghiurile unui triunghi ascuţitunghic. Într-adevăr, se observă imediat că α, β, γ ∈<br />

(0, 2) şi atunci există A, B ∈0, π 2pentru care α = 2 cos A, β = 2 cos B. Ecuaţia<br />

γ 2 + 4 cos A cos B · γ + 4(cos 2 A + cos 2 B − 1) = c are discriminantul 16(cos 2 A −<br />

1)(cos 2 B − 1) = 16 sin 2 A sin 2 B şi singura soluţie cu şansa de a fi pozitivă este<br />

γ = −2 cos(A + B), dacă A + B > π 2 . Considerând C = π − (A + B) ∈0, π 2,<br />

avem că γ = 2 cos C. Reciproc, un triplet de forma anunţată este soluţie a ecuaţiei,<br />

fapt care rezultă din identitatea cos 2 x + cos 2 y + cos 2 z + 2 cos x cos y cos z − 1 =<br />

4 cos x + y + z · cos −x + y + z · cos x − y + z · cos x + y − z . În concluzie, avem că<br />

2<br />

2<br />

2<br />

2<br />

a+b = 2 cos A, b+c = 2 cos B, c+a = 2 cos C şi, cu substituţiile x =Éa<br />

bc , y =Éb<br />

ca ,<br />

67


z =Éc<br />

, inegalitatea de demonstrat devine<br />

ab<br />

x 2 + y 2 + z 2 ≥ 2xy cos A + 2yz cos B + 2zx cos C,<br />

(∗)<br />

unde x, y, z ∈ R ∗ +, iar A, B, C sunt unghiurile unui triunghi ascuţitunghic.<br />

Rămâne să justificăm (∗). Trecem totul în stânga şi gândim expresia ca fiind de<br />

gradul II în x. Astfel, ar fi suficient să demonstrăm că discriminantul este negativ;<br />

avem:<br />

∆ = 4(y cos A + z cos C) 2 − 4(y 2 + z 2 − 2yz cos B) =<br />

= 4y 2 (cos 2 A − 1) + 4z 2 (cos 2 C − 1) + 8yz(cos A cos C + cos(A + C)) =<br />

= −4y 2 sin 2 A − 4z 2 sin 2 C + 8yz sin A sin C = −4(y sin A − z sin C) 2 ≤ 0,<br />

∀y, z ∈ R, ceea ce încheie soluţia problemei.<br />

L141. Dacă x, y, z sunt numere reale pozitive cu x 3 + y 3 + z 3 = 3, atunci<br />

x + 2<br />

2x 2 + 1 + y + 2<br />

2y 2 + 1 + z + 2<br />

2z 2 + 1 ≥ 3.<br />

Titu Zvonaru, Comăneşti şi Nela Ciceu, Bacău<br />

Soluţie. Pentru x ≥ 0 avem că x + 2<br />

2x 2 + 1 ≥ 3<br />

x 3 (1), deoarece<br />

+ 2<br />

(1) ⇔ x 4 + 2x 3 − 6x 2 + 2x + 1 ≥ 0 ⇔ (x 2 + 4x + 1)(x − 1) 2 ≥ 0,<br />

2‹<br />

iar ultima inegalitate este evident adevărată pentru x pozitiv. Folosind (1) şi analoagele<br />

sale, precum şi inegalitatea mediilor MH ≤ MA, obţinem:<br />

x + 2<br />

2x 2 + 1 + y + 2<br />

2y 2 + 1 + z + 2<br />

2z 2 + 1 ≥ 3 · 1<br />

x 3 + 2 + 1<br />

y 3 + 1 + 1<br />

z 3 +<br />

≥ 3 ·<br />

9<br />

x 3 + 2 + y 3 + 2 + z 3 + 2 = 3 · 9<br />

= 3. (1)<br />

6 + 3<br />

Egalitatea se atinge pentru x = y = z = 1.<br />

Notă. Soluţie corectă s-a primit de la Marius Olteanu, Rm. Vâlcea, care<br />

observă că inegalitatea are loc pentru x, y, z din [ √ 3 − 2, +∞) cu x 3 + y 3 + z 3 = 3.<br />

L142. Considerăm n ∈ N ∗ , numerele reale strict pozitive a 1 < a 2 < · · · < a n<br />

şi A mulţimea tuturor sumelor ±a 1 ± a 2 ± · · · ± a n , unde semnele se aleg în toate<br />

modurile posibile. Arătaţi că |A| > n2 + n + 2<br />

mX<br />

şi determinaţi numerele a n pentru<br />

2<br />

care se atinge egalitatea.<br />

Gheorghe Iurea, Iaşi<br />

Soluţie. Dacă S = a k , iar b i ∈ A, atunci b i = S − 2(a i1 + a i2 + . . . + a ik ),<br />

k=1<br />

unde a i1 , . . . , a ik , k ≤ n, sunt termenii care apar cu minus în b i . Pentru prima cerinţă<br />

68


a problemei, ar fi suficient să punem în evidenţă n2 + n + 2<br />

termeni distincţi de<br />

2<br />

forma a i1 + a i2 + . . . + a ik , k ∈ 0, n (pentru k = 0, termenul este egal cu 0). Aceşti<br />

termeni sunt 0 < a 1 < a 2 < . . . < a n < a n + a 1 < a n + a 2 < . . . < a n + a n−1 <<br />

a b + a n−1 + a 1 < . . . < a n + a n−1 + a n−2 < . . . < a n + a n−1 + . . . + a 2 + a 1 , în număr<br />

de 1 + n + (n − 1) + . . . + 1 = n2 + n + 2<br />

.<br />

2<br />

Dacă A are exact n2 + n + 2<br />

elementele, atunci orice sumă de forma a i1 + a i2 +<br />

2<br />

. . . + a ik , 0 ≤ k ≤ n, trebuie să se regăsească în lista de mai sus. Avem că a n−1 <<br />

a 1 + a n−1 < a 1 + a n , deci a 1 + a n−1 = a n . Apoi, a n−2 < a 1 + a n−2 < a 1 + a n−1 = a n ,<br />

de unde a 1 + a n−2 = a n−1 . Procedând analog, găsim că a 1 + a n−3 = a n−2 , . . . , a 1 +<br />

a 3 = a 4 , a 1 + a 2 = a 3 , prin urmare a k = a 2 + (k − 2)a 1 , k ∈ 3, n. Totodată,<br />

a n = a 1 + a n−1 < a 2 + a n−2 < a 2 + a n , deci a 2 + a n−2 = a n + a 1 , adică a 2 = 2a 1<br />

şi astfel a k = ka 1 , k ∈ 1, n. Se vede, uşor că mulţimea A a sumelor de forma<br />

a 1 (±1 ± 2 ± . . . ± n), cu a 1 ∈ R ∗ n<br />

+ oarecare, are exact n2 + n + 2<br />

elemente.<br />

2<br />

L143. Să se arate că pentru p număr natural prim şi m, n ∈ {0, 1, . . . , p − 1},<br />

+ m + m<br />

m > n, avem2p<br />

mod p<br />

2p + n≡2p<br />

p + n−m<br />

2.<br />

Marian Tetiva, Bârlad<br />

Soluţie.<br />

X<br />

Considerăm identitatea (1 + X) 2p+m = (1 + X) p · (1 + X) p · (1 +<br />

X) m şi egalăm coeficienţii lui X 2p+n + m<br />

din cei doi membri; obţinem că2p<br />

2p + n=<br />

Deoarecep<br />

p) pentru 1 ≤ q ≤ p − 1, rezultă<br />

k.<br />

q≡0(mod<br />

i+j+k=2p+np<br />

X<br />

că toţi termenii din suma precedentă care cprespund unor valori i, j ∈ {1, 2, . . . , p−1}<br />

sunt 0(mod p 2 + m<br />

), prin urmare2p<br />

p<br />

2p + n≡2·<br />

k−m<br />

n(mod 2 ). Dacă<br />

ip jm<br />

mai ţinem seama şi de binecunoscuta identitate<br />

exact congruenţa din enunţ.<br />

i+k=p+np<br />

X<br />

im<br />

i+k=p+np<br />

im<br />

k=p + m<br />

p + n, găsim<br />

L144. Fie p ∈ N, p ≥ 2; definim şirurile (x n ) n≥1 şi (y n ) n≥1<br />

prin: x 1 =<br />

Èp (p − 1), x n+1 =Èp (p − 1) + x n , y n =2 n p n−1 x n , ∀n ∈ N ∗ , unde {·} desemnează<br />

partea fracţionară. Să se arate că şirul (y n ) este strict monoton.<br />

Sorin Puşpană, Craiova<br />

Soluţie. Se justifică uşor prin inducţie inegalităţile<br />

2 n p n−1 x n < (2p) n − 1, ∀n ∈ N ∗ ; (1)<br />

2 n p n−1 x n > (2p) n − 2 +<br />

8<br />

(2p) n+1 , ∀n ∈ N∗ . (2)<br />

Din (1) şi (2) rezultă că [2 n p n−1 x n ] = (2p) n −2, prin urmare y n = 2−2 n p n−1 (p−x n ).<br />

69


Obţinem că<br />

y n+1 − y n = 2 n p n−1 (−2p 2 + p + 2px n+1 − x n ), ∀n ∈ N ∗ . (3)<br />

Din recurenţa care-l defineşte pe x n , avem că p 2 − x 2 n+1 = p − x n , iar din (1) deducem<br />

că x n ≤ p, ∀n ∈ N ∗ . În acest fel,<br />

p − x n+1 1<br />

= > 1<br />

p − x n p + x n+1 2p ⇒ −2p2 + p + 2px n+1 − x n < 0<br />

şi, ţinând seama de (3), urmează că (y n ) este strict descrescător.<br />

L145. Fie 0 < α < β; definim şirurile (x n ) n≥0<br />

, (y n ) n≥0<br />

prin x 0 = α, y 0 = 0,<br />

x<br />

n<br />

n+1 =Ry<br />

x n<br />

e − α2<br />

t 2 dt, y<br />

n<br />

n+1 =Rx<br />

y n<br />

e − β2<br />

t 2 dt, ∀n ∈ N. Arătaţi că cele două şiruri sunt<br />

convergente şi aflaţi limitele lor.<br />

Marius Apetrii, Iaşi<br />

Soluţie. Folosim teorema de medie:<br />

x n+1 − y n+1 =Zy n<br />

x n<br />

(e − α2<br />

t 2<br />

+ e − β2<br />

t 2 )dt = (e − α2<br />

c 2 n + e − β2<br />

c 2 n )(y n − x n ),<br />

unde c n este un număr între x n şi y n . Rezultă că termenii şirului (x n − y n ) au<br />

semne alternante şi, folosind faptul că x 0 − y 0 > 0, deducem că x 2n ≥ y 2n , x 2n+1 ≤<br />

y 2n+1 , ∀n ∈ N. Din modul de definire al şirurilor, vom avea că x 2n ≥ 0, x 2n+1 ≤ 0,<br />

y 2n ≤ 0 şi y 2n+1 ≥ 0, ∀n ∈ N. Demonstrăm prin inducţie că |x n − y n | ≤ α, ∀n ∈ N.<br />

Afirmaţia este adevărată pentru n = 0. Dacă presupunem că |x n − y n | ≤ α, cum<br />

x n y n ≤ 0, deducem că |x n | ≤ α, |y n | ≤ α, deci |c n | ≤ α, obţinem că<br />

|x n+1 − y n+1 | = |e − α2<br />

c 2 n<br />

+ e − β2<br />

c 2 n | · |x n − y n | ≤ 2 e · |x n − y n | < |x n − y n | < α.<br />

Din relaţia |x n+1 −y n+1 | ≤ 2 e |x n −y n | mai rezultă că |x n+1 −y n+1 | ≤<br />

2 e‹n<br />

·|x 0 −y 0 |,<br />

deci şirul (x n − y n ) converge la zero. Cum |x n+1 | ≤ |x n − y n | şi |y n+1 | ≤ |x n − y n |,<br />

∀n ∈ N ∗ , obţinem că şirurile din enunţ sunt ambele convergente spre zero.<br />

Semnalăm cititorilor reeditarea colecţiei complete a revistei<br />

RECREAŢII ŞTIINŢIFICE (1883-1888)<br />

la 125 de ani de la apariţia primului număr, cu respectarea formei în care a fost<br />

publicată iniţial. <strong>Revista</strong> prezintă şi astăzi interes prin culoarea limbii române şi<br />

terminologiei folosite, prin conţinutul interesant şi de un înalt nivel ştiinţific, precum<br />

şi prin forma grafică frumoasă. Cei interesaţi pot consulta site-ul revistei<br />

http://www.recreatiistiintifice.ro<br />

de unde se poate prelua gratuit. La această adresă pot fi găsite diverse materiale<br />

dedicate revistei, cât şi aspecte de la câteva manifestări consacrate ei.<br />

70


Clasele primare<br />

Probleme propuse 1<br />

P.164. Scrie vecinii vecinului comun al numerelor 16 şi 18.<br />

(Clasa I )<br />

Diana Tănăsoaie, elevă, Iaşi<br />

P.165. După ce dau celor doi fraţi mai mari câte două banane, mănânc şi eu trei<br />

banane. În coş îmi rămâne un număr de banane ce poate fi scris cu două cifre diferite<br />

şi care este cel mai mic număr de acest fel. Câte banane am avut în coş<br />

(Clasa I )<br />

Inst. Maria Racu, Iaşi<br />

P.166. Din cei 8 căţeluşi albi sau negri, cel mult 3 sunt albi. Care este numărul<br />

maxim de căţeluşi negri Dar cel minim<br />

(Clasa a II-a)<br />

Ioana Bărăgan, elevă, Iaşi<br />

P.167. Într-o cameră se joacă un pisoi cu doi pisici, un căţeluş care ţine în gură<br />

o păpuşă şi un băieţel care stă călare pe un căluţ de lemn. Câte picioare participă la<br />

joc<br />

(Clasa a II-a)<br />

Alexandru Dumitru Chiriac, elev, Iaşi<br />

P.168. Există numerele naturale a, b, c, d astfel încât a + b + c + d = 123 şi<br />

a : b = b : c = c : d = 1<br />

(Clasa a III-a)<br />

Amalia Cantemir, elevă, Iaşi<br />

P.169. Calculează diferenţa următoare, fără a efectua parantezele: (2 + 4 + 6 +<br />

8 + . . . + 1000) − (1 + 3 + 5 + 7 + . . . + 999) =<br />

(Clasa a III-a)<br />

Mădălina Bucşă, elevă, Iaşi<br />

P.170. Doi fraţi au cumpărat un teren în formă de pătrat pe care l-au împărţit în<br />

două dreptunghiuri egale. Fiecare doreşte să împrejmuiască propriul teren cu gard.<br />

Cât mai are de lucru fiecare, dacă primul a realizat 430 m, al doilea 470 m, iar<br />

perimetrul pătratului este de 1000 m<br />

(Clasa a III-a)<br />

Dragoş Iacob, elev, Iaşi<br />

P.171. Dacă a+b+c = 175 şi a+2c = 200, calculaţi produsul (2a+b+3c)·(c−b).<br />

(Clasa a IV-a)<br />

Inst. Marian Ciuperceanu, Craiova<br />

P.172. Câte numere abc au suma cifrelor 7 şi pot fi rotunjite cu numărul ab0<br />

(Clasa a IV-a)<br />

Maria Nastasiu, elevă, Iaşi<br />

P.173. Se formează şirul de numere: 34, 334, 344, 3334, 3444, . . .. Câte cifre de<br />

3 are numărul de pe locul 2008<br />

(Clasa a IV-a)<br />

Petru Asaftei, Iaşi<br />

Clasa a V-a<br />

V.102. Un întreprinzător doreşte să cumpere un număr de frigidere de la un<br />

angrosist, pe care urmează să le transporte către firma sa cu ajutorul unui camion<br />

de mare tonaj, care consumă 10 l de motorină la 100 km (1l de motorină costă 3 lei).<br />

Întreprinzătorul poate opta între doi furnizori: A vinde frigiderul cu 1000 lei/buc.,<br />

1 Se primesc soluţii până la data de 31 decembrie 2009.<br />

71


iar B vinde acelaşi produs cu 990 lei/buc., însă are depozitul mai departe decât A, la<br />

o distanţă pe şosea AB = 150 km.<br />

a) Dacă întreprinzătorul doreşte să cumpere 20 de frigidere, ce furnizor va alege<br />

b) La ce număr de frigidere, costurile de achiziţie nu depind de furnizor<br />

Marian Ciuperceanu, Craiova<br />

V.103. Se consideră numerele naturale m = 3x + 5<br />

2x + 2 , a = 2y + 5 , b = 5z + 2 ,<br />

3<br />

5<br />

unde x, y, z ∈ N. Demonstraţi că m nu poate fi divizor al lui a, dar poate fi divizor<br />

al lui b.<br />

Claudiu Ştefan Popa, Iaşi<br />

V.104. Scrieţi numărul 2008 ca sumă de trei cuburi perfecte. (Găsiţi toate<br />

posibilităţile!)<br />

Veronica Plăeşu şi Dan Plăeşu, Iaşi<br />

V.105. Se consideră numărul a = 7 + 7 2 + 7 3 + . . . + 7 2009 .<br />

a) Demonstraţi că a nu poate fi pătrat perfect.<br />

b) Aflaţi restul împărţirii lui a la 400.<br />

Damian Marinescu, Târgovişte<br />

V.106. Să se determine numărul natural a şi cifra b, dacă (a + 3) · 200b = a · 2009.<br />

Enache Pătraşcu, Focşani<br />

V.107. Dacă n ∈ N\{0, 1} este dat, determinaţi x, y ∈ N ∗ pentru care x(x + 2y +<br />

1) = 2 n · 135.<br />

Petru Asaftei, Iaşi<br />

V.108. Pe tablă sunt scrise numerele 2, 0, 0, 9. Putem şterge de pe tablă oricare<br />

două numere, scriind în loc succesorii acestora. Este posibil ca, în urma mai multor<br />

operaţii de acest fel, să obţinem patru numere egale<br />

Cătălin Budeanu, Iaşi<br />

Clasa a VI-a<br />

VI.102. O asociaţie de locatari este <strong>format</strong>ă din trei familii care au consumat<br />

într-o lună 27m 3 , 16m 3 , respectiv 4m 3 de apă potabilă. Din consumul total, pentru<br />

38m 3 de apă trebuie plătită o taxă de canalizare, care se împarte proporţional cu<br />

consumul fiecărei familii. Dacă preţul apei este de 1,6 lei/m 3 , taxa de canalizare este<br />

de 0,56 lei/m 3 şi fiecărei sume i se aplică T.V.A. de 19 %, aflaţi ce sumă trebuie să<br />

plătească fiecare familie (efectuaţi calculele cu două zecimale exacte).<br />

Petru Asaftei, Iaşi<br />

VI.103. Să se determine numărul prim p şi numerele întregi a şi x pentru care<br />

(x − a)(x − 1)(a − 1) = p.<br />

Gheorghe Iurea, Iaşi<br />

VI.104. Determinaţi numerele prime p şi q, ştiind că există x, y ∈ N ∗ astfel încât<br />

x 2 + y 2 = p, iar x + y + 1 = q.<br />

Andrei Cozma, elev, Bucureşti<br />

VI.105. Să se arate că numărul N = 3 32009 − 3 32008 se poate scrie ca produs a trei<br />

numere naturale consecutive.<br />

Dan Nedeianu, Drobeta-Tr. Severin<br />

72


VI.106. Se consideră unghiulÔxOy şi punctele A, B ∈ (Ox, C, D ∈ (Oy astfel încât<br />

A ∈ (OB), iar C ∈ (OD). Mediatoarele segmentelor [AB] şi [CD] se intersectează în<br />

S, iarÕSAB ≡ÕSCD.<br />

a) Demonstraţi că BC = AD.<br />

b) Dacă, în plus, punctele B, D şi S sunt coliniare, iar m(ÕSAB) = 60 ◦ , arătaţi că<br />

AC⊥SC ⇔ BS = 2 · SD.<br />

Romanţa Ghiţă şi Ioan Ghiţă, Blaj<br />

VI.107. Se consideră A, B, C, D, E, F şase puncte în plan astfel încât AB =<br />

CD = CF = DF = 3cm, BC = BE = CE = 5cm, iar AD = 11cm. Stabiliţi câte<br />

drepte determină cele şase puncte.<br />

Gabriel Popa, Iaşi<br />

VI.108. Un ogar situat în vârful A al unei curţi dreptunghiulare ABCD (AB =<br />

80m, BC = 160m), porneşte în urmărirea a trei iepuri aflaţi în B, C şi D, alergând<br />

de-a lungul gardurilor. Dacă viteza ogarului este 4m/s, iar vitezele iepurilor sunt<br />

3m/s, aflaţi după cât timp reuşeşte ogarul să prindă fiecare iepure.<br />

Marian Ciuperceanu, Craiova<br />

Clasa a VII-a<br />

VII.102. În urma unui război dus între două triburi de canibali, în mâinile<br />

învingătorilor rămân zece prizonieri, printre care şi căpetenia învinşilor. Şeful de<br />

trib al învingătorilor alege, pentru prepararea cinei, câţiva prizonieri (măcar unul), la<br />

întâmplare. Care este probabilitatea ca şeful tribului învins să rămână în viaţă<br />

Gabriel Popa, Iaşi<br />

VII.103. Aflaţi numerele întregi x şi y pentru care y − 4x + 6 < 0, 2y − x − 2 > 0<br />

şi 3y + 2x − 24 < 0.<br />

Gheorghe Iurea, Iaşi<br />

VII.104. Spunem că un număr natural are proprietatea (P ) dacă este prim, cel<br />

puţin egal cu 5 şi se poate scrie ca sumă de două pătrate perfecte. Dacă numerele<br />

p 1 , p 2 , . . . , p n au proprietatea (P ), arătaţi că numărul A = p 1 +p 2 +. . .+p n +n 2 −n+2<br />

nu poate fi pătrat perfect.<br />

Cosmin Manea şi Dragoş Petrică, Piteşti<br />

VII.105. Pentru x, y ∈ R, definim a(x, y) = min(2x − y 2 , 2y − x 2 ). Arătaţi că:<br />

a) a(x, y) ≤ 1, ∀x, y ∈ R; b) max{a(x, y)|x, y ∈ R} = 1.<br />

Ovidiu Pop, Satu Mare<br />

VII.106. Se consideră paralelogramul ABCD, E şi F mijloacele laturilor [AB],<br />

respectiv [AD], {G} = CE∩BD, {H} = CF ∩BD, {P } = F G∩BC, {Q} = EH∩CD.<br />

Arătaţi că 3EF = 2P Q.<br />

Mirela Marin, Iaşi<br />

VII.107. Fie ABC un triunghi cu m(ÒC) = 60 ◦ , L proiecţia lui A pe BC, M<br />

proiecţia lui B pe AC, iar D mijlocul lui [AB]. Demonstraţi că triunghiul DML este<br />

echilateral.<br />

Neculai Roman, Mirceşti (Iaşi)<br />

73


VII.108. Considerăm în plan trei cercuri distincte, congruente, ale căror centre<br />

nu sunt coliniare. Construiţi cu rigla şi compasul un cerc la care cercurile date să fie<br />

tangente interior.<br />

Adrian Corduneanu, Iaşi<br />

Clasa a VIII-a<br />

VIII.102. Rezolvaţi în R ecuaţia<br />

x + 2<br />

+<br />

x − 1‹2<br />

x − 2 −<br />

x + 1‹2<br />

26<br />

5 · x2 − 4<br />

x 2 − 1 = 0.<br />

Vasile Chiriac, Bacău<br />

VIII.103. Arătaţi că oricare ar fi n ∈ N ∗ , există m ∈ N ∗ astfel încât n 4 · m + 1<br />

este număr compus.<br />

Lucian Tuţescu şi Ion Vişan, Craiova<br />

VIII.104. Fie x, y, z ∈ R ∗ + astfel încât x 2 y 2 +y 2 z 2 +z 2 x 2 = 3x 2 y 2 z 2 . Demonstraţi<br />

1<br />

că<br />

x 2 + x + 1 + 1<br />

y 2 + y + 1 + 1<br />

z 2 + z + 1 ≤ 1.<br />

Răzvan Ceucă, elev, Iaşi<br />

VIII.105. Determinaţi x, y ∈ N ∗ pentru care x 3 − y 3 = 3xy + 17.<br />

Liviu Smarandache şi Ion Vişan, Craiova<br />

VIII.106. În tetraedul V ABC, avem AB = 4cm, BC = 5cm, CA = 6cm, iar<br />

ariile feţelor V AB, V BC şi V CA sunt egale cu 15√ 7<br />

cm 2 . Calculaţi sinusurile unghiurilorÕAV<br />

B,ÕBV C şiÕCV A.<br />

4<br />

Vlad Emanuel, student, Bucureşti<br />

VIII.107. Fie ABCD un tetraedru, iar m 1 , m 2 şi m 3 lungimile bimedianelor sale.<br />

Demonstraţi că 3(AB 2 + AC 2 + AD 2 + BC 2 + CD 2 + DB 2 ) ≥ 4(m 1 + m 2 + m 3 ) 2 .<br />

D.M. Bătineţu-Giurgiu, Bucureşti<br />

VIII.108. Într-un reper cartezian xOy, se consideră punctele A ij (i, j), unde<br />

1 ≤ i, j ≤ 5. Determinaţi numărul triunghiurilor care au ca vârfuri trei dintre punctele<br />

date.<br />

Gabriel Popa, Iaşi<br />

Clasa a IX-a<br />

IX.96. Determinaţi triunghiurile în care tangentele unghiurilor se exprimă prin<br />

numere naturale. (În legătură cu X.78 din RecMat 1/2007.)<br />

Titu Zvonaru, Comăneşti<br />

IX.97. Demonstraţi că în orice triunghi are loc inegalitatea m 2 ah b h c + m 2 b h ch a +<br />

m 2 ch a h b ≥ 4S 22 + r<br />

2R.<br />

Cătălin Cristea, Craiova<br />

IX.98. Aflaţi a, b, c ∈ R, a ≠ 0, pentru care |ax 2 + bx + c| ≤ x − 1 a‹2<br />

, ∀x ∈ R.<br />

74<br />

Marian Ursărescu, Roman


IX.99. Fie k ∈ [0, 1), n ∈ N ∗ şi numerele α i ∈ R ∗ , β i ∈ R, ε i ∈ {−1, 1}, i = 1, n,<br />

astfel încât ε 1 α 1 + ε 2 α 2 + . . . + ε n α n = 0. Rezolvaţi ecuaţia<br />

|α 1 x + β 1 | + |α 2 x + β 2 | + . . . + |α n x + β n | = k|ε 1 β 1 + ε 2 β 2 + . . . + ε n β n |.<br />

nX<br />

nX<br />

Dumitru Mihalache<br />

nX<br />

şi Gabi Ghidoveanu, Bârlad<br />

IX.100. Fie (a n ) n≥1 şi (b n ) n≥1 două şiruri de numere reale, cu a n ≠ 0, ∀n ≥ 1<br />

şi 3 · (a k b 2 k − a 2 kb k ) = a k!3<br />

− a 3 k, ∀n ≥ 1. Demonstraţi că, pentru orice<br />

k=1<br />

k=1<br />

k=1<br />

n ≥ 1, există α n ∈ {0, 1} astfel încât b n = α n (a 1 +. . .+a n )−(1−α n )(a 1 +. . .+a n−1 ).<br />

Marian Tetiva, Bârlad<br />

Clasa a X-a<br />

X.96. Dacă a, b, c sunt numere reale pozitive cu suma 1, demonstraţi că a b · b c · c a<br />

+ b a · c b · a c ≤ 2(ab + bc + ca).<br />

Dorin Mărghidanu, Craiova<br />

X.97. Fie a, b, c ∈ C ∗ numere complexe distincte astfel încât (a − b) 3 = (b − c) 3 =<br />

(c − a) 3 . Arătaţi că |2a − b − c| = |2b − c − a| = |2c − a − b|.<br />

Dan Nedeianu, Drobeta-Tr. Severin<br />

X.98. Fie A i (z i ), i = 1, 3 vârfurile unui triunghi din planul xOy şi P (z) un punct<br />

din acest plan (z i şi z sunt afixele punctelor A i , respectiv P ). Să se arate că P este<br />

situat în interiorul triunghiului A 1 A 2 A 3 sau pe una din laturile sale dacă şi numai<br />

dacă există α i ≥ 0, i = 1, 3, astfel încât α 1 + α 2 + α 3 = 1 şi z = α 1 z 1 + α 2 z 2 + α 3 z 3 .<br />

Adrian Corduneanu, Iaşi<br />

X.99. Considerăm triunghiurile echilaterale ABC şi A 1 B 1 C 1 şi construim triunghiurile<br />

echilaterale AA 1 A 2 , BB 1 B 2 , CC 1 C 2 , AB 1 A 3 , BC 1 B 3 , CA 1 A 3 , AC 1 A 4 ,<br />

BA 1 B 4 şi CB 1 C 4 ; toate triunghiurile citate sunt orientate pozitiv. Fie punctele<br />

M 2 ∈ A 2 B, N 2 ∈ B 2 C, P 2 ∈ C 2 A, M 3 ∈ A 3 B, N 3 ∈ B 3 C, P 3 ∈ C 3 A, M 4 ∈ A 4 B,<br />

N 4 ∈ B 4 C şi P 4 ∈ C 4 A astfel încât M 2A 2<br />

M 2 B = N 2B 2<br />

N 2 C = P 2C 2<br />

P 2 A = M 3A 3<br />

M 3 B = N 3B 3<br />

N 3 C =<br />

P 3 C 3<br />

P 3 A = M 4A 4<br />

M 4 B = N 4B 4<br />

N 4 C = P 4C 4<br />

P 4 A . Demonstraţi că triunghiurile M 2N 2 P 2 , M 3 N 3 P 3 şi<br />

M 4 N 4 P 4 sunt echilaterale şi au acelaşi centru.<br />

Cătălin Ţigăeru, Suceava<br />

X.100. Demonstraţi că în orice triunghi ABC are loc inegalitatea<br />

1<br />

sin 2 A(sin B + sin C) + 1<br />

2 sin 2 B(sin C + sin A) + 1<br />

2 sin 2 C(sin A + sin B) ≥ 4 2 3 .<br />

Marius Olteanu, Rm. Vâlcea<br />

Clasa a XI-a<br />

XI.96. Fie ε rădăcina primitivă de ordin trei a unităţii, iar A, B ∈ M 3 (R) cu<br />

det(A + εB) = 0. Demonstraţi că det(A − B) = det A − det B.<br />

Dan Popescu, Suceava<br />

75


XI.97. Fie n ≥ 3 un număr natural. Arătaţi că pentru orice k ∈ {2, 3, . . . , n − 1},<br />

există A ∈ M n ({0, 1}) astfel încât A p ≠ I n , ∀p ∈ {1, 2, . . . , k − 1} şi A k = I n .<br />

Gheorghe Iurea, Iaşi<br />

XI.98. Demonstraţi că funcţia f :0, π − cos x<br />

f(x) = lnÉ1<br />

2→R,<br />

1 + cos x este<br />

concavă şi, folosind eventual acest lucru, arătaţi că în orice triunghi ascuţitunghic<br />

ABC are loc inegalitatea 1 − cos A<br />

1 + cos A · 1 − cos B<br />

1 + cos B · 1 − cos C<br />

1 + cos C ≤ 1<br />

27 .<br />

Bogdan Victor Grigoriu, Fălticeni<br />

XI.99. Studiaţi convergenţa şirului (v n ) n≥1 definit prin v n+1 = (vc n + d) 1/c<br />

v n<br />

,<br />

∀n ≥ 1, unde v 1 , c şi d sunt numere reale pozitive date.<br />

Gheorghe Costovici şi Adrian Corduneanu, Iaşi<br />

XI.100. Demonstraţi că<br />

(x + 1) sin<br />

π<br />

x + 1 − cos π<br />

x + 1‹ 0, b > 0. Determinaţi parabola y = λx 2 + µ care este tangentă<br />

în T la AB, ştiind că aria suprafeţei determinată de parabolă şi axele de coordonate<br />

este maximă.<br />

Adrian Corduneanu, Iaşi<br />

76


Probleme pentru pregătirea concursurilor<br />

A. Nivel gimnazial<br />

G156. Dacă a, b, c ∈ R ∗ +,<br />

b 2 + 1<br />

√<br />

b2 − b + 1 + c 2 + 1<br />

√<br />

c2 − c + 1 ≥ 6.<br />

1<br />

a + 1 b + 1 c ≤ 3, demonstraţi că a 2 + 1<br />

√<br />

a2 − a + 1 +<br />

I.V. Maftei, Bucureşti şi Mihai Haivas, Iaşi<br />

G157. Spunem că un număr natural are proprietatea (P ) dacă se poate scrie ca<br />

sumă a trei pătrate perfecte nenule şi că are proprietatea (Q) dacă se poate scrie ca<br />

sumă a patru pătrate perfecte nenule.<br />

a) Daţi exemple de numere naturale care au: numai proprietatea (P ); numai<br />

proprietatea (Q); atât proprietatea (P ) cât şi proprietatea (Q).<br />

b) Dacă a, b, c ∈ N ∗ au suma pară şi oricare dintre ele este diferit de suma celorlaltor<br />

două, demonstraţi că numărul a 2 + b 2 + c 2 are proprietatea (Q).<br />

Ovidiu Pop, Satu Mare<br />

G158. Se consideră ecuaţia x 2 +y 2 +z 2 = (x−y) 2 +(y −z) 2 +(z −x) 2 , x, y, z ∈ N.<br />

a) Arătaţi că ecuaţia are o infinitate de soluţii.<br />

b) Dacă (x, y, z) este soluţie a ecuaţiei, demonstraţi că fiecare dintre numerele<br />

xy, yz, zx şi xy + yz + zx este pătrat perfect.<br />

Liviu Smarandache,<br />

=§<br />

Craiova<br />

G159. Aflaţi ultimele două cifre ale numerelor (70n + 6) · 6 n−1 , n ∈ N.<br />

Ion Săcăleanu, Hârlău<br />

a<br />

G160. Se consideră mulţimile A = {1, 2, 3, . . . , 2009}, B<br />

=§<br />

c<br />

d<br />

b, c, d ∈ A, a, b, c, d distincteªşi C<br />

c + da,<br />

a + d +<br />

d<br />

b + d +<br />

b<br />

b + d +<br />

a + d +<br />

d b, c, d ∈ A,<br />

c + da,<br />

a, b, c, d distincteª. Determinaţi A ∩ B ∩ C. (În legătură cu E: 13650 din G.M. 5-<br />

6/2008.)<br />

Andrei Crăcană, elev, Iaşi<br />

G161. Fie M mulţimea numerelor de forma abc, cu a · b · c ≠ 0. Determinaţi<br />

cardinalul maxim al unei submulţimi N a lui M astfel încât x + y ≠ 1109, ∀x, y ∈ N.<br />

Petru Asaftei şi Gabriel Popa, Iaşi<br />

G162. Putem înlocui un triplet de numere întregi (a, b, c) cu unul dintre tripletele<br />

(2b + 2c − a, b, c), (a, 2a + 2c − b, c) sau (a, b, 2a + 2b − c). Arătaţi că dacă pornim<br />

de la tripletul (31329, 24025, 110224) şi efectuăm succesiv asemenea înlocuiri, se obţin<br />

mereu triplete <strong>format</strong>e numai din pătrate perfecte.<br />

Marian Tetiva, Bârlad<br />

G163. Fie ABC un triunghi cu m(bA) ≠ 90 ◦ şi punctele B 1 ∈ (AC) şi C 1 ∈ (AB).<br />

Arătaţi că axa radicală a cercurilor de diametre [BB 1 ] şi [CC 1 ] trece prin punctul A<br />

dacă şi numai dacă B 1 C 1 ∥BC.<br />

Neculai Roman, Mirceşti (Iaşi)<br />

77


G164. Fie B, b numere reale date, cu B > b > 0. Dintre toate trapezele circumscriptibile<br />

care au lungimile bazelor B şi b, determinaţi-l pe cel de arie maximă.<br />

Claudiu Ştefan Popa, Iaşi<br />

G165. Fie ABC un triunghi isoscel (AB = AC), M mijlocul laturii [BC], iar P<br />

un punct în interiorul triunghiului ABM. Notăm {D} = BP ∩ AC, {E} = CP ∩ AB.<br />

Demonstraţi că BE < CD şi P E < P D.<br />

Cristian Pravăţ, Iaşi şi Titu Zvonaru, Comăneşti<br />

B. Nivel liceal<br />

L156. Fie M un punct exterior cercului C de centru O şi rază R. Notăm cu T 1 , T 2<br />

punctele de contact cu cercul ale tangentelor duse din M la C şi cu A punctul de<br />

intersecţie a dreptei OM cu cercul C, astfel încât A /∈ [OM]. Determinaţi punctele<br />

M cu proprietatea că se poate construi un triunghi cu segmentele [MT 1 ], [MT 2 ] şi<br />

[MO], dar nu se poate construi un triunghi cu [MT 1 ], [MT 2 ] şi [MA].<br />

Temistocle Bîrsan, Iaşi<br />

L157. În planul △ABC definim transformarea P → P ′ astfel: 1. punctul P se<br />

proiectează pe dreptele BC, CA, AB în D, E şi respectiv F ; 2. simetricele punctelor<br />

D, E, F în raport cu mijloacelor laturilor [BC], [CA] şi respectiv [AB] se notează<br />

D ′ , E ′ , F ′ ; 3. P ′ este punctul de concurenţă a perpendicularelor în D ′ , E ′ , F ′ pe<br />

BC, CA şi respectiv AB. Arătaţi că transformarea P → P ′ coincide cu simetria în<br />

raport cu O, centrul cercului circumscris △ABC.<br />

Temistocle Bîrsan, Iaşi<br />

L158. În interiorul triunghiului ABC cu latura [BC] fixă şi vârful A mobil, considerăm<br />

punctul T asfel încâtÕAT B ≡ÕBT C ≡ÕCT A. Determinaţi poziţia punctului<br />

A în planul triunghiului pentru care m(ÕBAC) = α < 5π , iar suma distanţelor de la<br />

6<br />

T la vârfurile triunghiului este maximă.<br />

Cătălin Calistru, Iaşi<br />

L159. Dacă a, b, c ∈ R ∗ + şi x ∈0, π 2, demonstraţi inegalitatea<br />

a<br />

sin x<br />

x<br />

‹3<br />

+ b<br />

sin x<br />

x<br />

‹2<br />

+ c<br />

sin x‹+3 3√ abc<br />

x<br />

L160. Demonstraţi că în orice triunghi are loc inegalitatea<br />

m a + m b + m c ≥ 6r<br />

m a<br />

m b + m c<br />

+<br />

tg x<br />

x<br />

‹>6 · 3√ abc.<br />

D.M. Bătineţu-Giurgiu, Bucureşti<br />

m b<br />

m a + m c<br />

+<br />

m c<br />

m a + m b‹≥9r.<br />

Marius Olteanu, Rm. Vâlcea<br />

L161. Dacă a, b, c ∈ R ∗ + şi a + b + c = 1, demonstraţi inegalitatea<br />

1<br />

1 + a‹.<br />

3 +X(a − b) 2 + (a − c) 2<br />

1 + a<br />

≤ 4(a 2 + b 2 + c 2 )<br />

X<br />

Titu Zvonaru, Comăneşti<br />

78


L162. Dacă n ∈ Z ∗ este fixat, rezolvaţi în R ecuaţiahx<br />

ni=•[x]<br />

n˜.<br />

Dumitru Mihalache şi Gabi Ghidoveanu, Bârlad<br />

L163. Fie a un număr întreg impar, iar n ∈ N ∗ . Arătaţi că polinomul X 2n + a 2n<br />

este ireductibil în Z[X] însă, pentru orice număr prim p, polinomul redus modulo p<br />

este reductibil în Z p [X].<br />

Dorel Miheţ, Timişoara<br />

L164. O secvenţă x 1 , x 2 , . . . , x n , y 1 , y 2 , . . . , y n de 2n numere reale are proprietatea<br />

(P ) dacă x 2 i + y2 i = 1, ∀i = 1, n. Fie n ∈ N ∗ astfel încât pentru orice secvenţă cu<br />

proprietatea (P ), există 1 ≤ i < j ≤ n cu x i x j + y i y j ≥ 0, 947. Determinaţi cea mai<br />

bună constantă α aşa încât x i x j + y i y j ≥ α, pentru orice secvenţă cu proprietatea<br />

(P ).<br />

Vlad Emanuel, student, Bucureşti<br />

L165. Fie n ≥ 2 un număr natural. Determinaţi cel mai mare număr natural<br />

m pentru care există submulţimile nevide şi distincte A 1 , A 2 , . . . , A m ale lui A =<br />

{1, 2, . . . , n}, cu proprietatea că fiecare element al lui A este conţinut în cel mult k<br />

dintre ele, unde:<br />

a) k = 2; b) k = n; c) k = n + 1.<br />

Marian Tetiva, Bârlad<br />

Training problems for mathematical contests<br />

A. Junior highschool level<br />

G156. If a, b, c ∈ R ∗ +, 1 a + 1 b + 1 c<br />

≤ 3, prove that<br />

a 2 + 1<br />

√<br />

a2 − a + 1 + b 2 + 1<br />

√<br />

b2 − b + 1 + c 2 + 1<br />

√<br />

c2 − c + 1 ≥ 6.<br />

I.V. Maftei, Bucureşti and Mihai Haivas, Iaşi<br />

G157. We say that a natural number has the property (P ) if it can be written<br />

as the sum of three nonzero perfect squares, and it has the property (Q) if it can be<br />

written as the sum of four nonzero perfect squares.<br />

a) Give examples of natural numbers that have: property (P ) only; property (Q)<br />

only; both property (P ) and property (Q).<br />

b) If the numbers a, b, c ∈ N ∗ have an even sum and each of them differs from the<br />

sum of the other two numbers, show that a 2 + b 2 + c 2 has the property (Q).<br />

Ovidiu Pop, Satu Mare<br />

G158. The equation x 2 + y 2 + z 2 = (x − y) 2 + (y − x) 2 + (z − x) 2 with x, y, z ∈ N<br />

is considered.<br />

a) Show that this equation has infinitely many solutions.<br />

b) If (x, y, z) is a solution to the equation, show that each of the numbers x y, y z,<br />

z x and x y + y z + z x is a perfect square.<br />

Liviu Smarandache, Craiova<br />

79


=§<br />

G159. Find the last two digits of the number (70n + 6) · 6 n−1 .<br />

Ion Săcăleanu, Hârlău<br />

a<br />

G160. Three sets are considered, namely: A = {1, 2, 3, . . . , 2009}; B<br />

b<br />

b + d +<br />

c<br />

c + da, b, c, d ∈ A and mutually distinctª; C =§<br />

d<br />

a + d +<br />

d<br />

b + d +<br />

a + d +<br />

d<br />

c + da,<br />

b, c, d ∈ A and mutually distinctª. Determine A ∩ B ∩ C. (The problem is connected<br />

with E: 13650 of Gazeta Matematică 5-6/2008).<br />

Andrei Crăcană, highschool student, Iaşi<br />

G161. Let M be the set of numbers of the form abc with a · b · c ≠ 0. Determine<br />

the maximal cardinal number of a subset N of M such that x + y ≠ 1109, ∀x, y ∈ N.<br />

Petru Asaftei and Gabriel Popa, Iaşi<br />

G162. We may replace the triple of integer numbers (a, b, c) by one of the triples<br />

(2b + 2c − a, b, c) , (a, 2a + 2c − b, c) , (a, b, 2a + 2b − c) . Show that if we start from<br />

the triple (31329, 24025, 110224) and successively aply such replacings only triples<br />

consisting of perfect squares are obtained.<br />

Marian Tetiva, Bârlad<br />

G163. Let ABC be a triangle with m(A) ≠ 90 0 and take the points B 1 ∈ (AC)<br />

and C 1 ∈ (AB). Prove that the radical axis of the circles of diameters [BB 1 ] and<br />

[CC 1 ] passes through the point A if and only if B 1 C 1 ∥ BC.<br />

Neculai Roman, Mirceşti (Iaşi)<br />

G164. Let B, b be given real numbers with B > b > 0. Among all the circumscriptible<br />

trapesiums with the lengths of their bases (respectively) equal to B and b<br />

select the one of maximum area.<br />

Claudiu Ştefan Popa, Iaşi<br />

G165. Let ABC be an isosceles triangle (AB = AC), M the mid-point of the side<br />

[BC] , and P a point in the interior of triangle ABM. We denote {D} = BP ∩ AC,<br />

{E} = CP ∩ AB. Prove that BE < CD and P E < P D.<br />

Cristian Pravăţ, Iaşi and Titu Zvoranu, Comăneşti<br />

B. Highschool level<br />

L156. Let M be a point that is exterior to the circle C of center O and radius<br />

R. We denote by T 1 , T 2 the contact points, with this circle, of the tangents from M<br />

to C, and let A be the intersection point of the straight line OM with circle C such<br />

that A /∈ [OM]. Determine the points M with the property that a triangle can be<br />

built with the line segments [MT 1 ], [MT 2 ] and [MO] as its sides, while a triangle<br />

with [MT 1 ], [MT 2 ] and [MA] as its sides cannot be built.<br />

Temistocle Bîrsan, Iaşi<br />

L157. In the plane of △ABC, we define the trans<strong>format</strong>ion P → P ′ as follows:<br />

1 0 the point P is projected onto the lines BC, CA, AB at the points D, E and<br />

respectively F ; 2 0 the symmetric points of D, E, F with respect to the mid-points of<br />

the sides [BC], [CA], and respectively [AB] are denoted as D ′ , E ′ , F ′ ; 3 0 P ′ is the<br />

80


common (or meeting) point of the perpendicular lines at D ′ , E ′ , F ′ on BC, CA and<br />

respectively AB. Show that the trans<strong>format</strong>ion P → P ′ coincides with the symmetry<br />

with respect to O − the center of the circumscribed circle to △ABC.<br />

Temistocle Bîrsan, Iaşi<br />

L158.We consider the point T in the interior of triangle ABC with its side<br />

[BC] fixed and its vertex A mobile such thatÕAT B =ÕBT C =ÕCT A . Determine the<br />

position of the point A in the plane of the triangle such that m(ÕBAC) = α < 5π 6 ,<br />

and the sum of the distances from T to the vertices of the triangle is maximum.<br />

Cătălin Calistru, Iaşi<br />

L159. If a, b, c ∈ R ∗ + and x ∈0, π 2, prove the inequality<br />

a<br />

sin x<br />

x<br />

‹3<br />

+ b<br />

sin x<br />

x<br />

‹2<br />

+ c<br />

sin x‹+3 3√ abc<br />

x<br />

tan x‹≥6 · 3√ abc.<br />

x<br />

D.M. Bătineţu-Giurgiu, Bucureşti<br />

L160. Prove that the following inequality holds in any triangle:<br />

m a m b m c<br />

m a + m b + m c ≥ 6 r<br />

+ +<br />

r.<br />

m b + m c m a + m c m a + m b‹≥9<br />

Marius XOlteanu, Rm. Vâlcea<br />

L161. If a, b, c ∈ R ∗ + and a + b + c = 1, prove the inequality<br />

− b)<br />

3 +X(a 2 + (a − c) 2<br />

≤4(a 2 + b 2 + c 2 1<br />

)<br />

1 + a<br />

1 + a‹.<br />

Titu Zvoranu, Comăneşti<br />

L162. If n ∈ Z ∗ is fixed, solve in R the equationhx<br />

ni=•[x]<br />

n˜.<br />

Dumitru Mihalache and Gabi Ghidoveanu, Bârlad<br />

L163. Let a be an odd number and let n be a nonzero number. Prove that the<br />

polynomial X 2n + a 2n is irreducible in Z[X], while it factors modulo f for all prime<br />

p.<br />

Dorel Miheţ, Timişoara<br />

L164. A sequence x 1 , x 2 , . . . , x n , y 1 , y 2 , . . . , y n of 2 n real numbers is said to<br />

have the property (P ) if x 2 i + y2 i = 1, ∀i = 1, n. Let n ∈ N ∗ such that for any<br />

sequence with property (P ) there are subscripts i, j with 1 ≤ i < j ≤ n such that<br />

x i<br />

x j + y i y j ≥ 0.947. Determine the best constant α such that x i<br />

x j + y i y j ≥ α, for<br />

any sequence with property (P ).<br />

Vlad Emanuel, student, Bucureşti<br />

L165. Let n ≥ 2 be a natural number. Determine the largest natural number<br />

m such that m nonempty and distinct subsets A 1 , A 2 , . . . , A m of A = {1, 2, . . . , n}<br />

exist with the property that each element of A belongs to at most k such subsets,<br />

where:<br />

a) k = 2; b) k = n; c) k = n + 1.<br />

Marian Tetiva, Bârlad<br />

81


Pagina rezolvitorilor<br />

CRAIOVA<br />

Colegiul Naţional ”Fraţii Buzeşti”. Clasa a VI-a (prof. TUŢESCU Lucian).<br />

ENE Cristina-Elena: V(95-99); Clasa a VIII-a (prof. TUŢESCU Lucian). STĂNI-<br />

CIOIU Nicu: VIII(95,96), IX.94, X(91,93); Clasa a IX-a (prof. TUŢESCU Lucian).<br />

DOBRESCU Lorena Roberta: VIII (95,96), IX.94, X(91,93,95), G146; RADU Noela:<br />

VIII(95,96,100), IX(93,94), X.95, G146; Clasa a X-a (prof. TUŢESCU Lucian).<br />

BORUZ Ana-Maria: VIII.96, IX.94, X(91,93,95), XI.91, G146; DOBRESCU Cristian<br />

Bogdan: VIII.96, X(91,93,95), G146; VRĂJITOAREA Marius-Andrei: X(91,93,95),<br />

XI(91,92).<br />

GALAŢI<br />

Colegiul Naţional ”V. Alecsandri”. Clasa a IX-a. STAN Georgiana: VII(95,98),<br />

VIII(95,96,100).<br />

HÂRLĂU<br />

Liceul Teoretic ”Ştefan cel Mare”. Clasa a V-a (prof. SĂCĂLEANU Ioan).<br />

BOBÎRNĂ Petru Costin: P(151-153), V(88,89), VI.90; RUGINĂ Rareş Teodor: P(151-<br />

153), V.88, VI.90; SĂCĂLEANU Emilian Gabriel: P(151-153), V.88, V.89; Clasa a<br />

VI-a (prof. SĂCĂLEANU Ioan). NEICU Mara: P.151, V(88-90), VI.88; CĂLINES-<br />

CU Ana Ioana: P(151,153), V(88,93), VI(88,90), VII.91; MITITELU Melissa Florina:<br />

P(151,153), V(88,93), VI.88; Clasa a VII-a (prof. SĂCĂLEANU Ioan). BARĂU<br />

Larisa Ionela: V(88,89), VI(88,93,94).<br />

IAŞI<br />

Şcoala nr. 14 ”Gh. Mârzescu”. Clasa a IV-a (inst. NUŢĂ Elena). BACIU<br />

Tudor: P(154-158,161); CHIRILUŢĂ George Ştefan: P(154-158,161); POSTUDOR<br />

Georgiana Mădălina: P(154-158,161); STOICA Adriana: P(154-158,161).<br />

Şcoala nr. 26 ”G. Coşbuc”. Clasa a III-a (înv. BUCATARIU RICA). CHIRIAC<br />

Alexandra: P(144-150); IVANOV Alexandra: P(144-146; 148-150); MÎNDRU Liana:<br />

P(144-150). Clasa a IV-a (inst. RACU Maria). APACHIŢEI Aura-Georgiana:<br />

P(154-160); BURA Emma-Andreea: P(154-160); CRĂCIUN Ioana-Daniela: P(154-<br />

160); FILIP Ingrid-Ştefania: P(154-160); HRISCU Ovidiu-Constatin: P(154-157,159,<br />

160), HUZA Mădălina: P(154-158,160); LEŞOVSCHI Alexandra-Ioana: P(154-157,<br />

159,160); LUPU Roxana-Elena P(154-158,161); MAXIM Alexandra-Camelia: P(154-<br />

160); TUDOSE Ema-Alina P(154-158); ŢUCĂ Cosmin: P(155-160); VASILE Bogdan-<br />

Andrei: P(154-158,160). Clasa a IV-a (înv. HRIMIUC Valeria). BRUMĂ Andrei-<br />

Alexandru: P(154-161); DU<strong>MB</strong>RAVĂ Bianca: P(154-161); HARAPCIUC Eduard-<br />

Gabriel: P(154-161); MANTALEA Alex-Adrian: P(154-161); OLARU Alexandra:<br />

P(154-161); VORNICU Sorin: P(154-161).<br />

Colegiul Naţional Iaşi. Clasa a VI-a (prof. POPA Gabriel). STOLERU Georgiana<br />

Ingrid: V(95-99); ŞTREANGĂ Iulia: V(95-99).<br />

Colegiul Naţional ”C. Negruzzi”. Clasa a VI-a (prof. ILIE Gheorghe). SUFRA-<br />

GIU Călin: V(95-101), VI(95-98), VII(98,101). Clasa a VIII-a (prof. SAVA Radu).<br />

IONIŢĂ Norbert-Traian: VI(89,94), VII(88-94), VIII(88-90,92), G(139,140,142).<br />

82


Clasa a VIII-a (prof. IONESEI Silviana). PĂVĂLOI Alexandru: VII(96,98-100),<br />

VIII(95-97,100).<br />

SFÂNTU GHEORGHE (Tulcea)<br />

Şcoala generală Sfântu Gheorghe. Clasa a IV-a (înv. GAVRILĂ Elena). BĂLAN<br />

Silviu: P(144-147), P(150-153); CLADIADE Bogdan Robert: P(144,146,147,150-<br />

153); CUCU Delia: P(144-147,150-153); EFIMOV Cosmin: P(144-147,150-153). Clasa<br />

a VI-a (prof. SĂILEANU Sorin). SIDORENCU Andrei: V(89,90,93), VI(88-90,92,93),<br />

VII(88-94).<br />

SUCEAVA<br />

Şcoala cu clasele I-VIII nr. 3. Clasa a IV-a (înv. TABARCEA Silvestru).<br />

FECHET Ştefan: P(147-151). Clasa a V-a (prof. APOSTOL Geta). FECHET<br />

Mircea: P(151-153), V(88-90).<br />

Elevi rezolvitori premiaţi<br />

Şcoala nr. 14 ”Gh. Mârzescu”, Iaşi<br />

POSTUDOR Georgiana-Mădălina (cl. a IV-a): 1/2008(6pb), 2/2008(5pb),<br />

1/2009(6pb).<br />

Şcoala nr. 26 ”G. Coşbuc”, Iaşi<br />

APACHIŢEI Aura Georgiana (cl. a IV-a): 1/2008(6pb), 2/2008(6pb), 1/2009(7pb);<br />

BURA Emma-Andreea (cl. a IV-a): 1/2008(6pb), 2/2008(6pb), 1/2009(7pb);<br />

FILIP Ingrid-Ştefania (cl. a IV-a): 1/2008(6pb), 2/2008(6pb), 1/2009(7pb);<br />

HRISCU Ovidiu-Constantin (cl. a IV-a): 1/2008(6pb), 2/2008(7pb), 1/2009(6pb);<br />

HUZA Mădălina (cl. a IV-a): 1/2008(6pb), 2/2009(7pb), 1/2009(6pb);<br />

MAXIM Alexandra-Camelia (cl. a IV-a): 1/2008(6pb), 2/2008(6pb), 1/2009(7pb);<br />

TUDOSE Ema-Alina (cl. a IV-a): 1/2008(6pb), 2/2008(7pb), 1/2009(5pb);<br />

VASILE Bogdan-Andrei (cl. a IV-a): 1/2008(6pb), 2/2008(6pb), 1/2009(6pb);<br />

BRUMĂ Andrei-Alexandru (cl. a IV-a): 1/2008(6pb), 2/2008(7pb), 1/2009(8pb);<br />

DU<strong>MB</strong>RAVĂ Bianca (cl. a IV-a): 1/2008(6pb), 2/2008(7pb), 1/2009(8pb);<br />

HARAPCIUC Eduard-Gabriel (cl. a IV-a): 1/2008(6pb), 2/2008(7pb),<br />

1/2009(8pb);<br />

MANTALEA Alex-Adrian (cl a IV-a): 1/2008(6pb), 2/2008(7pb), 1/2009(8pb);<br />

OLARU Alexandra (cl a IV-a): 1/2008(6pb), 2/2008(7pb), 1/2009(8pb).<br />

Şcoala cu clasele I-VIII, nr. 3, Suceava<br />

FECHET Ştefan (cl a IV-a): 1/2008(5pb), 2/2008(6pb), 1/2009(5pb),<br />

FECHET Mircea (cl. a V-a): 1/2008(7pb), 2/2008(9pb), 1/2009(6pb).<br />

Colegiul Naţional Iaşi, Iaşi<br />

STOLERU Georgiana Ingrid (cl. a VI-a): 1/2008(7pb), 2/2008(5pb),<br />

1/2009(5pb).<br />

Colegiul Naţional ”C. Negruzzi”, Iaşi<br />

IONIŢĂ Norbert-Traian (cl. a VIII-a): 1/2008(13pb), 2/2008(7pb), 1/2009(16pb).<br />

83


<strong>Revista</strong> semestrială RECREAŢII MATEMATICE este editată de<br />

ASOCIAŢIA “RECREAŢII MATEMATICE”. Apare la datele de 1 martie şi<br />

1 septembrie şi se adresează elevilor, profesorilor, studenţilor şi tuturor celor pasionaţi<br />

de matematica elementară.<br />

În atenţia tuturor colaboratorilor<br />

Materialele trimise redacţiei spre publicare (note şi articole, chestiuni de metodică,<br />

probleme propuse etc.) trebuie prezentate îngrijit, clar şi concis; ele trebuie să prezinte<br />

interes pentru un cerc cât mai larg de cititori. Se recomandă ca textele să nu depăşească<br />

patru pagini. Evident, ele trebuie să fie originale şi să nu fi apărut sau să fi fost<br />

trimise spre publicare altor reviste. Rugăm ca materialele tehnoredactate să fie<br />

însoţite de fişierele lor.<br />

Problemele destinate rubricilor: Probleme propuse şi Probleme pentru<br />

pregătirea concursurilor vor fi redactate pe foi separate cu enunţ şi demonstraţie/rezolvare<br />

(câte una pe fiecare foaie) şi vor fi însoţite de numele autorului, şcoala şi<br />

localitatea unde lucrează/învaţă.<br />

Redacţia va decide asupra oportunităţii publicării materialelor primite.<br />

În atenţia elevilor<br />

Numele elevilor ce vor trimite redacţiei soluţii corecte la problemele din rubricile<br />

de Probleme propuse şi Probleme pentru pregatirea concursurilor vor fi menţionate<br />

în Pagina rezolvitorilor. Elevii menţionaţi de trei ori vor primi o diplomă şi un<br />

premiu în cărţi. Elevii rezolvitori vor ţine seama de regulile:<br />

1. Pot trimite soluţii la minimum cinci probleme propuse în numărul prezent<br />

şi cel anterior al revistei (pe o foaie va fi redactată o singură problemă).<br />

2. Elevii din clasele VI-XII au dreptul să trimită soluţii la problemele propuse<br />

pentru clasa lor, pentru orice clasă mai mare, din două clase mai mici şi imediat<br />

anterioare. Cei din clasa a V-a pot trimite soluţii la problemele propuse pentru clasele a<br />

IV-a, a V-a şi orice clasă mai mare, iar elevii claselor I-IV pot trimite soluţii la<br />

problemele propuse pentru oricare din clasele primare şi orice clasă mai mare. Orice<br />

elev poate trimite soluţii la problemele de concurs (tip G şi L).<br />

3. Vor fi menţionate următoarele date personale: numele şi prenumele, clasa,<br />

şcoala şi localitatea, precum şi numele profesorului cu care învaţă.<br />

4. Plicul cu probleme rezolvate se va trimite prin poştă (sau va fi adus direct) la<br />

adresa Redacţiei:<br />

Prof. dr. Temistocle Bîrsan<br />

Str. Aurora, nr. 3, sc. D, ap. 6,<br />

700 474, Iaşi<br />

Jud. IAŞI<br />

E-mail: t_birsan@yahoo.com


CUPRINS<br />

O sută de ani de la naşterea Academicianului Nicolae Teodorescu ................................... 1<br />

ARTICOLE ŞI NOTE<br />

M. TETIVA – O problemă de colecţie .................................................................................... 3<br />

T. BÎRSAN – Drepte concurente în conexiune cu punctele I, Г,N ......................................... . 6<br />

F. POPOVICI – Asupra inegalităţii lui Jensen .......................................................................... 12<br />

M. BENCZE – O rafinare a inegalităţii lui Euler R r 2 .................................................. 15<br />

G. HĂVÂRNEANU – Cercuri tangente la două cercuri date ................................................... 17<br />

NOTA ELEVULUI<br />

O. CERRAHOGLU – Aplicaţii ale teoremei lui Van Aubel .................................................... 21<br />

CORESPONDENŢE<br />

A. REISNER – Une application de l’inversion ......................................................................... 23<br />

CHESTIUNI METODICE<br />

C-S. POPA – O demonstraţie simplă a inegalităţii mediilor ..................................................... 25<br />

GH. COSTOVICI – O demonstraţie a teoremei a doua a lui Ptolemeu ................................. 26<br />

D. VĂCARU – Asupra determinării imaginii unei funcţii de mai multe variabile.................. 27<br />

G. POPA – Principiul extremal.................................................................................................. 29<br />

ŞCOLI ŞI DASCĂLI<br />

Profesorul Constantin E. Popa la şaizeci de ani ...................................................................... 33<br />

CONCURSURI ŞI EXAMENE<br />

Concursul "Recreaţii <strong>Matematice</strong>" Ediţia a VI-a, Muncel (Iaşi), 26 august 2008 ..................... 35<br />

Concursul omagial "Recreaţii Ştiinţifice" ................................................................................... 37<br />

PROBLEME ŞI SOLUŢII<br />

Soluţiile problemelor propuse în nr. 1/2008 ............................................................................ 42<br />

Soluţiile problemelor pentru pregătirea concursurilor din nr. 1/2008 .................................. 60<br />

Probleme propuse ........................................................................................................................ 71<br />

Probleme pentru pregătirea concursurilor ................................................................................ 77<br />

Training problems for mathematical contests ......................................................................... 79<br />

Pagina rezolvitorilor ............................................................................................................ 82<br />

Rezolvitorii premiaţi ........................................................................................................... 83<br />

ISSN 1582 – 1765<br />

7 lei

Hooray! Your file is uploaded and ready to be published.

Saved successfully!

Ooh no, something went wrong!